分からない問題はここに書いてね368

このエントリーをはてなブックマークに追加
1132人目の素数さん
さあ、今日も1日頑張ろう★☆

前スレ
分からない問題はここに書いてね367
http://uni.2ch.net/test/read.cgi/math/1331655841/
2132人目の素数さん:2012/04/17(火) 09:52:54.68
前スレ>>995

a(x-di)^2+c=0の形の二次関数の二つの解は di ± √(-ac)/a
なので、これが異なるふたつの実数解を持つときには
ふたつの実数解は共益していると言えるかもしれない。
もちろん、異なるふたつの虚数解になるときには共益しない。

ax^2+cの形の二次関数の二つの解は ± √(-ac)/a だから
解がふたつあるなら、実数解でも虚数解でも共益していると言える。

3132人目の素数さん:2012/04/17(火) 09:54:45.64
× なので、これが異なるふたつの実数解を持つときには 
  ふたつの実数解は共益していると言えるかもしれない。

○ なので、これが異なるふたつの解を持つときには 
  ふたつの実数部を見れば共益していると言えるかもしれない。
4132人目の素数さん:2012/04/17(火) 10:21:10.38
× 共益
○ 共役
5132人目の素数さん:2012/04/17(火) 10:24:13.53
conjugate
6132人目の素数さん:2012/04/17(火) 10:26:03.51
○ 共役
○ 共軛
7132人目の素数さん:2012/04/17(火) 15:09:57.96
一辺が1の正方形と同じ面積の正三角形の一辺を求めよ。
8132人目の素数さん:2012/04/17(火) 15:49:02.15
>>7
1.51967
9132人目の素数さん:2012/04/17(火) 15:54:24.08
三角関数の不定積分

@∫sin ax sin bx dx =

A∫cos ax cos bx dx =

B∫sin ax cos bx dx =
10132人目の素数さん:2012/04/17(火) 16:18:03.17
解析概論読め
11132人目の素数さん:2012/04/17(火) 16:33:55.86
|a/b|=|a|/|b|
を証明せよという問題。

|b|≠0という条件はいらないのでしょうか?
12132人目の素数さん:2012/04/17(火) 17:28:07.02
>>11
b≠0だけ満たしていれば|b|≠0は同時に満たす
13132人目の素数さん:2012/04/17(火) 19:50:55.39
x^2+y^2=1,0≦z≦1の円柱の全表面をSとして、
A=x^2i-xyj+z^2k(i,j,kはそれぞれx,y,z方向の単位ベクトル)の
面積分∬(A・n)dSを求めろ

ガウスの定理を使ってコレを解くとπになるらしい。
当然円柱座標を用いて▽・A=rcosφ+2zを積分しようと思ってるんだけど
rの積分範囲は0から1、φの積分範囲は0から2π、zの積分範囲は0から1
として解いてもπにならない。
どこが間違ってるのか指摘お願いします。
14132人目の素数さん:2012/04/17(火) 20:03:19.84
命令形
15132人目の素数さん:2012/04/17(火) 20:15:03.45
>>14
問題文は転載しましたので、命令形になってしまいました。
申し訳ありません。
16132人目の素数さん:2012/04/17(火) 20:16:33.44
ハミルトンの四元数体の使い道を教えてください
17132人目の素数さん:2012/04/17(火) 20:22:49.77
間違いもクソも計算全部かけやアホ
文もうか
18132人目の素数さん:2012/04/17(火) 20:26:54.20
>>13
方針はすべてあってる。間違いはおそらく最後の積分計算:
∬(A・n)dS
=∫[0,1]dz∫[0,1]dr∫[0,2π]rdφ(rcosφ+2z)
ここで、∫[0,2π]dφ cosφ=0なので
=∫[0,1]dz∫[0,1]dr∫[0,2π]dφ 2rz
=∫[0,1]dz∫[0,1]dr 4πrz
=∫[0,1]dz 2πz
19132人目の素数さん:2012/04/17(火) 20:35:58.29
>>18
スッキリいきました。
ありがとうございます。
20132人目の素数さん:2012/04/17(火) 22:39:09.89
<=( ´∀`)  
 (    )  朝鮮人は宇宙一ニダ
 | | |   
 〈_フ__フ


  Λ_Λ    
 < ;`Д´>  あ…
 (    )ポロ 
 | | |  ヽヽ
 (__フ_フ =( ´∀`)
朝鮮人だらけの東京のテレビ局が日夜流す、デマや歪曲に騙されないようにしましょう。

21132人目の素数さん:2012/04/17(火) 22:43:57.12
ln(1-e^(-x))→0 (x→∞)
となる理由を教えて下さい
22132人目の素数さん:2012/04/17(火) 22:56:38.55
それがe^xの定理だったらどーすんだ
理由なんて無いぞwww
23132人目の素数さん:2012/04/17(火) 23:03:07.37
lim[x→∞](1-e^(-x))=1よりlim[x→∞](ln(1-e^(-x)))=lim[t→1](ln(t))

>>22
a>1ならlim[x→∞](1-a^(-x))=0だが?
24132人目の素数さん:2012/04/17(火) 23:59:52.16
|a/b|=|a|/|b|
を証明せよという問題。

b=0のときも含めて証明しなきゃいけないんですか?
25132人目の素数さん:2012/04/18(水) 00:11:09.57
ハミルトンは俺と似てるわ
キャサリン
26132人目の素数さん:2012/04/18(水) 00:35:22.47
対数積分関数li(x)=∫dt/lnt(0からxまでの定積分)について質問です。

ある本に次の記述があります。
引数xが、複素平面上の原点を中心とするある円上を進むとき、
li(x)は複素平面上で螺旋を描く。

ここで疑問なのは、引数xは円上なので毎周同じ値を繰り返すのに、何故、
li(x)は螺旋(毎周違う値)になるのでしょう?同じ引数値で関数値が変
わることに納得が行きません。

その本には、定義域が複素数の場合の、li(x)の計算方法が書かれていな
いので、どうしてもわかりません。よろしくお願いします。
27132人目の素数さん:2012/04/18(水) 00:58:36.58
馬鹿らしいわ
なぜ女は俺に振り向かない?
28132人目の素数さん:2012/04/18(水) 01:13:25.62
>>26
ln t が多価関数だから。
(x=a*e^(iu) (uが0〜2π) とすると ln x=ln a + iu となり2πiずれる)

li(x)=Ei(ln x) と書けるので (Ei(x)=∫[-∞,x]e^t/t dt)
原点をn周するとli(x)=Ei(ln x+2πin)ずれる。
29132人目の素数さん:2012/04/18(水) 02:11:50.20
>>24
b=0のとき各辺がそれぞれ定義されていれば
30132人目の素数さん:2012/04/18(水) 02:32:55.05
>>29
どういうことですか?

|a/b|=|a|/|b|
を証明せよという問題。

b≠0がなくても問題はないんでしょうか?
31132人目の素数さん:2012/04/18(水) 03:34:17.79
>>23
すみません、大変なとこ忘れてました…

x^(2n-1)ln(1-e^(-x))→0 (x→∞) でした

理由をお願いします
32132人目の素数さん:2012/04/18(水) 04:04:53.03
>>16
3次元直交行列が四元数で表せる。
要素が少ないので便利。
33132人目の素数さん:2012/04/18(水) 04:07:15.07
>>30
定義域も含めて一致する必要がある。
34132人目の素数さん:2012/04/18(水) 04:08:18.83
>>31
前スレと同様に、ロピタルの定理を繰り返し使って
x^(2n-1)log(1-e^(-x))→(log(1-e^(-x)))'/(1/x^(2n-1))'
=(e^(-x)/(1-e^(-x)))/(-(2n-1)/x^(2n))=-(2n-1)(x^(2n))/(e^x-1)
→-(2n-1)(x^(2n))'/(e^x-1)'=-(2n-1)(2n)x^(2n-1)/(e^x)
→…→-(2n-1)(2n)!/(e^x)
→0 (x→∞)
35132人目の素数さん:2012/04/18(水) 11:26:14.94
>>34
その表記は正しいのですか…?

ロピタルの定理を使わない方法はないでしょうか?
36132人目の素数さん:2012/04/18(水) 12:13:47.43
>>35

0<t<1/2のときy=ln(1-t)とy=-(2ln2)tの図の位置関係より
-(2ln2)t < ln(1-t) < 0 なので、t=e^(-x)と置くと
-(2ln2)e^(-x) < ln(1-e^(-x)) < 0

したがって、x>ln2のとき
-(2ln2)x^(2n-1)/e^x < x^(2n-1)ln(1-e^(-x)) < 0

ここで e^x = 納k=0,∞]x^k/k! > x^(2n)/(2n)! なので
(2ln2)x^(2n-1)/e^x < (2ln2)(2n)!/x →0 (x→∞)
37132人目の素数さん:2012/04/18(水) 17:55:18.18
そもそもリーマンζは物理会とはつながらないと
考える リサのいう五時減世界には
平和がある
38132人目の素数さん:2012/04/18(水) 19:35:23.67
0≦x≦1,0≦y≦1を満たすとき
z=3y^2-2y-4xy+3x+1
の最大値Mと最小値mを求めよ
39132人目の素数さん:2012/04/18(水) 19:54:38.95
>>38
z( x , y ) は x について高々1次関数なので両端で最大or最小
つまり求める最大値,最小値は z( 0 , y ) or z( 1 , y ) の中にある
x に 0 or 1 を代入したものは単なる y の2次関数だからあとは問題なかろう
40132人目の素数さん:2012/04/18(水) 20:01:12.24
次の各式を因数分解して下さい

(a+b)x+a+b

x^2-3ax+2a^2

6x^2+5xy+y^2

x^3-1

x^2-y^2-z^2-2yz

54x^3+16y^3

問題数多くてごめんなさい
Fラン大学でこの問題出されたから学校の図書館やググったりしたんだけどマジで解んないんだ…
周りの人は皆バカだから問題にすら手をつけてないんだ

最後の頼みはここだけなんだ本当にお願いします
41132人目の素数さん:2012/04/18(水) 20:05:06.96
>>40
さすがに一問目が図書館やググってもわからないとあっては
唖然とするレベル
42132人目の素数さん:2012/04/18(水) 20:21:09.49
>>40
中学校数学からやり直し
43132人目の素数さん:2012/04/18(水) 20:21:59.55
  Λ_Λ  
 <=( ´∀`)  
 (    )  朝鮮人は宇宙一ニダ
 | | |   
 〈_フ__フ


  Λ_Λ    
 < ;`Д´>  あ…
 (    )ポロ 
 | | |  ヽヽ
 (__フ_フ =( ´∀`)
朝鮮人だらけの東京のテレビ局が日夜流す、デマや歪曲に騙されないようにしましょう。
44132人目の素数さん:2012/04/18(水) 20:29:11.28
>>40
大学辞めて中学に入学しろ
45132人目の素数さん:2012/04/18(水) 20:34:33.34
>>38
z=3(y-(2x+1)/3)^2-(2x+1)^2/3+3x+1
46132人目の素数さん:2012/04/18(水) 20:45:56.30
>>38
z=(-4y+3)x+3y^2-2y+1
47132人目の素数さん:2012/04/18(水) 21:14:58.81
>>45
>>46
???
48132人目の素数さん:2012/04/18(水) 21:27:21.95
数学の本で区間?を
[0,∞[ のように書いてあることがあるんだけど
これは[0,∞]とは違うの?
49132人目の素数さん:2012/04/18(水) 21:29:29.68
>>45
>>46
???
50132人目の素数さん:2012/04/18(水) 21:29:33.21
はたして無限の彼方は]での終わりがあるのだろうか
いや、無い
51132人目の素数さん:2012/04/18(水) 21:36:14.41
(k+9)(k+1)>0

のkの値がk<-9、k<-1
となって不等号の向きが変わるのはなぜですか?
不等号の向きが変わるのは掛けたとき割ったときだけと聞いたんですがこれは移行しただけですよね
52132人目の素数さん:2012/04/18(水) 21:38:02.56
> (k+9)(k+1)>0
>
> のkの値がk<-9、k<-1
間違ってる
53132人目の素数さん:2012/04/18(水) 21:38:39.52
>>50
[0,∞)とはどこが違うんだろ?
あほな質問でごめん
54132人目の素数さん:2012/04/18(水) 21:38:47.99
>>41
地元と学校の周りに図書館なくて唯一大学にある図書館に因数分解の本がないんです…
1、2問でいいから教えてよー

>>45
出来たらしたいです…
せめてヒントだけども下さい!!
55132人目の素数さん:2012/04/18(水) 21:39:26.17
(k+9)(k+1) を y=(x+9)(x+1) に書き換えて
展開して
グラフ描いて画像うpしろ
56132人目の素数さん:2012/04/18(水) 21:41:05.40
>>54
大学生なら計画法の本あんだろ
>>38はその一番簡単なヤツだから
大学の図書館行って 数理計画法 とかの本借りて来い
それかORの本でも見ろ
57132人目の素数さん:2012/04/18(水) 21:43:05.15
>>45 は y の2次関数と見て,>>46 は x の1次関数と見て整理している
軸の位置で場合分けもいるので >>45 のほうが少し面倒
>>46>>39 のように続ける

参考書を持ってないなら1冊は買っておけ
それを見たほうが早い
5848:2012/04/18(水) 21:46:12.55
今ググったら開区間(a,b)を
]a,b[と書く流儀もあるということを知って解決しました。
59132人目の素数さん:2012/04/18(水) 21:47:01.01
>>56
計画法って本ですね
本当にありがとうございます!!
明日朝早くにいって探してきます
これでバカ組から脱出できるようマジで頑張ります
60132人目の素数さん:2012/04/18(水) 21:47:23.03
>>52
異なる二つの実数解をもつように定数kの値の範囲を求めよ
D=k^2+10k+9


という問いです
61132人目の素数さん:2012/04/18(水) 21:52:16.33
>>60
> (k+9)(k+1)>0
> のkの値がk<-9、k<-1

これは、例えば k=100 は (k+9)(k+1)>0 を満たさないと言ってるんだが?
62132人目の素数さん:2012/04/18(水) 21:55:12.86
満たすってなに
63132人目の素数さん:2012/04/18(水) 21:57:47.52
>>60
> (k+9)(k+1)>0
>
> のkの値がk<-9、k<-1

(k+9)(k+1)>0の解は k<-9,-1<k だと言っている
64132人目の素数さん:2012/04/18(水) 22:00:49.95
>>51
>> kの値がk<-9、k<-1

  kの値がk<-9、k>-1
の入力ミスだろう
(k+9)(k+1)>0 のときにこうなるのは
y = (k+9)(k+1) のグラフをイメージすればわかる
なお,数直線をイメージして,小さいほうから順に
  k < -9 ,-1 < k
と書くほうがいいと思う こういう書き方を嫌う先生もいるが
65132人目の素数さん:2012/04/18(水) 22:01:42.49
ゆとり
66132人目の素数さん:2012/04/18(水) 22:09:11.15
>>57
サンクス
受験生なんだが、オススメの参考書教えて下さい
67132人目の素数さん:2012/04/18(水) 22:10:08.68
>>66
チャート式でもやっとけ
68132人目の素数さん:2012/04/18(水) 22:25:22.47
>>66
単元割の本でもいいが
  『くらべてつなげてまとめる数学』(文英堂)
を薦めておこう
まだ改良の余地はあるが,こういうコンセプトの本はあまりないので
レベルはセンター+αくらい
69132人目の素数さん:2012/04/18(水) 23:54:56.88
-1, 0, +1の3種類のどれかの値をとるn個の数の列
{Xi} = {X1, X2, ... , Xn}
と、Xiの値を予想した数の列
{Yi} = {Y1, Y2, ... , Yn}
があって、
{X'i} = {Xi : |Xi - Yi| ≧ 2}
とするとき、
X'iの値を予想した数の列Y'iがYiに対してどれだけ改善したか
(Σ|X'i - Y'i|が小さくなったか)を評価するには
どのような指標を用いればいいですか?
70132人目の素数さん:2012/04/19(木) 00:00:46.66
>>69
問題がよくわからないが、Σ|X'i - Y'i|がどれだけ小さくなったかを評価するなら
Σ|X'i - Y'i|そのものを指標にしたらいかんの?
71132人目の素数さん:2012/04/19(木) 00:01:44.03
>>67
>>68
ありがとうございます。
明日本屋行ってきますw
72132人目の素数さん:2012/04/19(木) 00:18:16.92
ab-cd=1
1,2
2(n+1)-1(2n+1)=1
4-3=1
6-5=1
8-7=1
3(2n+1)-2(3n+1)=1
3-2=1
9-8=1
15-14=1
21-20=1

隣り合う数が合成数なのはこのデイオファンタスの解だから
その間の合成数の前後に素数がある。

これって正解ですか?
73132人目の素数さん:2012/04/19(木) 00:23:13.86
5ー4=1は5が合成数じゃないから3(2n+1)-2(3n+1)=1 の解に出てこない。
3と8のあいだの4、5、6、7に素数がある。
19も素数
74132人目の素数さん:2012/04/19(木) 00:24:25.77
隣あう数が両方共合成数なら差は1
75132人目の素数さん:2012/04/19(木) 00:25:28.57
あたらしい素数の判定方式を見つけた。。。かも
76132人目の素数さん:2012/04/19(木) 00:32:23.66
そうかすごいな世紀の大発見だな
77132人目の素数さん:2012/04/19(木) 00:51:13.98
そりゃあ……
隣り合ってる合成数の連続をぶった切ってる区切りが素数なんだから
そうなんじゃね?
78132人目の素数さん:2012/04/19(木) 01:04:20.99
>>28
e^(iθ)が周期2πで同じ値を繰り返すので、その逆関数である対数関数が多価関数
になるという理解でよいでしょうか?
79132人目の素数さん:2012/04/19(木) 01:07:00.21
n=2^200000000000000000000とかいれたら一発で世界最大の素数がみつかるかも。
80132人目の素数さん:2012/04/19(木) 01:08:50.35
螺旋ちゅーことで連続してるから納得いかないんだろう、
バームクーヒェンぽい不連続な同心円がならんでりゃー
きっと納得いくかもネ☆
81132人目の素数さん:2012/04/19(木) 01:12:13.09
せやな
可能性は高いな
その数はテキストファイルで1TBくらいになるかもな
それをどうやって判定するんだ
82132人目の素数さん:2012/04/19(木) 01:29:44.63
巨大素数はほとんど合成数の間にポツンとあるからその前後が判定しきから抜けてればおk。
83132人目の素数さん:2012/04/19(木) 01:38:40.25
とはいうものの 2の0.4億乗くらいしか
今のコンピュータじゃ計算できんのだぜ
それをどうやるか てのが
クソでfuckな問題で
ポツンとあるのはわかってんのに
具体的なモンはひょっこりとは出てこない、
マシンの限界を突破させるしかない
84132人目の素数さん:2012/04/19(木) 02:12:14.60
数を数えてるとき素数が不規則に現れるのはどうしてですか
85132人目の素数さん:2012/04/19(木) 02:27:56.65
リーマン展開してから
ζの1より大きい箇所を無視して
積分してγ表紙をあたえるとそれが
ベルヌーイ数と連結する
ベルヌーイ数の規則性が素数とまじわる
ベルヌーイは単調増加
86132人目の素数さん:2012/04/19(木) 02:43:47.01
ゼータ関数のゼロ点の間隔=原子核エネルギーの間隔なんだってね
87132人目の素数さん:2012/04/19(木) 02:51:50.19
嘘みたいな話だよな
88132人目の素数さん:2012/04/19(木) 03:24:39.60
d^2y/dx^2-2(dy/dx)^2-4dy/dx=0
という微分方程式ですけど、dy/dxをfという関数に置き換えて
df/dx-f^2-4f=0
という微分方程式にした後の解法がわかりません。
ちなみに解答をみるとyの関数としてf(y)=C(e^2y)-2となっています。
89132人目の素数さん:2012/04/19(木) 04:17:58.86
c≧0において |a|≦c⇔-c≦a≦c

この証明が何時間かけてもわかりません。
c=a+h(h≧0)とおけばいいと思うのですが、、、、、、どなたかお願いします。
90132人目の素数さん:2012/04/19(木) 04:25:44.85
>>89
数直線上で点 a と点 0 との距離が c 以下
っていう直観的な理解じゃ駄目なの?
91132人目の素数さん:2012/04/19(木) 04:29:03.89
>>89
aの正負で場合分けでいいだろ
92132人目の素数さん:2012/04/19(木) 04:33:33.95
厳密な証明なので直感ではダメなんですよ。

どこの高校数学の参考書にも載ってない、、、、、

>>91
その場合分けで頭がこんがらがってしまうんです。
93132人目の素数さん:2012/04/19(木) 04:44:38.48
絶対値の定義じゃないか?
定義っぽいので何も疑問も持たず普通に使ってたが
94132人目の素数さん:2012/04/19(木) 04:52:46.60
>>91
|a|の定義
a>0ならば|a|=a
a<0ならば|a|=-a
で場合分け
95132人目の素数さん:2012/04/19(木) 08:17:38.16
>>92
> その場合分けで頭がこんがらがってしまうんです。
どうこんがらがるのか具体的に書いてみてくれ。
96132人目の素数さん:2012/04/19(木) 10:15:07.41
pass
97132人目の素数さん:2012/04/19(木) 10:21:25.77
>>88
df/dx=2f^2+4f
df/(f(f+2))=2dx
∫df(1/f-1/(f+2))=∫4dx
log(f/(f+2))=4x+C
f/(f+2)=C*e^(4x) ∴f=2e^(4x)/(C-e^(4x))
y=∫fdx=-log(C-e^(4x))/2 ∴C-e^(4x)=e^(-2y)
よって、f=2Ce^(2y)-2
9892:2012/04/19(木) 13:19:20.32
c≧0において |a|≦c⇔-c≦a≦c

c=a+hとおくと
|a|≦a+h⇔-a-h≦a≦a+h

ここで、a<0のとき|a|≦a+h ← -a-h≦a≦a+h h>0
がわかりません。

99132人目の素数さん:2012/04/19(木) 13:24:29.08
-a-h≦-aなんでしょうねきっと
100132人目の素数さん:2012/04/19(木) 13:25:04.22
-a+h≦-aか。
101132人目の素数さん:2012/04/19(木) 13:27:21.64
c=a+hとおくと
|a|≦a+h⇔-a-h≦a≦a+h

a≧0のときは|a|=a よりa≦a+h
a<0のときは|a|=-aより-a-h≦-aに-1をかけa+h≧a

でいいのか!
102132人目の素数さん:2012/04/19(木) 13:29:17.18
>>78
うん。

リーマン面(Riemann surface)で検索すると理解できるかも
103132人目の素数さん:2012/04/19(木) 13:31:25.68
>c=a+hとおくと

↑なんでこんなバカなことやってんだ
104132人目の素数さん:2012/04/19(木) 13:32:32.63
>>103
c≧0において |a|≦c⇔-c≦a≦c
を示す
cはaより少しおおきいかずということをアピールしたいという教育的観点から
c=a+hとおきました

もっといい証明、ありますか?
105132人目の素数さん:2012/04/19(木) 13:34:10.19
>>101
だめ
106132人目の素数さん:2012/04/19(木) 13:36:43.41
>>98,104
> |a|≦a+h⇔-a-h≦a≦a+h

この条件は
c=a+h≧0 つまり h≧-a
であってh>0ではない
107132人目の素数さん:2012/04/19(木) 13:44:32.64
あーなるほど。

しかしcをaでおきかえての証明、どう証明すればいいんだ、、、、、、わけがわからなくなってきた。。。。。。
108132人目の素数さん:2012/04/19(木) 14:08:55.99
ベクトルの外積って3次元でだけで定義されてるんですか?
そうだとしたら、他の次元にも外積みたいなのに相当するものはありますか?
109132人目の素数さん:2012/04/19(木) 14:10:22.62
>>104
> cはaより少しおおきいかずということをアピールしたいという教育的観点から
「cはaより少しおおきいかず」がどういう意図かが分からないが、
aが負の場合は|a|≦cを満たすcは-a以上の数(例えばa=-100ならcは100以上)
になるから「少しおおきいかず」とは言えないんじゃないか?
110132人目の素数さん:2012/04/19(木) 14:11:28.46
置き換えるほうがわかりにくそう
{ a | |a| ≦ c } = { a | 0 ≦ a ≦ c ∨ 0 ≦ −a ≦ c }
             = { a | 0 ≦ a ≦ c ∨ −c ≦ a ≦ 0 }
             = { a | −c ≦ a ≦ c }
111104:2012/04/19(木) 14:21:20.44
cはaより0から遠いよ=a+hかなと思ったわけです。

しかし、>>110さんのいうとおり、場合分けが複雑になってしまいますね。

うーん、あきらめますかね。
ここはスタンダードな証明で行きます。
112132人目の素数さん:2012/04/19(木) 14:23:27.10
初恋の人と付き合うには
どうしたらいい?
113132人目の素数さん:2012/04/19(木) 15:13:34.17
写真を抱き枕のカバーにプリントしてください。
114132人目の素数さん:2012/04/19(木) 15:25:04.28
お兄ちゃん!?
私の写真を、変な枕に勝手にプリントしないでよねっ
115132人目の素数さん:2012/04/19(木) 17:30:58.33
0の0乗っていくつになるんでしょうか。y=0^x(^はべき乗)がどんな形状になるか知りたいんです。
116132人目の素数さん:2012/04/19(木) 17:32:10.93
どんな数でも0乗は1
117132人目の素数さん:2012/04/19(木) 17:34:01.41
「0の0」乗を「空集合から空集合への写像の個数」と定義するなら、0の0乗=1
しかし、そう定義する必然性はない
118132人目の素数さん:2012/04/19(木) 18:03:02.86
2の1/2乗は√2
しかし、そう定義する必然性はない
119132人目の素数さん:2012/04/19(木) 18:07:06.29
120132人目の素数さん:2012/04/19(木) 18:44:39.21
>>116
どんな数に0は入ってねーよ。
lim_[x→+0](x^x)=1だけど、0^0とは違う。
121132人目の素数さん:2012/04/19(木) 18:46:13.12
log(0)って解析的な値はなんですか?
122132人目の素数さん:2012/04/19(木) 18:50:00.25
0^0=1。
123132人目の素数さん:2012/04/19(木) 18:54:37.27
>>118
1/2乗には必然性あるよ
124132人目の素数さん:2012/04/19(木) 19:13:43.13
>>121
無い
125132人目の素数さん:2012/04/19(木) 19:34:45.42
高校レベルの確率の問題なんだけど・・・

球面上に3点a,b,cをそれぞれ独立に一様分布でランダムに落とすことを考える。
球の中心をoとしたとき角aob<90度ならばa,bは接触していると考えることにする。
またa,bが接触していて、かつb,cも接触しているならばa,cも接触していると考える。
このときa,b,cがすべて接触するような確率を求めよ。

答えは(π+2)/4πらしいんだけど解説お願いします。
126132人目の素数さん:2012/04/19(木) 19:38:17.43
すみません問題の最後は

a,b,cのどの2点も接触している確率

とすべきでした。
127132人目の素数さん:2012/04/19(木) 19:44:11.65
log(0)って解析的な値は「無い」そうですが、
それでは、limit [ log(|x|), x->0, x:Real ]の時のlog(|x|)の値はなんですか?
128132人目の素数さん:2012/04/19(木) 19:58:50.93
>>125
> またa,bが接触していて、かつb,cも接触しているならばa,cも接触していると考える。
∠aob<90°、∠boc<90°、∠aoc≧90° のときaとcは接触していると考えるのか否か
129132人目の素数さん:2012/04/19(木) 20:17:07.25
小学校3年の学力テスト

問題
子供が3人います。1人にあめを5こずつ配ります。
あめは何個いりますか?
あめの数を求める式を書きましょう。


答え
3×5=15 …… 不正解
5×3=15 …… 正解


質問
何故、3×5=15 が不正解になるんでしょうか?

子供に理解させる為に、理由を教えて下さい。
130132人目の素数さん:2012/04/19(木) 20:20:18.63
>>129
5皿ある。3こずつ林檎がのっている。で5×3は駄目!?
http://uni.2ch.net/test/read.cgi/math/1292334048/
こっちいけ
131132人目の素数さん:2012/04/19(木) 20:22:23.67
3×5=15を不正解として教えると将来数学ダメな子になると思う
132132人目の素数さん:2012/04/19(木) 20:23:21.23
>>129
教師の都合
中学で文字式を習ったら
「3人の子どもに a 個ずつあめを配るとき全部で何個いりますか」
という問題なら 3a個 を答えとするだろう
133132人目の素数さん:2012/04/19(木) 20:23:22.54
>>128
返信ありがとうございます。
その時も接触していると考えます。
134132人目の素数さん:2012/04/19(木) 20:23:35.92
>>130
専用スレあるんですね。
失礼しました。

誘導ありがとうございます。
135132人目の素数さん:2012/04/19(木) 20:26:32.70
>>129
不正解にする人は頭がおかしいだけだから気にするな
136132人目の素数さん:2012/04/19(木) 20:27:52.52
スカラーで交換法則が成り立たないと教えるのか……日本の数学教育はもう終わりだな
137132人目の素数さん:2012/04/19(木) 20:52:38.05
138132人目の素数さん:2012/04/19(木) 20:57:18.42
>>125
ざっくりと叩き台投げてみる

{AB接触 - (AB接触 and AC非接触)} or {AB非接触 and BC接触 and CA接触}
で解いた

【AB接触 and AC非接触】
球を地球に見立ててAを北極におくと、Bは北半球に存在する
∠BOA=θとおくと0≦θ<π/2
各θは緯線の長さ2πsinθ/(4π)に応じた確率をとる
(上の一行は正確な数学表現ではない)

南半球かつB側の半球(以下B半球)にないエリアというのは、
2本の経線で挟まれた舟のような形をしている
球面に対する表面積比は2(π-θ)/(4π)=(π-θ)/(2π)

求める確率は
∫_[ 0 ≦θ<π/2 ]2πsinθ/(4π)*(π-θ)/(2π) dθ = (π-1)/(4π)

【AB接触 - (AB接触 and AC非接触)】
AB接触確率が1/2なので1/2 - (π-1)/(4π) = (π+1)/(4π)

【AB非接触 and BC接触 and CA接触】
AB接触 and AC非接触と同様に考えて、同じく笹舟のような形なので
∫_[ π/2 ≦θ<π ](2πsinθ/4π)*(π-θ)/(2π) dθ = 1/(4π)

【答え】
(π+1)/(4π) + 1/(4π)= (π+2)/(4π)
139132人目の素数さん:2012/04/19(木) 20:58:25.15
>>125
単位球面を考えてaが(1,0,0)とするとbがx>0の位置に存在する場合に接触するから
その確率は1/2
aとbが接触し、bとcが接触する確率であれば(1/2)^2=1/4
点aに対し球面のx>0の部分をaを頂点とする半球とすると
求める確率は、点bとcがこの半球上に存在しかつ点cは点bを頂点とする半球上に
存在することが必要
140132人目の素数さん:2012/04/19(木) 20:59:01.86
訂正
誤:{AB接触 - (AB接触 and AC非接触)}
正:{AB接触 - (BC非接触 and AC非接触)}
>>138における全ての"{AB接触 - (AB接触 and AC非接触)}"に対して)
141138 ◆U8UAvXc9Z4aQ :2012/04/19(木) 21:07:52.53
悪い、グダグダになってた

{AB接触 - (AB接触 and BC非接触 and CA非接触)} or {AB非接触 and BC接触 and CA接触}
で解いた

【AB接触 and BC非接触 and CA非接触】
球を地球に見立ててAを北極におくと、Bは北半球に存在する
∠BOA=θとおくと0≦θ<π/2
各θは緯線の長さ2πsinθ/(4π)に応じた確率をとる
(上の一行は正確な数学表現ではない)

南半球かつB側の半球にないエリアというのは、
2本の経線で挟まれた舟のような形をしている
球面に対する表面積比は2(π-θ)/(4π)=(π-θ)/(2π)

求める確率は
∫_[ 0 ≦θ<π/2 ]2πsinθ/(4π)*(π-θ)/(2π) dθ = (π-1)/(4π)

【AB接触】
1/2

【AB非接触 and BC接触 and CA接触】
BC非接触 and CA非接触と同様に考えて、同じく笹舟のような形なので
∫_[ π/2 ≦θ<π ](2πsinθ/4π)*(π-θ)/(2π) dθ = 1/(4π)

【答え】
1/2 - (π-1)/(4π) + 1/(4π)= (π+2)/(4π)

休んでくる
142132人目の素数さん:2012/04/19(木) 21:14:56.92
連続関数fがR上可積分なら、fはR上で有界だといえますか?
143132人目の素数さん:2012/04/19(木) 21:44:26.38
>>128
∠aob>90°、∠boc<90°、∠aoc<90° のときaとbは接触?
∠aob<90°、∠boc>90°、∠aoc<90° のときbとcは接触?
144132人目の素数さん:2012/04/19(木) 21:58:43.23
145132人目の素数さん:2012/04/19(木) 22:13:35.67
a,bの座標をa(1,0,0,)
bの座標を(sinθ, 0, cosθ)とすると
とした場合に題意を満たす点cが存在する確率をP(θ)、求める確率をQとすると
P(θ)=(1-θ/π)/2
Q=2*∫[0,π/2]P(θ)dθ/2π=3/16
???
146132人目の素数さん:2012/04/19(木) 22:16:24.32
×Q=2*∫[0,π/2]P(θ)dθ/2π=3/16
?Q=2*∫[0,π/2]P(θ)dθ/π=3/16
147132人目の素数さん:2012/04/19(木) 22:27:55.22
P(θ)=(1-θ/π)/2 (0<=θ<=π/2)
P(θ)=0 (π/2<=θ<=π)
Q=∫[0,π]P(θ)dθ/π=3/16
148132人目の素数さん:2012/04/19(木) 22:54:33.87
>>142
∫[0,∞] xdx/(1+x^6sin^2x) は収束する。(解析概論より)
149132人目の素数さん:2012/04/19(木) 23:13:58.30
150132人目の素数さん:2012/04/19(木) 23:25:30.32
レスサンクス

なっちゃんの写真まくらにはるわ
151132人目の素数さん:2012/04/19(木) 23:45:41.68
>>141
詳しい回答サンクス。理解できました。

そもそもθの分布がsinθに比例するという基本的なところを見落としていたw
このようにABが接触するかどうかで場合分けするのがいいね。

AB接触の場合は補集合を考えなくても
∫_[ 0 ≦θ<π/2 ]2πsinθ/(4π)*(π+θ)/(2π) dθ = (π+1)/(4π)
といきなり出してしまったほうがいいね
152132人目の素数さん:2012/04/20(金) 00:10:02.22
>>97
ありがとうございました
153132人目の素数さん:2012/04/20(金) 00:12:48.69
>>92ですが、気になって眠れません。
a+hとおいたとき、どう証明すればいいでしょうか?
場合分けが頭こんがらがる

良問だ
154132人目の素数さん:2012/04/20(金) 00:15:25.26
aの正負で場合分けするだけ。数直線をイメージするとなおわかりやすい。
a+hとおくってのはよくわからん。その方法が適切という直観でもあるの?
155132人目の素数さん:2012/04/20(金) 00:19:21.04
>>89でしたね失礼
a<0ときc≧0において |a|≦c⇔-c≦a≦c

どうおけばいいんだろう?

c=a+hで
a<0ときc≧0において |a|≦a+h⇔-a-h≦a≦a+h (h>0)

でいい?
156132人目の素数さん:2012/04/20(金) 00:22:59.83
c=a+hで
a<0ときc≧0において |a|≦a-h⇔a+h≦a≦a-h (h>0)
か?

わけわからん

157132人目の素数さん:2012/04/20(金) 00:23:05.49
>>153
(i) a≧0ならば0≦a≦c
(ii) a<0ならば -c≦a<0
よって-c≦a≦c
158132人目の素数さん:2012/04/20(金) 00:25:07.89
訂正
c=a+hで
a<0ときc≧0において |a|≦a-h⇔a+h≦a≦-(a-h) (h>0)
か?

ふくざつすぎる、、、、
159132人目の素数さん:2012/04/20(金) 00:27:37.97
a<0のときはc=a-hとおかなきゃいけない、ここがトリックだったか。
これいい問題だ。

原稿一本かけるな。
160132人目の素数さん:2012/04/20(金) 00:39:15.81
有限体F4の乗算は何故下記になるのです?わけわかんないんですけど。
2×2=3
2×3=1
3×3=2
161132人目の素数さん:2012/04/20(金) 01:07:07.35
2とか3とか何のことだよ
162132人目の素数さん:2012/04/20(金) 01:16:26.50
          __ノ)-'´ ̄ ̄`ー- 、_
        , '´  _. -‐'''"二ニニ=-`ヽ、
      /   /:::::; -‐''"        `ーノ
     /   /:::::/           \
     /    /::::::/          | | |  |
     |   |:::::/ /     |  | | | |  |
      |   |::/ / / |  | ||  | | ,ハ .| ,ハ|
      |   |/ / / /| ,ハノ| /|ノレ,ニ|ル' 
     |   |  | / / レ',二、レ′ ,ィイ|゙/   私は只の数ヲタなんかとは付き合わないわ。
.     |   \ ∠イ  ,イイ|    ,`-' |      頭が良くて数学が出来てかっこいい人。それが必要条件よ。
     |     l^,人|  ` `-'     ゝ  |        さらに Ann.of Math に論文書けば十分条件にもなるわよ。
      |      ` -'\       ー'  人          一番嫌いなのは論文数を増やすためにくだらない論文を書いて
    |        /(l     __/  ヽ、           良い論文の出版を遅らせるお馬鹿な人。
     |       (:::::`‐-、__  |::::`、     ヒニニヽ、         あなたの論文が Ann of Math に accept される確率は?
    |      / `‐-、::::::::::`‐-、::::\   /,ニニ、\            それとも最近は Inv. Math. の方が上かしら?
   |      |::::::::::::::::::|` -、:::::::,ヘ ̄|'、  ヒニ二、 \
.   |      /::::::::::::::::::|::::::::\/:::O`、::\   | '、   \
   |      /:::::::::::::::::::/:::::::::::::::::::::::::::::'、::::\ノ  ヽ、  |
  |      |:::::/:::::::::/:::::::::::::::::::::::::::::::::::'、',::::'、  /:\__/‐、
  |      |/:::::::::::/::::::::::::::::::::::::::::::::::O::| '、::| く::::::::::::: ̄|
   |     /_..-'´ ̄`ー-、:::::::::::::::::::::::::::::::::::|/:/`‐'::\;;;;;;;_|
   |    |/::::::::::::::::::::::\:::::::::::::::::::::::::::::|::/::::|::::/:::::::::::/
    |   /:::::::::::::::::::::::::::::::::|:::::::::::::::::::::O::|::|::::::|:::::::::::::::/
163132人目の素数さん:2012/04/20(金) 01:34:16.18
あんた誰?(笑)
164132人目の素数さん:2012/04/20(金) 02:26:24.17
おまんこは臭いの?
165132人目の素数さん:2012/04/20(金) 02:28:08.23
          __ノ)-'´ ̄ ̄`ー- 、_
        , '´  _. -‐'''"二ニニ=-`ヽ、
      /   /:::::; -‐''"        `ーノ
     /   /:::::/           \
     /    /::::::/          | | |  |
     |   |:::::/ /     |  | | | |  |
      |   |::/ / / |  | ||  | | ,ハ .| ,ハ|
      |   |/ / / /| ,ハノ| /|ノレ,ニ|ル' 
     |   |  | / / レ',二、レ′ ,ィイ|゙/   私は只の数ヲタなんかとは付き合わないわ。
.     |   \ ∠イ  ,イイ|    ,`-' |      頭が良くて数学が出来てかっこいい人。それが必要条件よ。
     |     l^,人|  ` `-'     ゝ  |        さらに Ann.of Math に論文書けば十分条件にもなるわよ。
      |      ` -'\       ー'  人          一番嫌いなのは論文数を増やすためにくだらない論文を書いて
    |        /(l     __/  ヽ、           良い論文の出版を遅らせるお馬鹿な人。
     |       (:::::`‐-、__  |::::`、     ヒニニヽ、         あなたの論文が Ann of Math に accept される確率は?
    |      / `‐-、::::::::::`‐-、::::\   /,ニニ、\            それとも最近は Inv. Math. の方が上かしら?
   |      |::::::::::::::::::|` -、:::::::,ヘ ̄|'、  ヒニ二、 \
.   |      /::::::::::::::::::|::::::::\/:::O`、::\   | '、   \
   |      /:::::::::::::::::::/:::::::::::::::::::::::::::::'、::::\ノ  ヽ、  |
  |      |:::::/:::::::::/:::::::::::::::::::::::::::::::::::'、',::::'、  /:\__/‐、
  |      |/:::::::::::/::::::::::::::::::::::::::::::::::O::| '、::| く::::::::::::: ̄|
   |     /_..-'´ ̄`ー-、:::::::::::::::::::::::::::::::::::|/:/`‐'::\;;;;;;;_|
   |    |/::::::::::::::::::::::\:::::::::::::::::::::::::::::|::/::::|::::/:::::::::::/
    |   /:::::::::::::::::::::::::::::::::|:::::::::::::::::::::O::|::|::::::|:::::::::::::::/
166132人目の素数さん:2012/04/20(金) 02:28:31.17
          __ノ)-'´ ̄ ̄`ー- 、_
        , '´  _. -‐'''"二ニニ=-`ヽ、
      /   /:::::; -‐''"        `ーノ
     /   /:::::/           \
     /    /::::::/          | | |  |
     |   |:::::/ /     |  | | | |  |
      |   |::/ / / |  | ||  | | ,ハ .| ,ハ|
      |   |/ / / /| ,ハノ| /|ノレ,ニ|ル' 
     |   |  | / / レ',二、レ′ ,ィイ|゙/   私は只の数ヲタなんかとは付き合わないわ。
.     |   \ ∠イ  ,イイ|    ,`-' |      頭が良くて数学が出来てかっこいい人。それが必要条件よ。
     |     l^,人|  ` `-'     ゝ  |        さらに Ann.of Math に論文書けば十分条件にもなるわよ。
      |      ` -'\       ー'  人          一番嫌いなのは論文数を増やすためにくだらない論文を書いて
    |        /(l     __/  ヽ、           良い論文の出版を遅らせるお馬鹿な人。
     |       (:::::`‐-、__  |::::`、     ヒニニヽ、         あなたの論文が Ann of Math に accept される確率は?
    |      / `‐-、::::::::::`‐-、::::\   /,ニニ、\            それとも最近は Inv. Math. の方が上かしら?
   |      |::::::::::::::::::|` -、:::::::,ヘ ̄|'、  ヒニ二、 \
.   |      /::::::::::::::::::|::::::::\/:::O`、::\   | '、   \
   |      /:::::::::::::::::::/:::::::::::::::::::::::::::::'、::::\ノ  ヽ、  |
  |      |:::::/:::::::::/:::::::::::::::::::::::::::::::::::'、',::::'、  /:\__/‐、
  |      |/:::::::::::/::::::::::::::::::::::::::::::::::O::| '、::| く::::::::::::: ̄|
   |     /_..-'´ ̄`ー-、:::::::::::::::::::::::::::::::::::|/:/`‐'::\;;;;;;;_|
   |    |/::::::::::::::::::::::\:::::::::::::::::::::::::::::|::/::::|::::/:::::::::::/
    |   /:::::::::::::::::::::::::::::::::|:::::::::::::::::::::O::|::|::::::|:::::::::::::::/
167132人目の素数さん:2012/04/20(金) 02:29:45.41
          __ノ)-'´ ̄ ̄`ー- 、_
        , '´  _. -‐'''"二ニニ=-`ヽ、
      /   /:::::; -‐''"        `ーノ
     /   /:::::/           \
     /    /::::::/          | | |  |
     |   |:::::/ /     |  | | | |  |
      |   |::/ / / |  | ||  | | ,ハ .| ,ハ|
      |   |/ / / /| ,ハノ| /|ノレ,ニ|ル' 
     |   |  | / / レ',二、レ′ ,ィイ|゙/   私は只の数ヲタなんかとは付き合わないわ。
.     |   \ ∠イ  ,イイ|    ,`-' |      頭が良くて数学が出来てかっこいい人。それが必要条件よ。
     |     l^,人|  ` `-'     ゝ  |        さらに Ann.of Math に論文書けば十分条件にもなるわよ。
      |      ` -'\       ー'  人          一番嫌いなのは論文数を増やすためにくだらない論文を書いて
    |        /(l     __/  ヽ、           良い論文の出版を遅らせるお馬鹿な人。
     |       (:::::`‐-、__  |::::`、     ヒニニヽ、         あなたの論文が Ann of Math に accept される確率は?
    |      / `‐-、::::::::::`‐-、::::\   /,ニニ、\            それとも最近は Inv. Math. の方が上かしら?
   |      |::::::::::::::::::|` -、:::::::,ヘ ̄|'、  ヒニ二、 \
.   |      /::::::::::::::::::|::::::::\/:::O`、::\   | '、   \
   |      /:::::::::::::::::::/:::::::::::::::::::::::::::::'、::::\ノ  ヽ、  |
  |      |:::::/:::::::::/:::::::::::::::::::::::::::::::::::'、',::::'、  /:\__/‐、
  |      |/:::::::::::/::::::::::::::::::::::::::::::::::O::| '、::| く::::::::::::: ̄|
   |     /_..-'´ ̄`ー-、:::::::::::::::::::::::::::::::::::|/:/`‐'::\;;;;;;;_|
   |    |/::::::::::::::::::::::\:::::::::::::::::::::::::::::|::/::::|::::/:::::::::::/
    |   /:::::::::::::::::::::::::::::::::|:::::::::::::::::::::O::|::|::::::|:::::::::::::::/
168132人目の素数さん:2012/04/20(金) 02:30:40.12
          __ノ)-'´ ̄ ̄`ー- 、_
        , '´  _. -‐'''"二ニニ=-`ヽ、
      /   /:::::; -‐''"        `ーノ
     /   /:::::/           \
     /    /::::::/          | | |  |
     |   |:::::/ /     |  | | | |  |
      |   |::/ / / |  | ||  | | ,ハ .| ,ハ|
      |   |/ / / /| ,ハノ| /|ノレ,ニ|ル' 
     |   |  | / / レ',二、レ′ ,ィイ|゙/   私は只の数ヲタなんかとは付き合わないわ。
.     |   \ ∠イ  ,イイ|    ,`-' |      頭が良くて数学が出来てかっこいい人。それが必要条件よ。
     |     l^,人|  ` `-'     ゝ  |        さらに Ann.of Math に論文書けば十分条件にもなるわよ。
      |      ` -'\       ー'  人          一番嫌いなのは論文数を増やすためにくだらない論文を書いて
    |        /(l     __/  ヽ、           良い論文の出版を遅らせるお馬鹿な人。
     |       (:::::`‐-、__  |::::`、     ヒニニヽ、         あなたの論文が Ann of Math に accept される確率は?
    |      / `‐-、::::::::::`‐-、::::\   /,ニニ、\            それとも最近は Inv. Math. の方が上かしら?
   |      |::::::::::::::::::|` -、:::::::,ヘ ̄|'、  ヒニ二、 \
.   |      /::::::::::::::::::|::::::::\/:::O`、::\   | '、   \
   |      /:::::::::::::::::::/:::::::::::::::::::::::::::::'、::::\ノ  ヽ、  |
  |      |:::::/:::::::::/:::::::::::::::::::::::::::::::::::'、',::::'、  /:\__/‐、
  |      |/:::::::::::/::::::::::::::::::::::::::::::::::O::| '、::| く::::::::::::: ̄|
   |     /_..-'´ ̄`ー-、:::::::::::::::::::::::::::::::::::|/:/`‐'::\;;;;;;;_|
   |    |/::::::::::::::::::::::\:::::::::::::::::::::::::::::|::/::::|::::/:::::::::::/
    |   /:::::::::::::::::::::::::::::::::|:::::::::::::::::::::O::|::|::::::|:::::::::::::::/
169132人目の素数さん:2012/04/20(金) 02:31:11.61
          __ノ)-'´ ̄ ̄`ー- 、_
        , '´  _. -‐'''"二ニニ=-`ヽ、
      /   /:::::; -‐''"        `ーノ
     /   /:::::/           \
     /    /::::::/          | | |  |
     |   |:::::/ /     |  | | | |  |
      |   |::/ / / |  | ||  | | ,ハ .| ,ハ|
      |   |/ / / /| ,ハノ| /|ノレ,ニ|ル' 
     |   |  | / / レ',二、レ′ ,ィイ|゙/   私は只の数ヲタなんかとは付き合わないわ。
.     |   \ ∠イ  ,イイ|    ,`-' |      頭が良くて数学が出来てかっこいい人。それが必要条件よ。
     |     l^,人|  ` `-'     ゝ  |        さらに Ann.of Math に論文書けば十分条件にもなるわよ。
      |      ` -'\       ー'  人          一番嫌いなのは論文数を増やすためにくだらない論文を書いて
    |        /(l     __/  ヽ、           良い論文の出版を遅らせるお馬鹿な人。
     |       (:::::`‐-、__  |::::`、     ヒニニヽ、         あなたの論文が Ann of Math に accept される確率は?
    |      / `‐-、::::::::::`‐-、::::\   /,ニニ、\            それとも最近は Inv. Math. の方が上かしら?
   |      |::::::::::::::::::|` -、:::::::,ヘ ̄|'、  ヒニ二、 \
.   |      /::::::::::::::::::|::::::::\/:::O`、::\   | '、   \
   |      /:::::::::::::::::::/:::::::::::::::::::::::::::::'、::::\ノ  ヽ、  |
  |      |:::::/:::::::::/:::::::::::::::::::::::::::::::::::'、',::::'、  /:\__/‐、
  |      |/:::::::::::/::::::::::::::::::::::::::::::::::O::| '、::| く::::::::::::: ̄|
   |     /_..-'´ ̄`ー-、:::::::::::::::::::::::::::::::::::|/:/`‐'::\;;;;;;;_|
   |    |/::::::::::::::::::::::\:::::::::::::::::::::::::::::|::/::::|::::/:::::::::::/
    |   /:::::::::::::::::::::::::::::::::|:::::::::::::::::::::O::|::|::::::|:::::::::::::::/
170132人目の素数さん:2012/04/20(金) 02:32:00.06
          __ノ)-'´ ̄ ̄`ー- 、_
        , '´  _. -‐'''"二ニニ=-`ヽ、
      /   /:::::; -‐''"        `ーノ
     /   /:::::/           \
     /    /::::::/          | | |  |
     |   |:::::/ /     |  | | | |  |
      |   |::/ / / |  | ||  | | ,ハ .| ,ハ|
      |   |/ / / /| ,ハノ| /|ノレ,ニ|ル' 
     |   |  | / / レ',二、レ′ ,ィイ|゙/   私は只の数ヲタなんかとは付き合わないわ。
.     |   \ ∠イ  ,イイ|    ,`-' |      頭が良くて数学が出来てかっこいい人。それが必要条件よ。
     |     l^,人|  ` `-'     ゝ  |        さらに Ann.of Math に論文書けば十分条件にもなるわよ。
      |      ` -'\       ー'  人          一番嫌いなのは論文数を増やすためにくだらない論文を書いて
    |        /(l     __/  ヽ、           良い論文の出版を遅らせるお馬鹿な人。
     |       (:::::`‐-、__  |::::`、     ヒニニヽ、         あなたの論文が Ann of Math に accept される確率は?
    |      / `‐-、::::::::::`‐-、::::\   /,ニニ、\            それとも最近は Inv. Math. の方が上かしら?
   |      |::::::::::::::::::|` -、:::::::,ヘ ̄|'、  ヒニ二、 \
.   |      /::::::::::::::::::|::::::::\/:::O`、::\   | '、   \
   |      /:::::::::::::::::::/:::::::::::::::::::::::::::::'、::::\ノ  ヽ、  |
  |      |:::::/:::::::::/:::::::::::::::::::::::::::::::::::'、',::::'、  /:\__/‐、
  |      |/:::::::::::/::::::::::::::::::::::::::::::::::O::| '、::| く::::::::::::: ̄|
   |     /_..-'´ ̄`ー-、:::::::::::::::::::::::::::::::::::|/:/`‐'::\;;;;;;;_|
   |    |/::::::::::::::::::::::\:::::::::::::::::::::::::::::|::/::::|::::/:::::::::::/
    |   /:::::::::::::::::::::::::::::::::|:::::::::::::::::::::O::|::|::::::|:::::::::::::::/
171132人目の素数さん:2012/04/20(金) 07:00:13.49
          __ノ)-'´ ̄ ̄`ー- 、_
        , '´  _. -‐'''"二ニニ=-`ヽ、
      /   /:::::; -‐''"        `ーノ
     /   /:::::/           \
     /    /::::::/          | | |  |
     |   |:::::/ /     |  | | | |  |
      |   |::/ / / |  | ||  | | ,ハ .| ,ハ|
      |   |/ / / /| ,ハノ| /|ノレ,ニ|ル' 
     |   |  | / / レ',二、レ′ ,ィイ|゙/   私は只の数ヲタなんかとは付き合わないわ。
.     |   \ ∠イ  ,イイ|    ,`-' |      頭が良くて数学が出来てかっこいい人。それが必要条件よ。
     |     l^,人|  ` `-'     ゝ  |        さらに Ann.of Math に論文書けば十分条件にもなるわよ。
      |      ` -'\       ー'  人          一番嫌いなのは論文数を増やすためにくだらない論文を書いて
    |        /(l     __/  ヽ、           良い論文の出版を遅らせるお馬鹿な人。
     |       (:::::`‐-、__  |::::`、     ヒニニヽ、         あなたの論文が Ann of Math に accept される確率は?
    |      / `‐-、::::::::::`‐-、::::\   /,ニニ、\            それとも最近は Inv. Math. の方が上かしら?
   |      |::::::::::::::::::|` -、:::::::,ヘ ̄|'、  ヒニ二、 \
.   |      /::::::::::::::::::|::::::::\/:::O`、::\   | '、   \
   |      /:::::::::::::::::::/:::::::::::::::::::::::::::::'、::::\ノ  ヽ、  |
  |      |:::::/:::::::::/:::::::::::::::::::::::::::::::::::'、',::::'、  /:\__/‐、
  |      |/:::::::::::/::::::::::::::::::::::::::::::::::O::| '、::| く::::::::::::: ̄|
   |     /_..-'´ ̄`ー-、:::::::::::::::::::::::::::::::::::|/:/`‐'::\;;;;;;;_|
   |    |/::::::::::::::::::::::\:::::::::::::::::::::::::::::|::/::::|::::/:::::::::::/
    |   /:::::::::::::::::::::::::::::::::|:::::::::::::::::::::O::|::|::::::|:::::::::::::::/
172132人目の素数さん:2012/04/20(金) 07:21:06.53
          __ノ)-'´ ̄ ̄`ー- 、_
        , '´  _. -‐'''"二ニニ=-`ヽ、
      /   /:::::; -‐''"        `ーノ
     /   /:::::/           \
     /    /::::::/          | | |  |
     |   |:::::/ /     |  | | | |  |
      |   |::/ / / |  | ||  | | ,ハ .| ,ハ|
      |   |/ / / /| ,ハノ| /|ノレ,ニ|ル' 
     |   |  | / / レ',二、レ′ ,ィイ|゙/   私は只の数ヲタなんかとは付き合わないわ。
.     |   \ ∠イ  ,イイ|    ,`-' |      頭が良くて数学が出来てかっこいい人。それが必要条件よ。
     |     l^,人|  ` `-'     ゝ  |        さらに Ann.of Math に論文書けば十分条件にもなるわよ。
      |      ` -'\       ー'  人          一番嫌いなのは論文数を増やすためにくだらない論文を書いて
    |        /(l     __/  ヽ、           良い論文の出版を遅らせるお馬鹿な人。
     |       (:::::`‐-、__  |::::`、     ヒニニヽ、         あなたの論文が Ann of Math に accept される確率は?
    |      / `‐-、::::::::::`‐-、::::\   /,ニニ、\            それとも最近は Inv. Math. の方が上かしら?
   |      |::::::::::::::::::|` -、:::::::,ヘ ̄|'、  ヒニ二、 \
.   |      /::::::::::::::::::|::::::::\/:::O`、::\   | '、   \
   |      /:::::::::::::::::::/:::::::::::::::::::::::::::::'、::::\ノ  ヽ、  |
  |      |:::::/:::::::::/:::::::::::::::::::::::::::::::::::'、',::::'、  /:\__/‐、
  |      |/:::::::::::/::::::::::::::::::::::::::::::::::O::| '、::| く::::::::::::: ̄|
   |     /_..-'´ ̄`ー-、:::::::::::::::::::::::::::::::::::|/:/`‐'::\;;;;;;;_|
   |    |/::::::::::::::::::::::\:::::::::::::::::::::::::::::|::/::::|::::/:::::::::::/
    |   /:::::::::::::::::::::::::::::::::|:::::::::::::::::::::O::|::|::::::|:::::::::::::::/
173132人目の素数さん:2012/04/20(金) 10:27:10.70
∫∫P(θ,φ)/(4π)ds
174132人目の素数さん:2012/04/20(金) 11:16:19.00
a,bの座標をa(0,0,1),b(sinθcosφ,sinθsinφ,cosθ)とすると
a,bを頂点とする半球の重なり合う部分にcが存在する確率をP(θ)
とすると
P(θ)=(π-θ)/(2π)
求める確率は
∫P(θ)ds/(4π)=∫[0,π/2]dθ∫[0,2π]dφ(π-θ)sinθ/(2π)/(4π)
=(π-1)/(4π)
175132人目の素数さん:2012/04/20(金) 11:36:52.57
訂正
∫P(θ)ds/(4π)=∫[0,π]dθ∫[0,2π]dφ(π-θ)sinθ/(2π)/(4π)
=1/4
176132人目の素数さん:2012/04/20(金) 12:14:45.09
連続関数fがR上可積分であるならば、fは有界ですよね?
177132人目の素数さん:2012/04/20(金) 12:37:59.74
>>176
まるち
178132人目の素数さん:2012/04/20(金) 12:55:18.53
179132人目の素数さん:2012/04/20(金) 13:40:01.85
>>178
すみませんでした
180132人目の素数さん:2012/04/20(金) 14:12:42.45
そもそも積分微分なんて
数学じゃないんだけどな
つまりそれを用いた証明はお世話になります
181132人目の素数さん:2012/04/20(金) 14:32:33.16
僅かながら選挙の匂いが漂ってまいりました。

                    _____________
                    ||                      |
                    ||   ちょっと待て .  .     .|
                    ||         .           |
                    ||    その民主党員  . |
                    |l -――-               |
                     '"´: : : : : : : : :`丶 . 帰化鮮人|
                 ':.:.:.:.:.:.:.:.:.:.:.:.:.:.:.:.:.:.:.:.:.:ヽ______|
                  /.::.::./.::.::.::.:j.::.::.:|.:ム;ヘ.::.:ハ ̄ ̄ ̄ ̄ ̄
                  ,'.::.::.::i.::.::.::.:/|.::.:: l/  `|.::./7
                :.::.::.::j:|.:!.:_:/´|_.::_」   くV <|
                   |:ハ_::_ル'´     /⌒丶 j//V|
               |:::::::::i x==ミ     _ 〈/.:|.::|
               |:::::::::i:'"     ´ ゙̄Y}!.::.l.::|
                 八:::::::圦   、' _   "/_ノ.::,'.::j
             /⌒ヽ::::ト{\   _,.ィ__/.::/l:./
               / 丶∧::| 丶 `ニ´ 彡// :厶|∧
            {/  丶ヘ|     ノ / |:/ (こ ハ
                /       }ヽ、 ∧ /  'x┴〈 }_ゝ、
           /         \∨ ∨  /  ニW }  )
             〈       _ノ∧ 厶=7  ,.-、) 人ノ
           }⌒ヽ     `<__,>イ  |__ノ| |/∨
           /   ヘ   /  │  丶ノ.| |   \
             /    ヽ      \__/ | |    ノ
         /       >'"⌒\ 〃⌒\| ト、__/
182132人目の素数さん:2012/04/20(金) 15:33:10.97
>>174
bの座標はb(sinθ,0,cosθ)として一般性を失わない
座標系の方を回転させればいいから
183132人目の素数さん:2012/04/20(金) 15:43:58.48
集合論なのですが、
Φと{Φ}の違いを教えて下さい。
{Φ}は有限集合で、その要素はΦとΦ…アレ?という状況です。
184132人目の素数さん:2012/04/20(金) 15:53:03.00
空集合Φは元を持たない
集合{Φ}はΦを元に持つのでΦではない
185132人目の素数さん:2012/04/20(金) 15:53:39.84
空箱と空箱が入った箱の違いみたいなもん
186132人目の素数さん:2012/04/20(金) 16:16:52.97
循環小数のときに上につく点の名前ってなんて言うんですか?
塾の先生がわかったらラーメン奢ったるって言ってたんで、30杯ぐらい頼んで泣かせたいです
187132人目の素数さん:2012/04/20(金) 16:37:51.33
初歩的な質問かもしれませんが、線形代数のことで分からないことがあります。
ベクトルの線形性というのは、おおざっぱにいうと
Vの元、a,bの和やスカラー倍が、またVの元になっていることですよね?

で、微分方程式とかで出てくる線形性は、
f(ca+cb)=cf(a)+cf(b)みたいに出来る作用素を線形作用素と呼ぶ。
みたいなやつです。

加算とスカラー倍がキーワードになっているので
直感的に2つが関連していることは分かるのですが
2つは別の概念なのではないのですか?
なぜ、別の概念が同じ「線形性」という言葉で
表わされるのが、腑に落ちません。
どっちかが、より根本的な概念で
もう片方が、それに内包される概念(一例?)みたいなことでしょうか
188132人目の素数さん:2012/04/20(金) 16:42:48.15
準関節
189132人目の素数さん:2012/04/20(金) 16:45:52.41
>>187
おおざっぱにいやー
ベクトルのような演算が出来ることを
線形性がある
と呼んでる

ベクトルの計算のルールだけ抜き出して
そのルールと同じように計算できるなら
線形性がある
と呼んでいる

あとは群論の本でも読んでおく
190132人目の素数さん:2012/04/20(金) 16:50:16.90
>>187
nが自然数の時
n*a↑=Σ[k=1,n](a↑)
とスカラー倍は加算で表現できる
とかじゃいかんの?
191132人目の素数さん:2012/04/20(金) 16:54:00.14
>>187
二つの性質の両方を備えていることを線形と呼ぶんでは?
192132人目の素数さん:2012/04/20(金) 17:14:39.44
一辺10の正方形と同じ面積の正三角形の一辺は?
193132人目の素数さん:2012/04/20(金) 17:18:24.46
>>192
一辺がxの正三角形の面積は?
194132人目の素数さん:2012/04/20(金) 17:22:57.66
>>193
一辺が2の三角形の面積は?
195132人目の素数さん:2012/04/20(金) 17:23:25.08
>>7
196132人目の素数さん:2012/04/20(金) 17:30:50.43
┏  Φ  ┓     ┏   {Φ}   ┓
┃      ┃     ┃┌Φ┐┃
┃      ┃     ┃└─┘┃
┗━━━┛     ┗━━━┛
197132人目の素数さん:2012/04/20(金) 17:45:33.84
>>187
線型空間の定義を確認しろ、線型代数の本に書いてある
198132人目の素数さん:2012/04/20(金) 17:59:51.25
第二問

正八面体の辺上に動点Pがある。
このとき、異なる2頂点A,Bとのなす角(点PがAまたはBに一致するときは0°と定める)∠APBのとり得る値の範囲を求めよ。

これ教えて下さい
199132人目の素数さん:2012/04/20(金) 18:36:15.77
ゆとり乙
200138 ◆U8UAvXc9Z4aQ :2012/04/20(金) 18:41:41.89
>>198
★東大入試作問者になったつもりのスレ★ 第二十問
http://uni.2ch.net/test/read.cgi/math/1325575627/615
これか
201132人目の素数さん:2012/04/20(金) 21:10:26.70
640 名前:名無しさん@12周年[] 投稿日:2012/02/18(土) 15:05:47.13 ID:sskgsjsc0 [2/2]
『平清盛』プロデューサー在日朝鮮人 磯智明(天皇制度廃止論者)のプロデュース作品

@『かんさほうじん (2008)』反体制・反社会
A『最後の戦犯 (2008)』反日・天皇制度廃止・反体制・反社会
B『リミット -刑事の現場2- (2009)』反体制・反社会

大河の画面が汚いのも、役者が大根なのも、衣装がぼろぼろなのも、役者の下品な立ち回りも、
画面が薄暗いのも、役者が汚いのも全ての原因は




NHKが汚れているから
 史実うんぬんの話ではないのですよ。あの大河は役者、セット、演出等が、いまのNHK内部の汚れ具合を見事に反映しているのです。

薄汚れた空間内で繰り広げられる捏造・妄想(=今年の大河)は、反日・在日の脳内を表しているのだ。
202132人目の素数さん:2012/04/20(金) 21:32:11.96
数学の3大分野は代数・解析・幾何で合ってますか?
203132人目の素数さん:2012/04/20(金) 21:37:19.22
>>202
それは証明できることじゃねーぞ
だから合ってるかどうかは言えない
例のコピペに登場してもらいたいけど
スレの品位()を落とすので貼らない
204132人目の素数さん:2012/04/20(金) 21:39:56.52
ガウス曰く「数論は数学の女王」
205132人目の素数さん:2012/04/20(金) 22:12:06.97
lim [n→∞] ( (b/a)^1/n -1 ) / (1/n) = log (b/a)
となってるんですけどどうしてこうなるのかよく分かりません
誰か途中式を教えてください
206132人目の素数さん:2012/04/20(金) 22:19:56.87
( (b/a)^1/n -1 ) / (1/n)
まずはこれをどうにかする
具体的にはlogの定義の式までもってく
207132人目の素数さん:2012/04/20(金) 22:25:33.22
アーベルの総和公式の証明がわかりません(帰納法以外で)
学校の先生に
数列は「関数」であって差が「導関数」である
積の微分公式を用いよ
ってヒントもらったんですが・・・
208132人目の素数さん:2012/04/20(金) 22:39:08.41
>>206
lim[n→∞] 1/n = 0だから lim [n→∞] ( (b/a)^1/n -1 ) / (1/n) = 0 になると思ったんですけど・・
間違ってますかね?
209132人目の素数さん:2012/04/20(金) 22:45:19.01
ベクトル関数の問題ですが、
曲線r(t)=(√2t,t^2/2,log t) (t>0)についてt=1からt=2の曲線の長さを求めよ。
って問題がわかりません。
210132人目の素数さん:2012/04/20(金) 22:46:17.95
>>208
lim [n→∞] (b/a)^(1/n) -1 = 0
lim [n→∞] 1/n = 0
だから0/0型

というか
> lim[n→∞] 1/n = 0だから lim [n→∞] ( (b/a)^1/n -1 ) / (1/n) = 0
ってlim[n→∞] 1/nは分母だぞ
211132人目の素数さん:2012/04/20(金) 22:46:46.21
>>208
ねーよw
分母の1/nをひっくり返せば
n/(n-1) じゃんw

せっかく右がlogになってんだから
チャッチャと片づければいい
212132人目の素数さん:2012/04/20(金) 23:02:37.79
ブサイクは
213132人目の素数さん:2012/04/20(金) 23:20:41.74
>>209
曲線の長さ 積分 パラメータ表示 あたりでぐぐれ
214132人目の素数さん:2012/04/20(金) 23:21:24.84
sin^2θ+sin^2(θ-2π/3)+sin^2(θ-4π/3)=???
215132人目の素数さん:2012/04/20(金) 23:23:20.63
640 名前:名無しさん@12周年[] 投稿日:2012/02/18(土) 15:05:47.13 ID:sskgsjsc0 [2/2]
『平清盛』プロデューサー在日朝鮮人 磯智明(天皇制度廃止論者)のプロデュース作品

@『かんさほうじん (2008)』反体制・反社会
A『最後の戦犯 (2008)』反日・天皇制度廃止・反体制・反社会
B『リミット -刑事の現場2- (2009)』反体制・反社会

大河の画面が汚いのも、役者が大根なのも、衣装がぼろぼろなのも、役者の下品な立ち回りも、
画面が薄暗いのも、役者が汚いのも全ての原因は




NHKが汚れているから
 史実うんぬんの話ではないのですよ。あの大河は役者、セット、演出等が、いまのNHK内部の汚れ具合を見事に反映しているのです。

薄汚れた空間内で繰り広げられる捏造・妄想(=今年の大河)は、反日・在日の脳内を表しているのだ。
216132人目の素数さん:2012/04/20(金) 23:46:23.57
          __ノ)-'´ ̄ ̄`ー- 、_
        , '´  _. -‐'''"二ニニ=-`ヽ、
      /   /:::::; -‐''"        `ーノ
     /   /:::::/           \
     /    /::::::/          | | |  |
     |   |:::::/ /     |  | | | |  |
      |   |::/ / / |  | ||  | | ,ハ .| ,ハ|
      |   |/ / / /| ,ハノ| /|ノレ,ニ|ル' 
     |   |  | / / レ',二、レ′ ,ィイ|゙/   私は只の数ヲタなんかとは付き合わないわ。
.     |   \ ∠イ  ,イイ|    ,`-' |      頭が良くて数学が出来てかっこいい人。それが必要条件よ。
     |     l^,人|  ` `-'     ゝ  |        さらに Ann.of Math に論文書けば十分条件にもなるわよ。
      |      ` -'\       ー'  人          一番嫌いなのは論文数を増やすためにくだらない論文を書いて
    |        /(l     __/  ヽ、           良い論文の出版を遅らせるお馬鹿な人。
     |       (:::::`‐-、__  |::::`、     ヒニニヽ、         あなたの論文が Ann of Math に accept される確率は?
    |      / `‐-、::::::::::`‐-、::::\   /,ニニ、\            それとも最近は Inv. Math. の方が上かしら?
   |      |::::::::::::::::::|` -、:::::::,ヘ ̄|'、  ヒニ二、 \
.   |      /::::::::::::::::::|::::::::\/:::O`、::\   | '、   \
   |      /:::::::::::::::::::/:::::::::::::::::::::::::::::'、::::\ノ  ヽ、  |
  |      |:::::/:::::::::/:::::::::::::::::::::::::::::::::::'、',::::'、  /:\__/‐、
  |      |/:::::::::::/::::::::::::::::::::::::::::::::::O::| '、::| く::::::::::::: ̄|
   |     /_..-'´ ̄`ー-、:::::::::::::::::::::::::::::::::::|/:/`‐'::\;;;;;;;_|
   |    |/::::::::::::::::::::::\:::::::::::::::::::::::::::::|::/::::|::::/:::::::::::/
    |   /:::::::::::::::::::::::::::::::::|:::::::::::::::::::::O::|::|::::::|:::::::::::::::/
217132人目の素数さん:2012/04/21(土) 00:04:15.68
科学発展に寄与しない
ゴシップ記事を書いている
メスはなぜ
馬鹿なん?
枕営業?
218132人目の素数さん:2012/04/21(土) 01:02:28.60
お前は何か科学の発展に寄与する成果を上げたことがあるの?
219132人目の素数さん:2012/04/21(土) 02:18:48.22

(y/x)y' -1 = 0

上の微分方程式を変数分離して一般解を求めよ
また、x=3,y=2の特殊解もあわせて求めなさい

この問題の解答おねがいします!

220132人目の素数さん:2012/04/21(土) 02:27:57.45
An+1=√(An+1) A1=1 Anの極限を求めよ。


An+1=3An +4/2An +3 A1=1 Anの極限を求めよ。


解き方がぜんぜんわかりません。教科書は答えしかないですし。あと大学の数学は
普通に教授にアポとれば教えてくれるものですか?高校と違って聞きづらいのですが。


質問多くてすいませんが、解説が詳しいいい問題集とかありますか?大学基礎レベルで。
221132人目の素数さん:2012/04/21(土) 02:33:04.73
>>219
y y' = x
2y y' = 2x

y^2 = x^2 +c
222132人目の素数さん:2012/04/21(土) 02:34:09.98
>>220
もうちょっと伝わるように考えて数式を書かないと
どうにもならん。
223132人目の素数さん:2012/04/21(土) 02:37:36.43
224132人目の素数さん:2012/04/21(土) 03:29:21.15
>>220
(1) a[1] = 1 , a[n+1] = √( a[n] + 1 )
(2) a[1] = 1 , a[n+1] = ( 3a[n] + 4 )/( 2a[n] + 3 )
ってことですかね
いずれも極限値の存在を仮定してそれをαとおいて
漸化式で a[n] , a[n+1] をαに置き換えた方程式を解けば答えだけはわかる
高校の数学Vの参考書を探せばちゃんとしたやり方が出ている
225132人目の素数さん:2012/04/21(土) 03:50:25.97
うちの大学だと、アポなんて取らずにいきなり押し掛けても全然OKだった
連絡が必要なのは教授が研究室に全然寄り付かないような人だった場合ぐらい
226132人目の素数さん:2012/04/21(土) 04:39:45.11
複素関数のテイラー級数に関する質問です。

アールフォルス『複素解析』のP.193に次のようにあります。

定理3 f(z)がz_0を含む領域Ωで解析的とすると,z_0を中心とするΩに含まれる最大の開円板において
f(z)=f(z_0)+(f'(z_0)/1!)(z-z_0)+...+(f^(n)(z_0)/n!)(z-z_0)^n+...が成り立つ。
・・・
このテイラー級数の収束半径は、z_0からΩの境界までの最短距離以上である。それは本当に大きいこともある。
しかしそのとき、この級数がΩと収束円の共通部分の全ての点でf(z)を表しているという保証はない。


以下では上記級数で表される関数をg(z)とします。引用文によるとΩと収束円の共通部分では必ずしもf(z)=g(z)ではないということですが、
Ωと収束円の共通部分では、各点を中心とする十分小さい閉円板上でこの級数はzに関して一様に収束し、
g(z)は解析関数となり、一致の定理よりf(z)=g(z)となるのではないかと思うのです。何が間違っているのでしょうか?
また、f(z)≠g(z)となる例があれば教えてください。
227132人目の素数さん:2012/04/21(土) 04:51:37.53
Ωが連結でなくて、別の分岐になってる場合。
(平方根の別の根に切り替わってる場合とか)
228132人目の素数さん:2012/04/21(土) 05:03:40.54
レスありがとうございます。おかげで解決しました。
この本では領域と言えば連結開集合のことなのですが、あまり一般的ではないのかもしれませんね。言葉が足りずすみません。
ですが、たとえΩの連結性を仮定しても、開円板の共通部分が連結になるとは限らないので同じですね。
229132人目の素数さん:2012/04/21(土) 07:10:54.10
テイラーは誤差の範囲で一致するだけで完全に同じじゃない。
解析接続は一致したルートでテイラーを拡張してくだけ。
1<>0。9999999・・・・
230132人目の素数さん:2012/04/21(土) 07:43:19.95
          __ノ)-'´ ̄ ̄`ー- 、_
        , '´  _. -‐'''"二ニニ=-`ヽ、
      /   /:::::; -‐''"        `ーノ
     /   /:::::/           \
     /    /::::::/          | | |  |
     |   |:::::/ /     |  | | | |  |
      |   |::/ / / |  | ||  | | ,ハ .| ,ハ|
      |   |/ / / /| ,ハノ| /|ノレ,ニ|ル' 
     |   |  | / / レ',二、レ′ ,ィイ|゙/   私は只の数ヲタなんかとは付き合わないわ。
.     |   \ ∠イ  ,イイ|    ,`-' |      頭が良くて数学が出来てかっこいい人。それが必要条件よ。
     |     l^,人|  ` `-'     ゝ  |        さらに Ann.of Math に論文書けば十分条件にもなるわよ。
      |      ` -'\       ー'  人          一番嫌いなのは論文数を増やすためにくだらない論文を書いて
    |        /(l     __/  ヽ、           良い論文の出版を遅らせるお馬鹿な人。
     |       (:::::`‐-、__  |::::`、     ヒニニヽ、         あなたの論文が Ann of Math に accept される確率は?
    |      / `‐-、::::::::::`‐-、::::\   /,ニニ、\            それとも最近は Inv. Math. の方が上かしら?
   |      |::::::::::::::::::|` -、:::::::,ヘ ̄|'、  ヒニ二、 \
.   |      /::::::::::::::::::|::::::::\/:::O`、::\   | '、   \
   |      /:::::::::::::::::::/:::::::::::::::::::::::::::::'、::::\ノ  ヽ、  |
  |      |:::::/:::::::::/:::::::::::::::::::::::::::::::::::'、',::::'、  /:\__/‐、
  |      |/:::::::::::/::::::::::::::::::::::::::::::::::O::| '、::| く::::::::::::: ̄|
   |     /_..-'´ ̄`ー-、:::::::::::::::::::::::::::::::::::|/:/`‐'::\;;;;;;;_|
   |    |/::::::::::::::::::::::\:::::::::::::::::::::::::::::|::/::::|::::/:::::::::::/
    |   /:::::::::::::::::::::::::::::::::|:::::::::::::::::::::O::|::|::::::|:::::::::::::::/
231132人目の素数さん:2012/04/21(土) 07:59:41.07
          __ノ)-'´ ̄ ̄`ー- 、_
        , '´  _. -‐'''"二ニニ=-`ヽ、
      /   /:::::; -‐''"        `ーノ
     /   /:::::/           \
     /    /::::::/          | | |  |
     |   |:::::/ /     |  | | | |  |
      |   |::/ / / |  | ||  | | ,ハ .| ,ハ|
      |   |/ / / /| ,ハノ| /|ノレ,ニ|ル' 
     |   |  | / / レ',二、レ′ ,ィイ|゙/   私は只の数ヲタなんかとは付き合わないわ。
.     |   \ ∠イ  ,イイ|    ,`-' |      頭が良くて数学が出来てかっこいい人。それが必要条件よ。
     |     l^,人|  ` `-'     ゝ  |        さらに Ann.of Math に論文書けば十分条件にもなるわよ。
      |      ` -'\       ー'  人          一番嫌いなのは論文数を増やすためにくだらない論文を書いて
    |        /(l     __/  ヽ、           良い論文の出版を遅らせるお馬鹿な人。
     |       (:::::`‐-、__  |::::`、     ヒニニヽ、         あなたの論文が Ann of Math に accept される確率は?
    |      / `‐-、::::::::::`‐-、::::\   /,ニニ、\            それとも最近は Inv. Math. の方が上かしら?
   |      |::::::::::::::::::|` -、:::::::,ヘ ̄|'、  ヒニ二、 \
.   |      /::::::::::::::::::|::::::::\/:::O`、::\   | '、   \
   |      /:::::::::::::::::::/:::::::::::::::::::::::::::::'、::::\ノ  ヽ、  |
  |      |:::::/:::::::::/:::::::::::::::::::::::::::::::::::'、',::::'、  /:\__/‐、
  |      |/:::::::::::/::::::::::::::::::::::::::::::::::O::| '、::| く::::::::::::: ̄|
   |     /_..-'´ ̄`ー-、:::::::::::::::::::::::::::::::::::|/:/`‐'::\;;;;;;;_|
   |    |/::::::::::::::::::::::\:::::::::::::::::::::::::::::|::/::::|::::/:::::::::::/
    |   /:::::::::::::::::::::::::::::::::|:::::::::::::::::::::O::|::|::::::|:::::::::::::::/
232132人目の素数さん:2012/04/21(土) 13:25:07.24
220です。ありがとうございます。なんか高校の数Vもほとんど忘れているみたいです。
復習しながら、授業を受けたいと思います。
233132人目の素数さん:2012/04/21(土) 13:25:22.36
          __ノ)-'´ ̄ ̄`ー- 、_
        , '´  _. -‐'''"二ニニ=-`ヽ、
      /   /:::::; -‐''"        `ーノ
     /   /:::::/           \
     /    /::::::/          | | |  |
     |   |:::::/ /     |  | | | |  |
      |   |::/ / / |  | ||  | | ,ハ .| ,ハ|
      |   |/ / / /| ,ハノ| /|ノレ,ニ|ル' 
     |   |  | / / レ',二、レ′ ,ィイ|゙/   私は只の数ヲタなんかとは付き合わないわ。
.     |   \ ∠イ  ,イイ|    ,`-' |      頭が良くて数学が出来てかっこいい人。それが必要条件よ。
     |     l^,人|  ` `-'     ゝ  |        さらに Ann.of Math に論文書けば十分条件にもなるわよ。
      |      ` -'\       ー'  人          一番嫌いなのは論文数を増やすためにくだらない論文を書いて
    |        /(l     __/  ヽ、           良い論文の出版を遅らせるお馬鹿な人。
     |       (:::::`‐-、__  |::::`、     ヒニニヽ、         あなたの論文が Ann of Math に accept される確率は?
    |      / `‐-、::::::::::`‐-、::::\   /,ニニ、\            それとも最近は Inv. Math. の方が上かしら?
   |      |::::::::::::::::::|` -、:::::::,ヘ ̄|'、  ヒニ二、 \
.   |      /::::::::::::::::::|::::::::\/:::O`、::\   | '、   \
   |      /:::::::::::::::::::/:::::::::::::::::::::::::::::'、::::\ノ  ヽ、  |
  |      |:::::/:::::::::/:::::::::::::::::::::::::::::::::::'、',::::'、  /:\__/‐、
  |      |/:::::::::::/::::::::::::::::::::::::::::::::::O::| '、::| く::::::::::::: ̄|
   |     /_..-'´ ̄`ー-、:::::::::::::::::::::::::::::::::::|/:/`‐'::\;;;;;;;_|
   |    |/::::::::::::::::::::::\:::::::::::::::::::::::::::::|::/::::|::::/:::::::::::/
    |   /:::::::::::::::::::::::::::::::::|:::::::::::::::::::::O::|::|::::::|:::::::::::::::/
234132人目の素数さん:2012/04/21(土) 13:31:44.59
>>214はどうなるの?っと
235132人目の素数さん:2012/04/21(土) 13:41:21.82
ンなモン
整流した三相交流だろjk
236ようじょ ◆hNziS2E8421X :2012/04/21(土) 14:38:54.97
ベータ関数B(p,q)とガンマ関数Γ(p)の間に
B(p,q)=Γ(p)Γ(q)/Γ(p+q)なる等式が成り立つことを示せ

がわかりません!おしえてください!
237132人目の素数さん:2012/04/21(土) 15:04:39.72
          __ノ)-'´ ̄ ̄`ー- 、_
        , '´  _. -‐'''"二ニニ=-`ヽ、
      /   /:::::; -‐''"        `ーノ
     /   /:::::/           \
     /    /::::::/          | | |  |
     |   |:::::/ /     |  | | | |  |
      |   |::/ / / |  | ||  | | ,ハ .| ,ハ|
      |   |/ / / /| ,ハノ| /|ノレ,ニ|ル' 
     |   |  | / / レ',二、レ′ ,ィイ|゙/   私は只の数ヲタなんかとは付き合わないわ。
.     |   \ ∠イ  ,イイ|    ,`-' |      頭が良くて数学が出来てかっこいい人。それが必要条件よ。
     |     l^,人|  ` `-'     ゝ  |        さらに Ann.of Math に論文書けば十分条件にもなるわよ。
      |      ` -'\       ー'  人          一番嫌いなのは論文数を増やすためにくだらない論文を書いて
    |        /(l     __/  ヽ、           良い論文の出版を遅らせるお馬鹿な人。
     |       (:::::`‐-、__  |::::`、     ヒニニヽ、         あなたの論文が Ann of Math に accept される確率は?
    |      / `‐-、::::::::::`‐-、::::\   /,ニニ、\            それとも最近は Inv. Math. の方が上かしら?
   |      |::::::::::::::::::|` -、:::::::,ヘ ̄|'、  ヒニ二、 \
.   |      /::::::::::::::::::|::::::::\/:::O`、::\   | '、   \
   |      /:::::::::::::::::::/:::::::::::::::::::::::::::::'、::::\ノ  ヽ、  |
  |      |:::::/:::::::::/:::::::::::::::::::::::::::::::::::'、',::::'、  /:\__/‐、
  |      |/:::::::::::/::::::::::::::::::::::::::::::::::O::| '、::| く::::::::::::: ̄|
   |     /_..-'´ ̄`ー-、:::::::::::::::::::::::::::::::::::|/:/`‐'::\;;;;;;;_|
   |    |/::::::::::::::::::::::\:::::::::::::::::::::::::::::|::/::::|::::/:::::::::::/
    |   /:::::::::::::::::::::::::::::::::|:::::::::::::::::::::O::|::|::::::|:::::::::::::::/
238132人目の素数さん:2012/04/21(土) 15:40:00.33
>>236
そんなことはどうでもいいから、姉の写真アップしる
239132人目の素数さん:2012/04/21(土) 15:52:39.46
「国の借金」について

日頃メディアや、反日工作員が必死になって「国の借金」という単語を使い
財政破綻論を展開させていますが、現実、現在の日本には「政府の借金」1000兆円近く存在いたしますが、
「国の借金」は存在いたしません。

朝日新聞やNHKは、雇い主である中国共産党より日本人に対して不安や政府に対する不信を持たせ、煽るために
局内の共産党員を使用して既に数十年間、「国の借金」を連呼し続けております。

<違和感なく「国の借金が1000兆円もある」という幻想に浸ってしまっている一般の方々は、朝日新聞やNHKに見事に騙されて続けている訳です>

数十年もテレビや新聞から情報を得てきた方々の中には、「メディアが嘘を付く訳ない」と思う、そう思いたい方もいるでしょう。
  しかし、長い目で見れば、もともと戦前から日本を転覆させるために存在してきた報道機関ですから、
これくらいの嘘は朝飯前で御座います。

それでも、「国の借金は1000兆円ある」と考えをお持ちの方は、複式簿記の勉強をしてから、日本のバランスシートをご覧ください。
 
  実質中国の広報機関であるNHK、朝日新聞は、これからも嘘を付き続けます。デマを流し続けます。捏造し続けます。
   
 ---そのニュース 核心はヤラセだ       長文失礼いたしました。---

240132人目の素数さん:2012/04/21(土) 16:15:33.95
対数らせんについてです。
2辺の長さがaの2等辺三角形ABC(角Aが直角)を作るとすると、
BCの長さは(√2)aになります。
次に、BCを1辺としAと逆の方向にBCに直交するようにCD=aとする直角三角形を作ると
BDの長さは(√3)aになります。
これを続けて、(√4)a、(√5)a、、、と作っていくときに、

頂点A、C、D、、、を結ぶ(?)と対数らせんに近い形になると思います。
このときの対数らせんを式であらわすには、
どのように式をもとめたらよいでしょうか。

対数らせん(パラメータ表示)
r(Θ)=(aexp(bΘ)cosΘ,aexp(bΘ)sinΘ)
とすると、

aは直角三角形のスタートの辺の長さaだと思うのですが、
bはどのような値になるのでしょうか…。
241132人目の素数さん:2012/04/21(土) 16:20:24.88
すいませんが、「トポロジー入門 小島定吉著 P45」記載の内容について
質問させて下さい。

【本に記載】
有限セル複体の間の任意の連続写像は階層を保つ写像にホモトピックである。
(胞体近似定理?)

*階層を保つ写像(胞体写像?)
各骨格の像が、同じ次元の骨格に含まれる。

【質問】
S^1->S^1への写像で、
定値写像は連続写像。恒等写像は連続かつ階層を保つ写像。
2つの写像は上記条件は満たしていると思う。
しかし、この2つの写像はホモトピックではない??。
何を勘違いしているのでしょうか?
242132人目の素数さん:2012/04/21(土) 17:08:53.67
>>240
対数らせんに近い形になると思うのはどうして?
243132人目の素数さん:2012/04/21(土) 17:25:13.35
>>240
ちょっと図描いて上げてみてよ
244132人目の素数さん:2012/04/21(土) 17:52:22.43
          __ノ)-'´ ̄ ̄`ー- 、_
        , '´  _. -‐'''"二ニニ=-`ヽ、
      /   /:::::; -‐''"        `ーノ
     /   /:::::/           \
     /    /::::::/          | | |  |
     |   |:::::/ /     |  | | | |  |
      |   |::/ / / |  | ||  | | ,ハ .| ,ハ|
      |   |/ / / /| ,ハノ| /|ノレ,ニ|ル' 
     |   |  | / / レ',二、レ′ ,ィイ|゙/   私は只の数ヲタなんかとは付き合わないわ。
.     |   \ ∠イ  ,イイ|    ,`-' |      頭が良くて数学が出来てかっこいい人。それが必要条件よ。
     |     l^,人|  ` `-'     ゝ  |        さらに Ann.of Math に論文書けば十分条件にもなるわよ。
      |      ` -'\       ー'  人          一番嫌いなのは論文数を増やすためにくだらない論文を書いて
    |        /(l     __/  ヽ、           良い論文の出版を遅らせるお馬鹿な人。
     |       (:::::`‐-、__  |::::`、     ヒニニヽ、         あなたの論文が Ann of Math に accept される確率は?
    |      / `‐-、::::::::::`‐-、::::\   /,ニニ、\            それとも最近は Inv. Math. の方が上かしら?
   |      |::::::::::::::::::|` -、:::::::,ヘ ̄|'、  ヒニ二、 \
.   |      /::::::::::::::::::|::::::::\/:::O`、::\   | '、   \
   |      /:::::::::::::::::::/:::::::::::::::::::::::::::::'、::::\ノ  ヽ、  |
  |      |:::::/:::::::::/:::::::::::::::::::::::::::::::::::'、',::::'、  /:\__/‐、
  |      |/:::::::::::/::::::::::::::::::::::::::::::::::O::| '、::| く::::::::::::: ̄|
   |     /_..-'´ ̄`ー-、:::::::::::::::::::::::::::::::::::|/:/`‐'::\;;;;;;;_|
   |    |/::::::::::::::::::::::\:::::::::::::::::::::::::::::|::/::::|::::/:::::::::::/
    |   /:::::::::::::::::::::::::::::::::|:::::::::::::::::::::O::|::|::::::|:::::::::::::::/
245132人目の素数さん:2012/04/21(土) 18:02:40.67
第二問

正八面体の辺上に動点Pがある。
このとき、異なる2頂点A,Bとのなす角(点PがAまたはBに一致するときは0°と定める)∠APBのとり得る値の範囲を求めよ。

これ教えて下さい
246132人目の素数さん:2012/04/21(土) 18:18:45.28
>>221
ありがとうございます

解説から一つ疑問点がうかんだので、よろしいですか

2y y' = 2x は何故、2が追加されるのでしょうか?
積分であれば1/2のはずです
この理由を教えてください
247132人目の素数さん:2012/04/21(土) 18:21:14.17
248132人目の素数さん:2012/04/21(土) 18:22:41.14
>>246
等式の両辺を各々2倍したんだろう
249132人目の素数さん:2012/04/21(土) 18:24:32.03
>>236
>ベータ関数B(p,q)とガンマ関数Γ(p)
各々の定義は?
250132人目の素数さん:2012/04/21(土) 18:39:32.94
>>249
アホかよ
半年ROMれ
251132人目の素数さん:2012/04/21(土) 18:47:22.46
>>250
ベータ関数を
B(p,q)=Γ(p)Γ(q)/Γ(p+q)
で定義したらいかんのかって話
252132人目の素数さん:2012/04/21(土) 19:27:29.74
xの整式p(x)はx-aで割り切れ、その時の商をQ(x)とする。また、Q(x)をx-bで割ると
商がx,余りが3となる。ただし、a,bは実数の定数である

Q(x)をbを用いて表せ
また、方程式P(x)=0が虚数解を持つようなbの値の範囲を求めよ。

分かる人教えて下さい。
253132人目の素数さん:2012/04/21(土) 19:30:45.19
>>251
マジもんのアホかよwww
2年は書き込むなw
アホがうつるwwwww
254132人目の素数さん:2012/04/21(土) 19:58:48.00
>>241
きみ何言ってんの?ぶっとばすよ?
255132人目の素数さん:2012/04/21(土) 20:02:42.08
∬(x^2)dxdy

条件:{(x^2)/(a^2)} + {(y^2)/(b^2)} <= 1 , a>0 , b>0

以上の二重積分の値を教えていただきたいです。
よろしくお願いいたします。
256132人目の素数さん:2012/04/21(土) 20:10:12.30
わーい!わーい!
257132人目の素数さん:2012/04/21(土) 20:17:54.81
>>252
P(x)=(x-a)Q(x)
Q(x)=x(x-b)+3
258132人目の素数さん:2012/04/21(土) 21:06:18.16
>>242 Bを原点とするとそこからA,C,D,,,までの距離が大きくなっていくので、
あとB(原点)中心に一方向に回転してるので、対数螺旋になるのかなと思いました。
259132人目の素数さん:2012/04/21(土) 21:06:55.30
          __ノ)-'´ ̄ ̄`ー- 、_
        , '´  _. -‐'''"二ニニ=-`ヽ、
      /   /:::::; -‐''"        `ーノ
     /   /:::::/           \
     /    /::::::/          | | |  |
     |   |:::::/ /     |  | | | |  |
      |   |::/ / / |  | ||  | | ,ハ .| ,ハ|
      |   |/ / / /| ,ハノ| /|ノレ,ニ|ル' 
     |   |  | / / レ',二、レ′ ,ィイ|゙/   私は只の数ヲタなんかとは付き合わないわ。
.     |   \ ∠イ  ,イイ|    ,`-' |      頭が良くて数学が出来てかっこいい人。それが必要条件よ。
     |     l^,人|  ` `-'     ゝ  |        さらに Ann.of Math に論文書けば十分条件にもなるわよ。
      |      ` -'\       ー'  人          一番嫌いなのは論文数を増やすためにくだらない論文を書いて
    |        /(l     __/  ヽ、           良い論文の出版を遅らせるお馬鹿な人。
     |       (:::::`‐-、__  |::::`、     ヒニニヽ、         あなたの論文が Ann of Math に accept される確率は?
    |      / `‐-、::::::::::`‐-、::::\   /,ニニ、\            それとも最近は Inv. Math. の方が上かしら?
   |      |::::::::::::::::::|` -、:::::::,ヘ ̄|'、  ヒニ二、 \
.   |      /::::::::::::::::::|::::::::\/:::O`、::\   | '、   \
   |      /:::::::::::::::::::/:::::::::::::::::::::::::::::'、::::\ノ  ヽ、  |
  |      |:::::/:::::::::/:::::::::::::::::::::::::::::::::::'、',::::'、  /:\__/‐、
  |      |/:::::::::::/::::::::::::::::::::::::::::::::::O::| '、::| く::::::::::::: ̄|
   |     /_..-'´ ̄`ー-、:::::::::::::::::::::::::::::::::::|/:/`‐'::\;;;;;;;_|
   |    |/::::::::::::::::::::::\:::::::::::::::::::::::::::::|::/::::|::::/:::::::::::/
    |   /:::::::::::::::::::::::::::::::::|:::::::::::::::::::::O::|::|::::::|:::::::::::::::/
260132人目の素数さん:2012/04/21(土) 21:07:55.49
          __ノ)-'´ ̄ ̄`ー- 、_
        , '´  _. -‐'''"二ニニ=-`ヽ、
      /   /:::::; -‐''"        `ーノ
     /   /:::::/           \
     /    /::::::/          | | |  |
     |   |:::::/ /     |  | | | |  |
      |   |::/ / / |  | ||  | | ,ハ .| ,ハ|
      |   |/ / / /| ,ハノ| /|ノレ,ニ|ル' 
     |   |  | / / レ',二、レ′ ,ィイ|゙/   私は只の数ヲタなんかとは付き合わないわ。
.     |   \ ∠イ  ,イイ|    ,`-' |      頭が良くて数学が出来てかっこいい人。それが必要条件よ。
     |     l^,人|  ` `-'     ゝ  |        さらに Ann.of Math に論文書けば十分条件にもなるわよ。
      |      ` -'\       ー'  人          一番嫌いなのは論文数を増やすためにくだらない論文を書いて
    |        /(l     __/  ヽ、           良い論文の出版を遅らせるお馬鹿な人。
     |       (:::::`‐-、__  |::::`、     ヒニニヽ、         あなたの論文が Ann of Math に accept される確率は?
    |      / `‐-、::::::::::`‐-、::::\   /,ニニ、\            それとも最近は Inv. Math. の方が上かしら?
   |      |::::::::::::::::::|` -、:::::::,ヘ ̄|'、  ヒニ二、 \
.   |      /::::::::::::::::::|::::::::\/:::O`、::\   | '、   \
   |      /:::::::::::::::::::/:::::::::::::::::::::::::::::'、::::\ノ  ヽ、  |
  |      |:::::/:::::::::/:::::::::::::::::::::::::::::::::::'、',::::'、  /:\__/‐、
  |      |/:::::::::::/::::::::::::::::::::::::::::::::::O::| '、::| く::::::::::::: ̄|
   |     /_..-'´ ̄`ー-、:::::::::::::::::::::::::::::::::::|/:/`‐'::\;;;;;;;_|
   |    |/::::::::::::::::::::::\:::::::::::::::::::::::::::::|::/::::|::::/:::::::::::/
    |   /:::::::::::::::::::::::::::::::::|:::::::::::::::::::::O::|::|::::::|:::::::::::::::/
261132人目の素数さん:2012/04/21(土) 21:34:58.84
>>240
r/aが大きいところでは対数螺線でなくアルキメデスの螺線に近いようだ。
262132人目の素数さん:2012/04/21(土) 21:36:45.24
>>245
よくわからんけど、
問題文から、点PがAまたはBに一致するときの0゜が最小で、
ABが隣接する頂点の場合、点Pが辺AB上にあれば180゜でこれが最大じゃないの?
263132人目の素数さん:2012/04/21(土) 21:41:36.25
>>262
あ、そうか...
ありがとうございます
264132人目の素数さん:2012/04/21(土) 21:53:51.57
>二次式を複素数の範囲で因数分解せよ

この「複素数の範囲」とはどういう意味ですか?
265132人目の素数さん:2012/04/21(土) 21:57:11.01
チンポいつもお世話になります
266132人目の素数さん:2012/04/21(土) 22:00:28.84
カンタンに言えば
「虚数iが登場してもいいですよ」
てな意味
267132人目の素数さん:2012/04/21(土) 22:04:08.39
複素数を使ってもいいですよてことです
268132人目の素数さん:2012/04/21(土) 22:04:16.24
しょうがねぇ、今回は虚数も登場させてもいいぞ
269132人目の素数さん:2012/04/21(土) 22:06:36.48
γ行列を使ってもいいということですよ
270132人目の素数さん:2012/04/21(土) 22:25:21.05
この問題、どのように解くのでしょうか…
行き詰まってしまっています。
どなたかわかりましたら解説お願いします。

次の極限値を求めよ
lim(x→2) 1/(x^2−4) インテグラル(2→x)t^4e^t dt
271132人目の素数さん:2012/04/21(土) 22:42:55.16
∫の部分は分母に入るのか?
それとも上か?
どっち?
括弧が足りない
272132人目の素数さん:2012/04/21(土) 23:50:36.93
248 名前:名無しさん@12周年[] 投稿日:2012/04/18(水) 10:00:41.07 ID:/XMjSBlc0
                    ,,,,,,,,,,,,,,,,,,,,,,,     
                   /::::::::::::::::::::::::ヾヽ
                  .i:::::::://///::::::::::::::::::|  放尿ステーション 
                 .|::::::/ ::////⌒⌒ i.:::::ノ   古田チン子伊恥朗
                .|:::::/         |::::|
                 |::/.  .ヘ    ヘ.  |::|
                 .⊥|.-(=・).-.(・=)-.|⊥   | ̄ ̄ ̄ ̄ ̄ ̄ ̄ ̄ ̄
                l .!:;  ⌒´.し.`⌒  ::|. l   .|あっあーと、なんということでしょう
                 ゝ.ヘ         /ィ  _ノ 元総理 鳩山さん落選です
               __,. -‐ヘ  <ニ二ニ>  /─- __ ̄ ̄ ̄ ̄ ̄ ̄ ̄ ̄ ̄ ̄
          _ -‐ ''"   / !\  ̄ /!\     ゙̄ー- 、
         ハ       /.  |ヽ  ̄ ̄ /|  ヽ        ハ  
 ̄ ̄ ̄ ̄○ ̄ ̄ ̄ ̄ ̄ ̄ ̄ ̄ ̄ ̄ ̄ ̄ ̄ ̄ ̄ ̄ ̄ ̄ ̄ ̄
       O
         。ノ´⌒ヽ,,               
ガバっ! γ⌒´      ヽ,       
      // ""⌒⌒\  ):     
     :i / ⌒\ /  ヽ .)     うわああああ・・・・・。 
      !゙:li|.:(○)三(○):.゙!  
      | U::::: ((__人__):::u| : 
      \ u  |:┬:|  /    
   _〃/:::::::: `ー'  ::ヽ____   
   |   |::::::::      :::::::|      |
   |\⌒⌒⌒⌒⌒⌒⌒⌒⌒⌒⌒..\
   \ \          zzz     \
.     \  \    γ⌒ヽハ,,ハ       \
        \  \  ("_) (-ェ-,,)        \
        \  |⌒⌒⌒⌒⌒⌒⌒⌒⌒⌒⌒.|
           \|____________|
273132人目の素数さん:2012/04/21(土) 23:52:01.52
>>248
>等式の両辺を各々2倍したんだろう

・・?、2倍にするのはなぜですか
274132人目の素数さん:2012/04/21(土) 23:53:05.03
>>270
多分 インテグラル(2→x)t^4e^t dt =F(x)-F(2) などと考えるんだろう
275132人目の素数さん:2012/04/21(土) 23:55:16.69
>>273
解説者が x=d(x^2/2)/dx より 2x=d(x^2)/dx の方を好んだんじゃない?
276132人目の素数さん:2012/04/21(土) 23:58:36.24
          __ノ)-'´ ̄ ̄`ー- 、_
        , '´  _. -‐'''"二ニニ=-`ヽ、
      /   /:::::; -‐''"        `ーノ
     /   /:::::/           \
     /    /::::::/          | | |  |
     |   |:::::/ /     |  | | | |  |
      |   |::/ / / |  | ||  | | ,ハ .| ,ハ|
      |   |/ / / /| ,ハノ| /|ノレ,ニ|ル' 
     |   |  | / / レ',二、レ′ ,ィイ|゙/   私は只の数ヲタなんかとは付き合わないわ。
.     |   \ ∠イ  ,イイ|    ,`-' |      頭が良くて数学が出来てかっこいい人。それが必要条件よ。
     |     l^,人|  ` `-'     ゝ  |        さらに Ann.of Math に論文書けば十分条件にもなるわよ。
      |      ` -'\       ー'  人          一番嫌いなのは論文数を増やすためにくだらない論文を書いて
    |        /(l     __/  ヽ、           良い論文の出版を遅らせるお馬鹿な人。
     |       (:::::`‐-、__  |::::`、     ヒニニヽ、         あなたの論文が Ann of Math に accept される確率は?
    |      / `‐-、::::::::::`‐-、::::\   /,ニニ、\            それとも最近は Inv. Math. の方が上かしら?
   |      |::::::::::::::::::|` -、:::::::,ヘ ̄|'、  ヒニ二、 \
.   |      /::::::::::::::::::|::::::::\/:::O`、::\   | '、   \
   |      /:::::::::::::::::::/:::::::::::::::::::::::::::::'、::::\ノ  ヽ、  |
  |      |:::::/:::::::::/:::::::::::::::::::::::::::::::::::'、',::::'、  /:\__/‐、
  |      |/:::::::::::/::::::::::::::::::::::::::::::::::O::| '、::| く::::::::::::: ̄|
   |     /_..-'´ ̄`ー-、:::::::::::::::::::::::::::::::::::|/:/`‐'::\;;;;;;;_|
   |    |/::::::::::::::::::::::\:::::::::::::::::::::::::::::|::/::::|::::/:::::::::::/
    |   /:::::::::::::::::::::::::::::::::|:::::::::::::::::::::O::|::|::::::|:::::::::::::::/
277132人目の素数さん:2012/04/22(日) 01:31:21.63
1 8 8
0 7 6
0 -8 -7
という3×3行列の対角化を行うという問題ですが、
まず固有値、固有ベクトルを求めるという手順を踏むとおもいます。
固有値は1,-1で1が縮退しています。
固有ベクトルをそれぞれ求めると直行しないのですが、
その際はどうすればよいでしょうか。
ちなみに固有値1の固有ベクトルを1/√2(0,-1,-1),1/√3(1,-1,1)
固有値-1のときの固有ベクトルを(4,3,-4)としました。
どこが間違っていますか?
278132人目の素数さん:2012/04/22(日) 01:36:32.61
wolframやらmaximaで確かめたの?
279132人目の素数さん:2012/04/22(日) 01:42:49.29
分かりません
280132人目の素数さん:2012/04/22(日) 02:36:12.15
>>277
どっちから掛けるのか分からないが、固有ベクトルになってないのでは?
281132人目の素数さん:2012/04/22(日) 02:43:16.74
>>280
3つのベクトルとも固有ベクトルにはなっています。
その後3つのベクトルを使い行列Uをつくって
U-1AUと対角化の計算を行なってもうまく対角化されません
282132人目の素数さん:2012/04/22(日) 02:45:06.86
>>281
> 1 8 8
> 0 7 6
> 0 -8 -7
> という3×3行列

>1/√2(0,-1,-1)
に掛けてみて
283132人目の素数さん:2012/04/22(日) 02:57:55.78
>>282
申し訳ありません。
1/√2(0,1,-1)ですね
284132人目の素数さん:2012/04/22(日) 03:11:15.33
最小値をX
最大値をY
とする場合のnの割合を出すやり方誰かおしえて。

例えば、

・最小値0、最大値10の時5は50%
・最小値0、最大値100の時10は10%
・最小値5、最大値10の時5は0%
・最小値5、最大値10の時6は20%

これ計算式にするとどうやんだ?
285132人目の素数さん:2012/04/22(日) 03:20:15.26
>>284
>・最小値5、最大値10の時6は20%
どうやって20%という答を出したの?
286132人目の素数さん:2012/04/22(日) 03:21:49.21
低レベルな質問ですが
(x^2−1)=(x-1)(x+1)ですよね?
(x^2+1)は何ですか?
287猫vs運営 ◆MuKUnGPXAY :2012/04/22(日) 03:29:00.70
>>286
ココは低レベルの人の集まりなので、その程度であれば大丈夫です。もっと
レベルが低い人達が沢山いますから。だから安心しましょう。


288132人目の素数さん:2012/04/22(日) 05:24:01.66
>>270
lim_(x→2) (1/(x^2−4)) ∫_(2,x) t^4 e^t dt
=lim_(x→2) (1/(x^2−4)) ∫_(2,x) t^4 e^2 dt
=e^2 lim_(x→2) (1/(x^2−4)) [x^5/5]_(2,x)
=e^2 lim_(x→2) (1/(x^2−4))(x^5−2^5)/5
=e^2 lim_(x→2) (1/(x+2))(x^4+2 x^3+2^2 x^2+2^3 x+2^4)/5
=e^2 (1/4) 2^4=4 e^2
289132人目の素数さん:2012/04/22(日) 06:04:33.28
          __ノ)-'´ ̄ ̄`ー- 、_
        , '´  _. -‐'''"二ニニ=-`ヽ、
      /   /:::::; -‐''"        `ーノ
     /   /:::::/           \
     /    /::::::/          | | |  |
     |   |:::::/ /     |  | | | |  |
      |   |::/ / / |  | ||  | | ,ハ .| ,ハ|
      |   |/ / / /| ,ハノ| /|ノレ,ニ|ル' 
     |   |  | / / レ',二、レ′ ,ィイ|゙/   私は只の数ヲタなんかとは付き合わないわ。
.     |   \ ∠イ  ,イイ|    ,`-' |      頭が良くて数学が出来てかっこいい人。それが必要条件よ。
     |     l^,人|  ` `-'     ゝ  |        さらに Ann.of Math に論文書けば十分条件にもなるわよ。
      |      ` -'\       ー'  人          一番嫌いなのは論文数を増やすためにくだらない論文を書いて
    |        /(l     __/  ヽ、           良い論文の出版を遅らせるお馬鹿な人。
     |       (:::::`‐-、__  |::::`、     ヒニニヽ、         あなたの論文が Ann of Math に accept される確率は?
    |      / `‐-、::::::::::`‐-、::::\   /,ニニ、\            それとも最近は Inv. Math. の方が上かしら?
   |      |::::::::::::::::::|` -、:::::::,ヘ ̄|'、  ヒニ二、 \
.   |      /::::::::::::::::::|::::::::\/:::O`、::\   | '、   \
   |      /:::::::::::::::::::/:::::::::::::::::::::::::::::'、::::\ノ  ヽ、  |
  |      |:::::/:::::::::/:::::::::::::::::::::::::::::::::::'、',::::'、  /:\__/‐、
  |      |/:::::::::::/::::::::::::::::::::::::::::::::::O::| '、::| く::::::::::::: ̄|
   |     /_..-'´ ̄`ー-、:::::::::::::::::::::::::::::::::::|/:/`‐'::\;;;;;;;_|
   |    |/::::::::::::::::::::::\:::::::::::::::::::::::::::::|::/::::|::::/:::::::::::/
    |   /:::::::::::::::::::::::::::::::::|:::::::::::::::::::::O::|::|::::::|:::::::::::::::/
290132人目の素数さん:2012/04/22(日) 07:05:40.01
>>286
x^2+1=(x-a)(x-b)と書けたとし、x=aを代入するとa^2+1=0とならねばならぬ。
ところがa^2は負でないからa^2+1=0が成り立つことはない。したがって初めにあるような分解は不可能。
291132人目の素数さん:2012/04/22(日) 08:10:47.81
新発見

∞は0だった!

∞÷1=0

∞=0×1

∞=0
292132人目の素数さん:2012/04/22(日) 08:37:13.05
>>286
x^2+1=(x+i)(x-i)

>>291
∞÷1=∞

1÷∞=0
1=0×∞=0 or ∞
293ようじょ ◆hNziS2E8421X :2012/04/22(日) 10:07:39.87
>>238お姉ちゃんいまお家にいない

>>236のわかるひといますか・・?
294猫vs運営 ◆MuKUnGPXAY :2012/04/22(日) 10:22:43.79
>>293
何回も出て来るなら、私が貴方(貴女)の撲滅を目指します。いい加減
にスル様に願います。私に冗談が通用しないのは知っての通りなので。


295あんでぃ ◆IUmn.OHxRw :2012/04/22(日) 10:29:14.35
まぁまぁ。

あんでぃ
296132人目の素数さん:2012/04/22(日) 10:30:53.81
>>293
杉浦みろ
297ようじょ ◆hNziS2E8421X :2012/04/22(日) 10:33:08.23
>>294
ねこさんおしえてよー

>>296
杉浦ってなに?
298132人目の素数さん:2012/04/22(日) 10:41:07.64
>>297
解析入門という聖書
299ようじょ ◆hNziS2E8421X :2012/04/22(日) 10:48:53.96
>>298
本かぁ・・お金ないからママに頼んでみる
ありがと!
300猫vs運営 ◆MuKUnGPXAY :2012/04/22(日) 10:52:52.25
>>297
>>299
叩きのめすヨ。


301132人目の素数さん:2012/04/22(日) 10:57:16.22
          __ノ)-'´ ̄ ̄`ー- 、_
        , '´  _. -‐'''"二ニニ=-`ヽ、
      /   /:::::; -‐''"        `ーノ
     /   /:::::/           \
     /    /::::::/          | | |  |
     |   |:::::/ /     |  | | | |  |
      |   |::/ / / |  | ||  | | ,ハ .| ,ハ|
      |   |/ / / /| ,ハノ| /|ノレ,ニ|ル' 
     |   |  | / / レ',二、レ′ ,ィイ|゙/   私は只の数ヲタなんかとは付き合わないわ。
.     |   \ ∠イ  ,イイ|    ,`-' |      頭が良くて数学が出来てかっこいい人。それが必要条件よ。
     |     l^,人|  ` `-'     ゝ  |        さらに Ann.of Math に論文書けば十分条件にもなるわよ。
      |      ` -'\       ー'  人          一番嫌いなのは論文数を増やすためにくだらない論文を書いて
    |        /(l     __/  ヽ、           良い論文の出版を遅らせるお馬鹿な人。
     |       (:::::`‐-、__  |::::`、     ヒニニヽ、         あなたの論文が Ann of Math に accept される確率は?
    |      / `‐-、::::::::::`‐-、::::\   /,ニニ、\            それとも最近は Inv. Math. の方が上かしら?
   |      |::::::::::::::::::|` -、:::::::,ヘ ̄|'、  ヒニ二、 \
.   |      /::::::::::::::::::|::::::::\/:::O`、::\   | '、   \
   |      /:::::::::::::::::::/:::::::::::::::::::::::::::::'、::::\ノ  ヽ、  |
  |      |:::::/:::::::::/:::::::::::::::::::::::::::::::::::'、',::::'、  /:\__/‐、
  |      |/:::::::::::/::::::::::::::::::::::::::::::::::O::| '、::| く::::::::::::: ̄|
   |     /_..-'´ ̄`ー-、:::::::::::::::::::::::::::::::::::|/:/`‐'::\;;;;;;;_|
   |    |/::::::::::::::::::::::\:::::::::::::::::::::::::::::|::/::::|::::/:::::::::::/
    |   /:::::::::::::::::::::::::::::::::|:::::::::::::::::::::O::|::|::::::|:::::::::::::::/
302132人目の素数さん:2012/04/22(日) 10:58:38.45
          __ノ)-'´ ̄ ̄`ー- 、_
        , '´  _. -‐'''"二ニニ=-`ヽ、
      /   /:::::; -‐''"        `ーノ
     /   /:::::/           \
     /    /::::::/          | | |  |
     |   |:::::/ /     |  | | | |  |
      |   |::/ / / |  | ||  | | ,ハ .| ,ハ|
      |   |/ / / /| ,ハノ| /|ノレ,ニ|ル' 
     |   |  | / / レ',二、レ′ ,ィイ|゙/   私は只の数ヲタなんかとは付き合わないわ。
.     |   \ ∠イ  ,イイ|    ,`-' |      頭が良くて数学が出来てかっこいい人。それが必要条件よ。
     |     l^,人|  ` `-'     ゝ  |        さらに Ann.of Math に論文書けば十分条件にもなるわよ。
      |      ` -'\       ー'  人          一番嫌いなのは論文数を増やすためにくだらない論文を書いて
    |        /(l     __/  ヽ、           良い論文の出版を遅らせるお馬鹿な人。
     |       (:::::`‐-、__  |::::`、     ヒニニヽ、         あなたの論文が Ann of Math に accept される確率は?
    |      / `‐-、::::::::::`‐-、::::\   /,ニニ、\            それとも最近は Inv. Math. の方が上かしら?
   |      |::::::::::::::::::|` -、:::::::,ヘ ̄|'、  ヒニ二、 \
.   |      /::::::::::::::::::|::::::::\/:::O`、::\   | '、   \
   |      /:::::::::::::::::::/:::::::::::::::::::::::::::::'、::::\ノ  ヽ、  |
  |      |:::::/:::::::::/:::::::::::::::::::::::::::::::::::'、',::::'、  /:\__/‐、
  |      |/:::::::::::/::::::::::::::::::::::::::::::::::O::| '、::| く::::::::::::: ̄|
   |     /_..-'´ ̄`ー-、:::::::::::::::::::::::::::::::::::|/:/`‐'::\;;;;;;;_|
   |    |/::::::::::::::::::::::\:::::::::::::::::::::::::::::|::/::::|::::/:::::::::::/
    |   /:::::::::::::::::::::::::::::::::|:::::::::::::::::::::O::|::|::::::|:::::::::::::::/
303132人目の素数さん:2012/04/22(日) 11:42:18.73
>>294
人生自体が冗談なお尻先生が言ってもなぁ
304猫vs運営 ◆MuKUnGPXAY :2012/04/22(日) 12:11:02.19
>>303
私は何度でも自分の考え方をきちんと主張するだけ。ソレをどう解釈する
のかは貴方の勝手。だからどうぞお好きに。私はこの場を焼き払う作業を
粛々と進めるだけなので。


305132人目の素数さん:2012/04/22(日) 12:35:21.34
>>304
焼き払うとか潰すとか
かなり前から言ってる気がするけど
一向に何の成果もないよね。
口先だけのオナニー増田は健在だな。
306猫vs運営 ◆MuKUnGPXAY :2012/04/22(日) 12:36:55.10
>>305
何の成果もなくても私は平気ですね。今後も作業を続行するだけなのでね。


307132人目の素数さん:2012/04/22(日) 12:38:35.75
>>306
そうだね。
作業(誰にも影響の無いオナニー)はこれからも毎日してね!!!
308132人目の素数さん:2012/04/22(日) 12:56:07.97
          __ノ)-'´ ̄ ̄`ー- 、_
        , '´  _. -‐'''"二ニニ=-`ヽ、
      /   /:::::; -‐''"        `ーノ
     /   /:::::/           \
     /    /::::::/          | | |  |
     |   |:::::/ /     |  | | | |  |
      |   |::/ / / |  | ||  | | ,ハ .| ,ハ|
      |   |/ / / /| ,ハノ| /|ノレ,ニ|ル' 
     |   |  | / / レ',二、レ′ ,ィイ|゙/   私は只の数ヲタなんかとは付き合わないわ。
.     |   \ ∠イ  ,イイ|    ,`-' |      頭が良くて数学が出来てかっこいい人。それが必要条件よ。
     |     l^,人|  ` `-'     ゝ  |        さらに Ann.of Math に論文書けば十分条件にもなるわよ。
      |      ` -'\       ー'  人          一番嫌いなのは論文数を増やすためにくだらない論文を書いて
    |        /(l     __/  ヽ、           良い論文の出版を遅らせるお馬鹿な人。
     |       (:::::`‐-、__  |::::`、     ヒニニヽ、         あなたの論文が Ann of Math に accept される確率は?
    |      / `‐-、::::::::::`‐-、::::\   /,ニニ、\            それとも最近は Inv. Math. の方が上かしら?
   |      |::::::::::::::::::|` -、:::::::,ヘ ̄|'、  ヒニ二、 \
.   |      /::::::::::::::::::|::::::::\/:::O`、::\   | '、   \
   |      /:::::::::::::::::::/:::::::::::::::::::::::::::::'、::::\ノ  ヽ、  |
  |      |:::::/:::::::::/:::::::::::::::::::::::::::::::::::'、',::::'、  /:\__/‐、
  |      |/:::::::::::/::::::::::::::::::::::::::::::::::O::| '、::| く::::::::::::: ̄|
   |     /_..-'´ ̄`ー-、:::::::::::::::::::::::::::::::::::|/:/`‐'::\;;;;;;;_|
   |    |/::::::::::::::::::::::\:::::::::::::::::::::::::::::|::/::::|::::/:::::::::::/
    |   /:::::::::::::::::::::::::::::::::|:::::::::::::::::::::O::|::|::::::|:::::::::::::::/
309132人目の素数さん:2012/04/22(日) 12:56:49.28
          __ノ)-'´ ̄ ̄`ー- 、_
        , '´  _. -‐'''"二ニニ=-`ヽ、
      /   /:::::; -‐''"        `ーノ
     /   /:::::/           \
     /    /::::::/          | | |  |
     |   |:::::/ /     |  | | | |  |
      |   |::/ / / |  | ||  | | ,ハ .| ,ハ|
      |   |/ / / /| ,ハノ| /|ノレ,ニ|ル' 
     |   |  | / / レ',二、レ′ ,ィイ|゙/   私は只の数ヲタなんかとは付き合わないわ。
.     |   \ ∠イ  ,イイ|    ,`-' |      頭が良くて数学が出来てかっこいい人。それが必要条件よ。
     |     l^,人|  ` `-'     ゝ  |        さらに Ann.of Math に論文書けば十分条件にもなるわよ。
      |      ` -'\       ー'  人          一番嫌いなのは論文数を増やすためにくだらない論文を書いて
    |        /(l     __/  ヽ、           良い論文の出版を遅らせるお馬鹿な人。
     |       (:::::`‐-、__  |::::`、     ヒニニヽ、         あなたの論文が Ann of Math に accept される確率は?
    |      / `‐-、::::::::::`‐-、::::\   /,ニニ、\            それとも最近は Inv. Math. の方が上かしら?
   |      |::::::::::::::::::|` -、:::::::,ヘ ̄|'、  ヒニ二、 \
.   |      /::::::::::::::::::|::::::::\/:::O`、::\   | '、   \
   |      /:::::::::::::::::::/:::::::::::::::::::::::::::::'、::::\ノ  ヽ、  |
  |      |:::::/:::::::::/:::::::::::::::::::::::::::::::::::'、',::::'、  /:\__/‐、
  |      |/:::::::::::/::::::::::::::::::::::::::::::::::O::| '、::| く::::::::::::: ̄|
   |     /_..-'´ ̄`ー-、:::::::::::::::::::::::::::::::::::|/:/`‐'::\;;;;;;;_|
   |    |/::::::::::::::::::::::\:::::::::::::::::::::::::::::|::/::::|::::/:::::::::::/
    |   /:::::::::::::::::::::::::::::::::|:::::::::::::::::::::O::|::|::::::|:::::::::::::::/
310132人目の素数さん:2012/04/22(日) 13:10:32.38
http://www.t-tsushin.net/picup/kyogaku/teikyo_daigaku/m_1.php

これの答えは正しいのですか?
311132人目の素数さん:2012/04/22(日) 13:14:47.86
>>310
どこが違うと思うのか書けよw
312132人目の素数さん:2012/04/22(日) 13:15:42.02
>>310
正しくない。
313132人目の素数さん:2012/04/22(日) 13:20:45.64
>>311
>>312
どもです

自分の答えと違いましたもので
314132人目の素数さん:2012/04/22(日) 13:38:19.83
下を導けという問題なんですがとっかかりがよくわかりません。どなたか助けて下さい。

 α+2p            {2p, n=0
∫   cos(nπt/p)dt = {
 α               {0, n=1,2,…
315241:2012/04/22(日) 14:32:13.97
さしあたって、自己解決
本に記載されている内容が、間違えてるぽい。

×…有限セル複体の間の任意の連続写像は階層を保つ写像にホモトピックである。
○…有限セル複体の間の任意の連続写像は階層を保つ写像にホモトピックなものが存在する。

で、疑問点は次のように理解。
0骨格への定値写像は、胞体写像であり、任意の定値写像とホモトピックである。

しかし、さしあたって、セル複体に関する部分については、
ずいぶん行間読まないと理解できなさそう。
316132人目の素数さん:2012/04/22(日) 14:34:39.52
お前の国語力がゼロなだけ
本のせいにするなアホ
317132人目の素数さん:2012/04/22(日) 14:35:30.38
>>314
∫[α,α+2p]dt=2p
318132人目の素数さん:2012/04/22(日) 14:43:08.23
数学の行間とは何ぞや
あなたはつかんだのでしょう?
教えてください
全てが記号とそれへの操作で完結するはず……だ
319132人目の素数さん:2012/04/22(日) 15:09:24.76
          __ノ)-'´ ̄ ̄`ー- 、_
        , '´  _. -‐'''"二ニニ=-`ヽ、
      /   /:::::; -‐''"        `ーノ
     /   /:::::/           \
     /    /::::::/          | | |  |
     |   |:::::/ /     |  | | | |  |
      |   |::/ / / |  | ||  | | ,ハ .| ,ハ|
      |   |/ / / /| ,ハノ| /|ノレ,ニ|ル' 
     |   |  | / / レ',二、レ′ ,ィイ|゙/   私は只の数ヲタなんかとは付き合わないわ。
.     |   \ ∠イ  ,イイ|    ,`-' |      頭が良くて数学が出来てかっこいい人。それが必要条件よ。
     |     l^,人|  ` `-'     ゝ  |        さらに Ann.of Math に論文書けば十分条件にもなるわよ。
      |      ` -'\       ー'  人          一番嫌いなのは論文数を増やすためにくだらない論文を書いて
    |        /(l     __/  ヽ、           良い論文の出版を遅らせるお馬鹿な人。
     |       (:::::`‐-、__  |::::`、     ヒニニヽ、         あなたの論文が Ann of Math に accept される確率は?
    |      / `‐-、::::::::::`‐-、::::\   /,ニニ、\            それとも最近は Inv. Math. の方が上かしら?
   |      |::::::::::::::::::|` -、:::::::,ヘ ̄|'、  ヒニ二、 \
.   |      /::::::::::::::::::|::::::::\/:::O`、::\   | '、   \
   |      /:::::::::::::::::::/:::::::::::::::::::::::::::::'、::::\ノ  ヽ、  |
  |      |:::::/:::::::::/:::::::::::::::::::::::::::::::::::'、',::::'、  /:\__/‐、
  |      |/:::::::::::/::::::::::::::::::::::::::::::::::O::| '、::| く::::::::::::: ̄|
   |     /_..-'´ ̄`ー-、:::::::::::::::::::::::::::::::::::|/:/`‐'::\;;;;;;;_|
   |    |/::::::::::::::::::::::\:::::::::::::::::::::::::::::|::/::::|::::/:::::::::::/
    |   /:::::::::::::::::::::::::::::::::|:::::::::::::::::::::O::|::|::::::|:::::::::::::::/
320132人目の素数さん:2012/04/22(日) 15:18:46.17
lim[x→a]f(x)=b, lim[y→b]g(y)=c ならば lim[x→a]g(f(x))=c は成り立つか?

成り立たないらしいので反例を出せばよいのですが思い付きません
よろしくお願いします
321132人目の素数さん:2012/04/22(日) 15:55:34.84
          __ノ)-'´ ̄ ̄`ー- 、_
        , '´  _. -‐'''"二ニニ=-`ヽ、
      /   /:::::; -‐''"        `ーノ
     /   /:::::/           \
     /    /::::::/          | | |  |
     |   |:::::/ /     |  | | | |  |
      |   |::/ / / |  | ||  | | ,ハ .| ,ハ|
      |   |/ / / /| ,ハノ| /|ノレ,ニ|ル' 
     |   |  | / / レ',二、レ′ ,ィイ|゙/   私は只の数ヲタなんかとは付き合わないわ。
.     |   \ ∠イ  ,イイ|    ,`-' |      頭が良くて数学が出来てかっこいい人。それが必要条件よ。
     |     l^,人|  ` `-'     ゝ  |        さらに Ann.of Math に論文書けば十分条件にもなるわよ。
      |      ` -'\       ー'  人          一番嫌いなのは論文数を増やすためにくだらない論文を書いて
    |        /(l     __/  ヽ、           良い論文の出版を遅らせるお馬鹿な人。
     |       (:::::`‐-、__  |::::`、     ヒニニヽ、         あなたの論文が Ann of Math に accept される確率は?
    |      / `‐-、::::::::::`‐-、::::\   /,ニニ、\            それとも最近は Inv. Math. の方が上かしら?
   |      |::::::::::::::::::|` -、:::::::,ヘ ̄|'、  ヒニ二、 \
.   |      /::::::::::::::::::|::::::::\/:::O`、::\   | '、   \
   |      /:::::::::::::::::::/:::::::::::::::::::::::::::::'、::::\ノ  ヽ、  |
  |      |:::::/:::::::::/:::::::::::::::::::::::::::::::::::'、',::::'、  /:\__/‐、
  |      |/:::::::::::/::::::::::::::::::::::::::::::::::O::| '、::| く::::::::::::: ̄|
   |     /_..-'´ ̄`ー-、:::::::::::::::::::::::::::::::::::|/:/`‐'::\;;;;;;;_|
   |    |/::::::::::::::::::::::\:::::::::::::::::::::::::::::|::/::::|::::/:::::::::::/
    |   /:::::::::::::::::::::::::::::::::|:::::::::::::::::::::O::|::|::::::|:::::::::::::::/
322132人目の素数さん:2012/04/22(日) 15:56:15.06
どうしてもわからないので質問させて下さい。

f(x)=f(a-x)のとき、
∫[0→a]f(x)dx = 2∫[0→a/2]f(x)dx
を証明せよ。

どなたか証明方法をわかる方、ご教授お願いいたします。
323132人目の素数さん:2012/04/22(日) 16:06:44.19
普通に考えればいいのでは?
324132人目の素数さん:2012/04/22(日) 16:34:43.08
g(x)=f(x-a/2) => g(x)=g(-x)

つまり、関数をa/2左にシフトすると、y軸を中心とした対称な関数で
積分の範囲が[-a/2,a/2]と[-a/2,0]となる
325132人目の素数さん:2012/04/22(日) 16:41:24.31
>>320 f(x)=0 (全ての x), g(x)=1 (x=0), 0 (x≠0).
326132人目の素数さん:2012/04/22(日) 16:51:56.68
>>325
すいません、説明してもらってもいいですか
327132人目の素数さん:2012/04/22(日) 17:20:42.31
>>317
何とかわかりそうです、ありがとうございました
328132人目の素数さん:2012/04/22(日) 17:44:57.76
>>326
そのままだろ
g(f(x)) → g(0) = 1 (x→0)
g(x) → 0 (x→0)
329132人目の素数さん:2012/04/22(日) 18:06:10.31
>>328
この問題は
lim[x→a]g(f(x))≠g(f(a))
となるようなものを設定すればいいということですかね?
330132人目の素数さん:2012/04/22(日) 18:37:40.86
教えて欲しいのですが
0001〜9999までで
0001なら0+0+0+1=1
1234なら1+2+3+4=10
とした場合、和で一番多い答えは幾つでしょうか。と言うのがあります。

目先一番大きい数字9999の36と一番小さい数字0001の1を足して2で割った
18もしくは19と思うのですが何か良い計算方法はないものでしょうか?
331132人目の素数さん:2012/04/22(日) 18:46:18.03
複素数体を特徴付ける条件として適当なものはありませんか?

例えば
実数体であれば「アルキメデス的かつコーシー完備な順序体は同型を除いて一通りである」
有理数体であれば「標数0の体であって、自分自身以外に部分体を持たないものは同型を除いて一通りである」

複素数体の場合にはどのような条件が知られているのでしょうか
332132人目の素数さん:2012/04/22(日) 18:48:13.35
0000〜9999なら、一桁でやって4倍
333132人目の素数さん:2012/04/22(日) 18:50:41.19
区画(0.1)に属する任意の有理数xに対して、xに収束する(0.1)に属する有理数列はどのように構成されますか…?
有理数は稠密だから存在するのはわかったのですが、具体的に構成する方法がわかりません
334132人目の素数さん:2012/04/22(日) 18:56:46.94
x+1/n
335132人目の素数さん:2012/04/22(日) 19:04:24.64
>>334
すみません有理数xではなく、無理数xについてでした…

無理数xに収束する有理数列の構成方法は具体的にどうなりますか?
336132人目の素数さん:2012/04/22(日) 19:19:32.29
>335
小数表示
床関数を使って近似列を構成
337132人目の素数さん:2012/04/22(日) 19:20:07.19
          __ノ)-'´ ̄ ̄`ー- 、_
        , '´  _. -‐'''"二ニニ=-`ヽ、
      /   /:::::; -‐''"        `ーノ
     /   /:::::/           \
     /    /::::::/          | | |  |
     |   |:::::/ /     |  | | | |  |
      |   |::/ / / |  | ||  | | ,ハ .| ,ハ|
      |   |/ / / /| ,ハノ| /|ノレ,ニ|ル' 
     |   |  | / / レ',二、レ′ ,ィイ|゙/   私は只の数ヲタなんかとは付き合わないわ。
.     |   \ ∠イ  ,イイ|    ,`-' |      頭が良くて数学が出来てかっこいい人。それが必要条件よ。
     |     l^,人|  ` `-'     ゝ  |        さらに Ann.of Math に論文書けば十分条件にもなるわよ。
      |      ` -'\       ー'  人          一番嫌いなのは論文数を増やすためにくだらない論文を書いて
    |        /(l     __/  ヽ、           良い論文の出版を遅らせるお馬鹿な人。
     |       (:::::`‐-、__  |::::`、     ヒニニヽ、         あなたの論文が Ann of Math に accept される確率は?
    |      / `‐-、::::::::::`‐-、::::\   /,ニニ、\            それとも最近は Inv. Math. の方が上かしら?
   |      |::::::::::::::::::|` -、:::::::,ヘ ̄|'、  ヒニ二、 \
.   |      /::::::::::::::::::|::::::::\/:::O`、::\   | '、   \
   |      /:::::::::::::::::::/:::::::::::::::::::::::::::::'、::::\ノ  ヽ、  |
  |      |:::::/:::::::::/:::::::::::::::::::::::::::::::::::'、',::::'、  /:\__/‐、
  |      |/:::::::::::/::::::::::::::::::::::::::::::::::O::| '、::| く::::::::::::: ̄|
   |     /_..-'´ ̄`ー-、:::::::::::::::::::::::::::::::::::|/:/`‐'::\;;;;;;;_|
   |    |/::::::::::::::::::::::\:::::::::::::::::::::::::::::|::/::::|::::/:::::::::::/
    |   /:::::::::::::::::::::::::::::::::|:::::::::::::::::::::O::|::|::::::|:::::::::::::::/
338132人目の素数さん:2012/04/22(日) 19:24:57.76
方程式2x/1+3y/1=12/1の自然数の解の内、x−yを最大にするxとyを求めよ。

お願いします
339132人目の素数さん:2012/04/22(日) 19:26:34.74
>>330
0000〜9999なら、

0と36が同じ確率あとは、なんちゃら分布だから、
その中心の18
340132人目の素数さん:2012/04/22(日) 19:29:28.82
>>338
すいません
1/2x+1/3y=1/12
でした。
341132人目の素数さん:2012/04/22(日) 19:34:24.01
          __ノ)-'´ ̄ ̄`ー- 、_
        , '´  _. -‐'''"二ニニ=-`ヽ、
      /   /:::::; -‐''"        `ーノ
     /   /:::::/           \
     /    /::::::/          | | |  |
     |   |:::::/ /     |  | | | |  |
      |   |::/ / / |  | ||  | | ,ハ .| ,ハ|
      |   |/ / / /| ,ハノ| /|ノレ,ニ|ル' 
     |   |  | / / レ',二、レ′ ,ィイ|゙/   私は只の数ヲタなんかとは付き合わないわ。
.     |   \ ∠イ  ,イイ|    ,`-' |      頭が良くて数学が出来てかっこいい人。それが必要条件よ。
     |     l^,人|  ` `-'     ゝ  |        さらに Ann.of Math に論文書けば十分条件にもなるわよ。
      |      ` -'\       ー'  人          一番嫌いなのは論文数を増やすためにくだらない論文を書いて
    |        /(l     __/  ヽ、           良い論文の出版を遅らせるお馬鹿な人。
     |       (:::::`‐-、__  |::::`、     ヒニニヽ、         あなたの論文が Ann of Math に accept される確率は?
    |      / `‐-、::::::::::`‐-、::::\   /,ニニ、\            それとも最近は Inv. Math. の方が上かしら?
   |      |::::::::::::::::::|` -、:::::::,ヘ ̄|'、  ヒニ二、 \
.   |      /::::::::::::::::::|::::::::\/:::O`、::\   | '、   \
   |      /:::::::::::::::::::/:::::::::::::::::::::::::::::'、::::\ノ  ヽ、  |
  |      |:::::/:::::::::/:::::::::::::::::::::::::::::::::::'、',::::'、  /:\__/‐、
  |      |/:::::::::::/::::::::::::::::::::::::::::::::::O::| '、::| く::::::::::::: ̄|
   |     /_..-'´ ̄`ー-、:::::::::::::::::::::::::::::::::::|/:/`‐'::\;;;;;;;_|
   |    |/::::::::::::::::::::::\:::::::::::::::::::::::::::::|::/::::|::::/:::::::::::/
    |   /:::::::::::::::::::::::::::::::::|:::::::::::::::::::::O::|::|::::::|:::::::::::::::/
342132人目の素数さん:2012/04/22(日) 19:36:25.69
          __ノ)-'´ ̄ ̄`ー- 、_
        , '´  _. -‐'''"二ニニ=-`ヽ、
      /   /:::::; -‐''"        `ーノ
     /   /:::::/           \
     /    /::::::/          | | |  |
     |   |:::::/ /     |  | | | |  |
      |   |::/ / / |  | ||  | | ,ハ .| ,ハ|
      |   |/ / / /| ,ハノ| /|ノレ,ニ|ル' 
     |   |  | / / レ',二、レ′ ,ィイ|゙/   私は只の数ヲタなんかとは付き合わないわ。
.     |   \ ∠イ  ,イイ|    ,`-' |      頭が良くて数学が出来てかっこいい人。それが必要条件よ。
     |     l^,人|  ` `-'     ゝ  |        さらに Ann.of Math に論文書けば十分条件にもなるわよ。
      |      ` -'\       ー'  人          一番嫌いなのは論文数を増やすためにくだらない論文を書いて
    |        /(l     __/  ヽ、           良い論文の出版を遅らせるお馬鹿な人。
     |       (:::::`‐-、__  |::::`、     ヒニニヽ、         あなたの論文が Ann of Math に accept される確率は?
    |      / `‐-、::::::::::`‐-、::::\   /,ニニ、\            それとも最近は Inv. Math. の方が上かしら?
   |      |::::::::::::::::::|` -、:::::::,ヘ ̄|'、  ヒニ二、 \
.   |      /::::::::::::::::::|::::::::\/:::O`、::\   | '、   \
   |      /:::::::::::::::::::/:::::::::::::::::::::::::::::'、::::\ノ  ヽ、  |
  |      |:::::/:::::::::/:::::::::::::::::::::::::::::::::::'、',::::'、  /:\__/‐、
  |      |/:::::::::::/::::::::::::::::::::::::::::::::::O::| '、::| く::::::::::::: ̄|
   |     /_..-'´ ̄`ー-、:::::::::::::::::::::::::::::::::::|/:/`‐'::\;;;;;;;_|
   |    |/::::::::::::::::::::::\:::::::::::::::::::::::::::::|::/::::|::::/:::::::::::/
    |   /:::::::::::::::::::::::::::::::::|:::::::::::::::::::::O::|::|::::::|:::::::::::::::/
343132人目の素数さん:2012/04/22(日) 20:29:18.32
>>275
そういうことだったんですか!?ありがとうございます
344132人目の素数さん:2012/04/22(日) 21:06:24.65
>>340
x=y+aとおく。つまりx-y=a だからもとめる答えはaになる。
(めんどくさいので、いい加減な仮定aは正の整数と勝手に仮定する。)

式にx=y+aを代入して、yの式とする。
すると、y(y-10)=a(4-y)となる。

y>=10 右辺 正(0を含む) 左辺 負(0を含まない)
y<=4  右辺 負(0を含まない) 左辺 正(0を含む)
(aが正の整数と仮定されている)

となり、条件を満たさない。つまり、y={5,6,7,8,9}
あとは、上記yを1つづつ代入してaを求めると。a=25が最大。
a=25は正なので、上記仮定も満たしている。

あとは、本当か確かめて。
345132人目の素数さん:2012/04/22(日) 21:28:53.28
おおスマートだ
式をxでまとめてx-yに代入、
「y(y-10)/(4-y)」の最大値をどうやって求めるのかで固まってたわ
346132人目の素数さん:2012/04/22(日) 21:30:27.12
xy-4x-6y=0
(x-6)(y-4)=24
x-6=±(1,2,3,4,6,8,12,24) x≠6
y-4=±(24,12,8,6,4,3,2,1) y≠4
∴max(x-y)=25 (x-6=24,y-2=1)
347132人目の素数さん:2012/04/22(日) 21:33:51.86
追加
x>0, x≠6
y>0, y≠4
348132人目の素数さん:2012/04/22(日) 21:50:47.03
nを自然数とする。
袋の中に(n^2+1)個の赤玉と1個の白玉が入っている。
この中から玉を1個取り出し色を確認して袋の中に戻すという試行を(n+1)回繰り返す。
このとき、少なくとも1回白玉を取り出す確率をp(n)とすると
limp(n)=0を示せ。
n→∞


うーん分からないです。
349132人目の素数さん:2012/04/22(日) 21:55:23.39
limp(n)=1
n→∞

でしょ?
350132人目の素数さん:2012/04/22(日) 22:10:59.71
p(n)=1-((n^2+1)/(n^2+2))^(n+1)
351132人目の素数さん:2012/04/22(日) 22:13:05.73
IをRの区間、f:I →R^n , g:I→R^nとする。

1.fとgがともに微分可能であるならば、
d/dt(f(t),g(t))=(f^t(t),g(t))+(f(t),g^t(t)) (t∈I)
が成り立つことを示せ。

2.質点が等速運動するならば(つまり時刻tにおける位置をf(t)と表すとき、||f^t(t)||が定数関数となる)、速度と加速度はつねに直交することを示せ。
352132人目の素数さん:2012/04/22(日) 22:17:38.97
>>346 訂正
x-6=±(1,2,3,4,6,8,12,24) x>0
y-4=±(24,12,8,6,4,3,2,1) y>0
353132人目の素数さん:2012/04/22(日) 22:25:15.59
>>351
d/dt(f(t),g(t))=((df/dt)(t)+(f(t),(dg/dt)(t))
354132人目の素数さん:2012/04/22(日) 22:33:04.70
1 名無しさんにズームイン! [] Date:2012/03/28(水) 08:28:15.02 ?ID:NWYs/2ZP Be:
 やらなけゃいけない
 電○の各局への圧力が半端ないんです
 昨日、一昨日前田AKB卒業ネタやった情報番組全てが前田AKB卒業ネタ中の毎分視聴率ダダ下がりしました。
 各局本音では毎分視聴率ダダ下がりするこのネタははやりたくなかったけど原子力村以上に電○からの圧力が凄いんです

なんか海外でも日本でAKBがバカ売れしてるっていう情報に騙されてAKBヲタになった人が最近気付き始めてる件
http://hayabusa3.2ch.net/test/read.cgi/morningcoffee/1334516585/

税金垂れ流しのヤラセブーム
355132人目の素数さん:2012/04/22(日) 22:34:43.24
          __ノ)-'´ ̄ ̄`ー- 、_
        , '´  _. -‐'''"二ニニ=-`ヽ、
      /   /:::::; -‐''"        `ーノ
     /   /:::::/           \
     /    /::::::/          | | |  |
     |   |:::::/ /     |  | | | |  |
      |   |::/ / / |  | ||  | | ,ハ .| ,ハ|
      |   |/ / / /| ,ハノ| /|ノレ,ニ|ル' 
     |   |  | / / レ',二、レ′ ,ィイ|゙/   私は只の数ヲタなんかとは付き合わないわ。
.     |   \ ∠イ  ,イイ|    ,`-' |      頭が良くて数学が出来てかっこいい人。それが必要条件よ。
     |     l^,人|  ` `-'     ゝ  |        さらに Ann.of Math に論文書けば十分条件にもなるわよ。
      |      ` -'\       ー'  人          一番嫌いなのは論文数を増やすためにくだらない論文を書いて
    |        /(l     __/  ヽ、           良い論文の出版を遅らせるお馬鹿な人。
     |       (:::::`‐-、__  |::::`、     ヒニニヽ、         あなたの論文が Ann of Math に accept される確率は?
    |      / `‐-、::::::::::`‐-、::::\   /,ニニ、\            それとも最近は Inv. Math. の方が上かしら?
   |      |::::::::::::::::::|` -、:::::::,ヘ ̄|'、  ヒニ二、 \
.   |      /::::::::::::::::::|::::::::\/:::O`、::\   | '、   \
   |      /:::::::::::::::::::/:::::::::::::::::::::::::::::'、::::\ノ  ヽ、  |
  |      |:::::/:::::::::/:::::::::::::::::::::::::::::::::::'、',::::'、  /:\__/‐、
  |      |/:::::::::::/::::::::::::::::::::::::::::::::::O::| '、::| く::::::::::::: ̄|
   |     /_..-'´ ̄`ー-、:::::::::::::::::::::::::::::::::::|/:/`‐'::\;;;;;;;_|
   |    |/::::::::::::::::::::::\:::::::::::::::::::::::::::::|::/::::|::::/:::::::::::/
    |   /:::::::::::::::::::::::::::::::::|:::::::::::::::::::::O::|::|::::::|:::::::::::::::/
356132人目の素数さん:2012/04/22(日) 22:39:54.88
xy平面上を動く点P(a^2+3a,2^a)がある。
原点をOとし、線分OPの長さの最小値を求めよ。
357132人目の素数さん:2012/04/22(日) 22:44:44.81
すみません、ベクトルが抜けていました。

IをRの区間、f(ベクトル):I →R^n , g(ベクトル):I→R^nとする。

1.f(ベクトル)とg(ベクトル)がともに微分可能であるならば、
d/dt(f(ベクトル)(t),g(ベクトル)(t))=(f^t(ベクトル)(t),g(ベクトル)(t))+(f(ベクトル)(t),g^t(ベクトル)(t)) (t∈I)
が成り立つことを示せ。

2.質点が等速運動するならば(つまり時刻tにおける位置をf(ベクトル)(t)と表すとき、||f(ベクトル)^t(t)||が定数関数となる)、速度と加速度はつねに直交することを示せ。
358132人目の素数さん:2012/04/23(月) 04:52:49.39
x^2+bx+c=0

のbは-(α+β)
cはαβになる証明をしてください
359132人目の素数さん:2012/04/23(月) 05:05:57.32
解をα,βとすると、x=α,βの時0になる
→(x-α)(x-β)=0
と書ける。

これを展開すると
x^2-αx-βx+αβ=0
=x^2-(α+β)x+αβ
360132人目の素数さん:2012/04/23(月) 08:10:59.23
          __ノ)-'´ ̄ ̄`ー- 、_
        , '´  _. -‐'''"二ニニ=-`ヽ、
      /   /:::::; -‐''"        `ーノ
     /   /:::::/           \
     /    /::::::/          | | |  |
     |   |:::::/ /     |  | | | |  |
      |   |::/ / / |  | ||  | | ,ハ .| ,ハ|
      |   |/ / / /| ,ハノ| /|ノレ,ニ|ル' 
     |   |  | / / レ',二、レ′ ,ィイ|゙/   私は只の数ヲタなんかとは付き合わないわ。
.     |   \ ∠イ  ,イイ|    ,`-' |      頭が良くて数学が出来てかっこいい人。それが必要条件よ。
     |     l^,人|  ` `-'     ゝ  |        さらに Ann.of Math に論文書けば十分条件にもなるわよ。
      |      ` -'\       ー'  人          一番嫌いなのは論文数を増やすためにくだらない論文を書いて
    |        /(l     __/  ヽ、           良い論文の出版を遅らせるお馬鹿な人。
     |       (:::::`‐-、__  |::::`、     ヒニニヽ、         あなたの論文が Ann of Math に accept される確率は?
    |      / `‐-、::::::::::`‐-、::::\   /,ニニ、\            それとも最近は Inv. Math. の方が上かしら?
   |      |::::::::::::::::::|` -、:::::::,ヘ ̄|'、  ヒニ二、 \
.   |      /::::::::::::::::::|::::::::\/:::O`、::\   | '、   \
   |      /:::::::::::::::::::/:::::::::::::::::::::::::::::'、::::\ノ  ヽ、  |
  |      |:::::/:::::::::/:::::::::::::::::::::::::::::::::::'、',::::'、  /:\__/‐、
  |      |/:::::::::::/::::::::::::::::::::::::::::::::::O::| '、::| く::::::::::::: ̄|
   |     /_..-'´ ̄`ー-、:::::::::::::::::::::::::::::::::::|/:/`‐'::\;;;;;;;_|
   |    |/::::::::::::::::::::::\:::::::::::::::::::::::::::::|::/::::|::::/:::::::::::/
    |   /:::::::::::::::::::::::::::::::::|:::::::::::::::::::::O::|::|::::::|:::::::::::::::/
361132人目の素数さん:2012/04/23(月) 09:21:11.37
>>351
1.微分の定義どおり書き下し、内積の線形性を使えばよい。
d/dt(f(t),g(t))
= lim[h→0] {(f(t+h),g(t+h)) - (f(t),g(t))}/h
= lim[h→0] {(f(t+h),g(t+h)) - (f(t+h),g(t)) + (f(t+h),g(t)) - (f(t),g(t))}/h
= lim[h→0] {(f(t+h),g(t+h)-g(t)) + (f(t+h)-f(t),g(t))}/h
= lim[h→0] {(f(t+h),(g(t+h)-g(t))/h) + ((f(t+h)-f(t))/h,g(t))}
=(f(t),g’(t)) + (f’(t),g(t))

2.||f’(t)||^2 = (f’(t),f’(t)) = const. を微分すればよい。
d/dt(f’(t),f’(t))
= (f’(t),f”(t)) + (f”(t),f’(t))
= 2 (f’(t),f”(t))
= 0
したがって速度f’(t)と加速度f”(t)はつねに直交する。
362132人目の素数さん:2012/04/23(月) 10:24:05.67
S={X| Xは複素成分2次正方で Xのトレースは0}
X Y∈S に対して
[X Y]=XY-YX
とするとき、Sは行列の加法 スカラー倍 [ ]によってリー環になりますが、このリー環が単純なリー環である理由を教えて下さい…
363132人目の素数さん:2012/04/23(月) 10:46:47.82
理由なんて無い
なぜかは知らないけど、それが定義を満たすから、それはそうだとしか言えない

全ての X, Y ∈ S が
単純なリー環の定義を満たすことを
確認すればいい
364132人目の素数さん:2012/04/23(月) 11:00:32.22
ちょっとレベル高い問題ばっかりで恐縮だけど……
箱の中に1番から31番までの数字が書かれたくじが入ってるとして、
1つ引いてまた箱の中に戻す、を120回繰り返した場合、
1番から7番以外のくじを最低1回は引ける確率ってどのくらい……?
1番から7番までのくじを引いてもかまわないものとして
365132人目の素数さん:2012/04/23(月) 11:22:50.58
1-(7/31)^120
366132人目の素数さん:2012/04/23(月) 11:28:10.17
>>365
なるほど、ありがとう!
367馬鹿:2012/04/23(月) 11:59:21.10
線型微分方程式がわかりません
368132人目の素数さん:2012/04/23(月) 15:09:00.55
>全ての X, Y ∈ S が
>単純なリー環の定義を満たすことを
>確認すればいい

何言ってだこいつ
369132人目の素数さん:2012/04/23(月) 15:10:22.11
と、解けないksがほざいておりますwww
370132人目の素数さん:2012/04/23(月) 15:51:03.80
そうだそうだ
>>363のような解けないksは消えろ
371132人目の素数さん:2012/04/23(月) 15:53:27.38
ンじゃおまえが解いてやれよwww
アホちゃんカスちゃんクズちゃnwwww
無能低能がそろい踏みだなオイオイオイオイw
その程度も解けないゴミカスは失せろよwwwwwww
372132人目の素数さん:2012/04/23(月) 15:53:28.66
スレの目的に従って、わからないことを書いただけのひとを
質問の仕方が悪いからって責めないであげて!
373132人目の素数さん:2012/04/23(月) 15:54:12.79
>>335
連分数展開とかいろいろある
374132人目の素数さん:2012/04/23(月) 15:57:07.03
質問の仕方が悪いって誰のことだよ
375132人目の素数さん:2012/04/23(月) 15:57:41.24
このスレに貼り付いて監視してる気持ち悪い奴がいるね
アーヤダヤダ
376132人目の素数さん:2012/04/23(月) 16:00:50.46
>>375
自己紹介乙wwwwww
377132人目の素数さん:2012/04/23(月) 16:03:09.13
378132人目の素数さん:2012/04/23(月) 16:10:08.47
>>374
たぶん>>368のことだろ
379132人目の素数さん:2012/04/23(月) 16:33:17.66
>>331
うーん、たとえば、
標数が0でQ上の超越次数が連続体濃度の代数的閉体はCと同型
(これは代数的閉包の一意性から簡単にわかる)
380132人目の素数さん:2012/04/23(月) 16:36:24.91
単純リー環の定義ぐらいググりゃすぐ分かるだろうに
>全ての X, Y ∈ S が
なんて書いてる>>363は馬鹿
381132人目の素数さん:2012/04/23(月) 16:37:55.10
ンじゃあテメーがさっさと教えてやれよ
しかもそれもしないんだろwww
テイノーちゃんは黙ってろよks
プ
382132人目の素数さん:2012/04/23(月) 16:47:50.73
哲也のウンコ踏め!
383132人目の素数さん:2012/04/23(月) 16:49:48.43
ふみみました
これで私も数学の甜☆菜になれますか?
384132人目の素数さん:2012/04/23(月) 17:28:41.41
「関数y=x^2」が最大値を持たないことを、背理法によって証明せよ
385132人目の素数さん:2012/04/23(月) 17:31:50.43
最大値Mがあるとかていする
ここでヨシキよりM^2がそんざいし
M^2>Mである
しあたがって最大値をもtない
386132人目の素数さん:2012/04/23(月) 17:47:30.41
>>384
定義域による
387132人目の素数さん:2012/04/23(月) 17:52:24.11
R∪{∞}をもってきてみる
388132人目の素数さん:2012/04/23(月) 18:24:15.68
平面において、点A(a↑)を中心とした半径rの円をCとし、
点P0(p0↑)はこの円周上の点でないとする。
点P0を通り単位方向ベクトルe↑の直線lがこの円Cと共有点P1,P2を持つとする。
ただし、点P0(p0↑)が円Cの外部の点のときはP1=P2の場合(直線lが円Cに接する場合)
も許すこととする。P0P1↑=t1*e↑、P0P2↑=t2*e↑とおく。
このとき「t1・t2=|p0↑−a↑|^2−r^2」であることを示しなさい。

よろしくお願いします。


389132人目の素数さん:2012/04/23(月) 18:29:15.64
http://beebee2see.appspot.com/i/azuYytqkBgw.jpg

よろしくお願いします
390132人目の素数さん:2012/04/23(月) 18:49:25.68
たぶん
・微分積分学の基本定理f={∫f}'を使う
・部分積分の公式∫fg=Fg-∫Fg'を使う
・置換積分を使う
・努力の跡を見せる
391132人目の素数さん:2012/04/23(月) 18:54:33.73
三次元直交座標系xyzで 平面S:ax + by + cz + d = 0 上の点 O(x0,y0,z0)
と平面S上でないの点 P(x1,y1,z1)を通る直線OPと平面Sがなす角θを求めよ。

ベクトル使うのかとか考えたんですがわかりませんでした。
よろしくお願いします
392132人目の素数さん:2012/04/23(月) 18:56:48.25
×考えた
○面倒臭そうだから誰か解いてくれ
393132人目の素数さん:2012/04/23(月) 19:36:15.29
平面Sの法線ベクトルをn↑、直線OPの方向ベクトルをm↑、求める角度をθとすると
n↑・m↑=|n||m|cos(π/2-θ)
394あのこうちやんは始皇帝だった:2012/04/23(月) 19:38:54.32

 まったく、最近のクソガキどもは、3次元解析幾何学もまともに知らないのか!

 それじゃあ、5球座標の話をしても無駄だな!
395132人目の素数さん:2012/04/23(月) 19:39:13.95
          __ノ)-'´ ̄ ̄`ー- 、_
        , '´  _. -‐'''"二ニニ=-`ヽ、
      /   /:::::; -‐''"        `ーノ
     /   /:::::/           \
     /    /::::::/          | | |  |
     |   |:::::/ /     |  | | | |  |
      |   |::/ / / |  | ||  | | ,ハ .| ,ハ|
      |   |/ / / /| ,ハノ| /|ノレ,ニ|ル' 
     |   |  | / / レ',二、レ′ ,ィイ|゙/   私は只の数ヲタなんかとは付き合わないわ。
.     |   \ ∠イ  ,イイ|    ,`-' |      頭が良くて数学が出来てかっこいい人。それが必要条件よ。
     |     l^,人|  ` `-'     ゝ  |        さらに Ann.of Math に論文書けば十分条件にもなるわよ。
      |      ` -'\       ー'  人          一番嫌いなのは論文数を増やすためにくだらない論文を書いて
    |        /(l     __/  ヽ、           良い論文の出版を遅らせるお馬鹿な人。
     |       (:::::`‐-、__  |::::`、     ヒニニヽ、         あなたの論文が Ann of Math に accept される確率は?
    |      / `‐-、::::::::::`‐-、::::\   /,ニニ、\            それとも最近は Inv. Math. の方が上かしら?
   |      |::::::::::::::::::|` -、:::::::,ヘ ̄|'、  ヒニ二、 \
.   |      /::::::::::::::::::|::::::::\/:::O`、::\   | '、   \
   |      /:::::::::::::::::::/:::::::::::::::::::::::::::::'、::::\ノ  ヽ、  |
  |      |:::::/:::::::::/:::::::::::::::::::::::::::::::::::'、',::::'、  /:\__/‐、
  |      |/:::::::::::/::::::::::::::::::::::::::::::::::O::| '、::| く::::::::::::: ̄|
   |     /_..-'´ ̄`ー-、:::::::::::::::::::::::::::::::::::|/:/`‐'::\;;;;;;;_|
   |    |/::::::::::::::::::::::\:::::::::::::::::::::::::::::|::/::::|::::/:::::::::::/
    |   /:::::::::::::::::::::::::::::::::|:::::::::::::::::::::O::|::|::::::|:::::::::::::::/
396132人目の素数さん:2012/04/23(月) 19:54:14.38
>>393
点PとSを面と点の距離の公式で求めて(l1)、
点Oと点Pの距離を求めて(l2)
sinθ=l1/l2でもオッケーですか?
397あのこうちやんは始皇帝だった:2012/04/23(月) 20:05:22.86
>>396

 公式・・・・・・・を導出するのだ!!!!!!!!!!!!!!!!!!

 コレが出来ないヤツは、ゴミ・クズ・カスのクソガキ!!!!!!!!!!!!!!!!!
398132人目の素数さん:2012/04/23(月) 20:11:51.03

やっぱり
×考えた
○面倒臭そうだから誰か解いてくれ
だろwwwwwww
399132人目の素数さん:2012/04/23(月) 20:25:48.06
Π(n=1〜∞)(1+(1/n))(1-(1/(1+n)))
は収束しますか?
400132人目の素数さん:2012/04/23(月) 20:39:11.51
>>399
(1+(1/n))(1-(1/(1+n))) を計算してみ
401132人目の素数さん:2012/04/23(月) 20:50:55.60
>>400
1+(1/n)-(1/(1+n))-1/(n(n+1))

Σ((1/n)-(1/(1+n))-1/(n(n+1)))は絶対収束するから、>>399の無限積は収束する

こうですか…?
402132人目の素数さん:2012/04/23(月) 20:58:12.36
>>401
アホなこと言ってないで計算しろよ
403132人目の素数さん:2012/04/23(月) 20:58:36.49
>>401
> 1+(1/n)-(1/(1+n))-1/(n(n+1))
通分してみ
404132人目の素数さん:2012/04/23(月) 21:10:42.74
>>389
f(x)-∫[1,x]t*f'(t)dt
f(1)=-1/6-1/12+a+2=a+7/4
与式の両辺を微分すると
f'(x)-x*f'(x)=-x^2/2-x/6+a
(1-x)*f'(x)=-x^2/2-x/6+a
x=1のとき右辺が0になることから
-1/2-1/6+a=0 ∴a=2/3, f(1)=29/12
(1-x)*f'(x)=-x^2/2-x/6+2/3
=-(3*x+4)(x-1)/6
f'(x)=x/2+2/3
f(x)=x^2/4+2*x/3+C
f(1)=1/4+2/3+C=29/12からC=3/2
f(x)=x^2/4+2*x/3+3/2
405132人目の素数さん:2012/04/23(月) 21:18:13.15
Kを体とする。
ZからKへの自然な環準同型を
f(1)=1(Kの単位元)
となるように定めるとf(Z)は整域になる。

のはなぜですか?
406132人目の素数さん:2012/04/23(月) 21:20:25.20
マクローリン展開の式にx=iθを代入しても条件を満たすことをベキ級数の収束半径を使用して証明せよ。

物理で使うのですが、まずマクローリン(orテイラー)展開って実数以外を代入してよかったのかと、
ベキ級数がなぜ円になるのかよくわからないので詰んでます。
407132人目の素数さん:2012/04/23(月) 21:23:55.06
物理的に明らかだからだろ
408132人目の素数さん:2012/04/23(月) 21:24:30.58
>>406
アーベルの定理
409132人目の素数さん:2012/04/23(月) 21:44:52.80
直線x+2y=0に関して、点A(3.ー4)と対称な点Bを求めよ。
これ誰か教えてください。
410132人目の素数さん:2012/04/23(月) 21:50:34.60
>>408
ぐぐってみました。楕円関数論についての定理でしょうか?
大学数学だとどの分野にありますか?
411132人目の素数さん:2012/04/23(月) 22:11:01.97
{x} ∈ {{x},{y},{z}}
{x} ⊂ {{x},{y},{z}}

どちらが正しいのですか?

また、
a ∈ {a,b}
a ⊂ {a,b}
この場合、aは要素で、下のほうの記号(⊂)は集合について使うので要素であるaに用いているので間違い
という認識であってますか?
412132人目の素数さん:2012/04/23(月) 22:18:15.33

{x} ⊂ {{x},{y},{z}}
a ∈ {a,b}
413132人目の素数さん:2012/04/23(月) 22:19:14.52
>>412

{x} ∈ {{x},{y},{z}}
{{x}} ⊂ {{x},{y},{z}}
414132人目の素数さん:2012/04/23(月) 22:25:08.72
>>413

{x} ⊂ {{x},{y},{z}}
{{x}} ⊂ {{x},{y},{z}}
415132人目の素数さん:2012/04/23(月) 22:32:55.78
>>412,414
> {x} ⊂ {{x},{y},{z}}
集合{{x},{y},{z}}の要素は{x},{y},{z} の3つ
集合{x}の要素はxの1つ
{x}の要素であるxを{{x},{y},{z}}は要素に持たない(x≠{x})から
{x}は{{x},{y},{z}}の部分集合ではない
416132人目の素数さん:2012/04/23(月) 22:36:53.13
>>410
当然、べき級数に関する定理です
これは数学以前の、読解能力・情報処理能力の問題です
こんなことも分からないのであれば、あなたには学問などまだ早いです
417132人目の素数さん:2012/04/23(月) 22:39:48.33
>>416
つっこんでいいかな?
418132人目の素数さん:2012/04/23(月) 22:41:27.10
もっと分かりやすく言ってやれよwww
「ンなこともシラネ―のかよw 失せろアホwww」
とかダイレクトに言わないと分かんないだろさwwwwww
419132人目の素数さん:2012/04/23(月) 22:59:46.65
>>416
参考書のベキ級数の該当ページに、アーベルの定理について記述がありませんでした。
ぐぐって調べてみたところ、群理論と楕円関数論が主要な研究らしいので、
べき級数の図形的な意味に関係した楕円関数論の定理かと思いました。
420:2012/04/23(月) 23:03:08.65
>>406
めんどくせーな、解析接続できるつーことだよ
421132人目の素数さ:2012/04/23(月) 23:16:55.53
>>419
ベキ級数 収束半径 アーベルの定理 でぐぐれ
422132人目の素数さん:2012/04/23(月) 23:31:28.56
>>420
>>421
了解しました。ありがとうございました。
423132人目の素数さん:2012/04/23(月) 23:37:23.15
>>412-415
ややこしくてすみません・・・
認識が合っているようでよかったです

{x} ∈ {{x},{y},{z}}
こっちが正解、ということでいいのでしょうか
{{x},{y},{z}}の要素は{x},{y},{z}であり、x,y,zではないということですね?
424132人目の素数さん:2012/04/23(月) 23:57:12.81
425132人目の素数さん:2012/04/24(火) 00:04:55.53
∫_v[x×(rotM)]dv = ∫_S [n(x・M)-(n・x)M]dS + 2∫_v [M]dv
x:位置ベクトル
M:任意のベクトル
n:Sの単位法ベクトル
S=∂v

という式が成り立つそうですが、どうすれば証明できるかわかりません。
証明はめんどうだと思うので、知ってる方は参考文献などを教えてもらえませんか?
426425:2012/04/24(火) 00:06:46.19
分野は電磁気です。
427132人目の素数さん:2012/04/24(火) 00:26:29.21
ストローは麦の定理
428132人目の素数さん:2012/04/24(火) 00:43:59.16
ストローハムギの定理ですか?
見当たらないのですが。
429132人目の素数さん:2012/04/24(火) 00:49:23.29
>>409
点対称だから傾きは一緒。どこを通るか調べてその座標(a,b)を
(x-a)+2(y-b)=0
に代入すれば出る。図を描くと早い。



なんか公式があった気がするけど、
これでも簡単に出るからとりあえずこれでやってみな
430132人目の素数さん:2012/04/24(火) 01:10:11.37
分からない問題というか、厳密にはどうななんでしょうということなんですが
例えばy=x^2という関数があるとして、
x=0のとき、y=0で、零点ってやつなんだけど、
この零点って他の点とちがって連続せずに孤立しているんですか?

実数・関数の連続性から、-∞->0と+∞->0で単調増加・減少および有界を調べてみると
0のところは両側からきて同じだからくっつけてしまっても理論破綻起こさないから
そうしているんでしょうか?
431132人目の素数さん:2012/04/24(火) 01:19:17.96
孤立してる零点と
孤立していない零点の 二つがある

孤立している点を 孤立点 と呼ぶ
432132人目の素数さん:2012/04/24(火) 01:28:56.74
>>431
イメージが沸かないので、
孤立している零点を持つ関数と
孤立していないあ零点を持つ関数の
具体例が知りたいです
433132人目の素数さん:2012/04/24(火) 01:41:02.55
なんの話してるのかしらんけど

f(x)=(x-a)(x-b)
f(x,y)=x-y
434132人目の素数さん:2012/04/24(火) 02:36:00.42
すいません知恵を貸して下さい。

Q.
100回建てのビルにエレベーターが設置されています。
このエレベーターには上昇ボタンと下降ボタンしかありません。
上昇ボタンを押すと2〜9階ランダムで上昇し、下降ボタンを押すと2〜9階ランダムで下降します。
(2〜9の確率は均等とします。また、100階を超える時は必ず100階で止まります)

このエレベーターに1階から乗り込み、上昇ボタンのみを17回押した時に100階に到着できる確率を求めなさい。
435132人目の素数さん:2012/04/24(火) 02:50:43.54
エレベーターは1階には来ないので解答不能
隠し特殊ボタン?で1階到達という設定自体が無茶フリ
436434:2012/04/24(火) 02:51:18.01
問題の書き方が悪かったので捕捉を、

17回とありますが、例えば15回の時点で100階に到着してもOKです。
437132人目の素数さん:2012/04/24(火) 07:27:09.22
          __ノ)-'´ ̄ ̄`ー- 、_
        , '´  _. -‐'''"二ニニ=-`ヽ、
      /   /:::::; -‐''"        `ーノ
     /   /:::::/           \
     /    /::::::/          | | |  |
     |   |:::::/ /     |  | | | |  |
      |   |::/ / / |  | ||  | | ,ハ .| ,ハ|
      |   |/ / / /| ,ハノ| /|ノレ,ニ|ル' 
     |   |  | / / レ',二、レ′ ,ィイ|゙/   私は只の数ヲタなんかとは付き合わないわ。
.     |   \ ∠イ  ,イイ|    ,`-' |      頭が良くて数学が出来てかっこいい人。それが必要条件よ。
     |     l^,人|  ` `-'     ゝ  |        さらに Ann.of Math に論文書けば十分条件にもなるわよ。
      |      ` -'\       ー'  人          一番嫌いなのは論文数を増やすためにくだらない論文を書いて
    |        /(l     __/  ヽ、           良い論文の出版を遅らせるお馬鹿な人。
     |       (:::::`‐-、__  |::::`、     ヒニニヽ、         あなたの論文が Ann of Math に accept される確率は?
    |      / `‐-、::::::::::`‐-、::::\   /,ニニ、\            それとも最近は Inv. Math. の方が上かしら?
   |      |::::::::::::::::::|` -、:::::::,ヘ ̄|'、  ヒニ二、 \
.   |      /::::::::::::::::::|::::::::\/:::O`、::\   | '、   \
   |      /:::::::::::::::::::/:::::::::::::::::::::::::::::'、::::\ノ  ヽ、  |
  |      |:::::/:::::::::/:::::::::::::::::::::::::::::::::::'、',::::'、  /:\__/‐、
  |      |/:::::::::::/::::::::::::::::::::::::::::::::::O::| '、::| く::::::::::::: ̄|
   |     /_..-'´ ̄`ー-、:::::::::::::::::::::::::::::::::::|/:/`‐'::\;;;;;;;_|
   |    |/::::::::::::::::::::::\:::::::::::::::::::::::::::::|::/::::|::::/:::::::::::/
    |   /:::::::::::::::::::::::::::::::::|:::::::::::::::::::::O::|::|::::::|:::::::::::::::/
438132人目の素数さん:2012/04/24(火) 08:52:42.93
>>385
その解答、おかしくないか?
最大値をとるxの値がmのときにyはm^2になるけど…
439132人目の素数さん:2012/04/24(火) 09:20:50.20
          __ノ)-'´ ̄ ̄`ー- 、_
        , '´  _. -‐'''"二ニニ=-`ヽ、
      /   /:::::; -‐''"        `ーノ
     /   /:::::/           \
     /    /::::::/          | | |  |
     |   |:::::/ /     |  | | | |  |
      |   |::/ / / |  | ||  | | ,ハ .| ,ハ|
      |   |/ / / /| ,ハノ| /|ノレ,ニ|ル' 
     |   |  | / / レ',二、レ′ ,ィイ|゙/   私は只の数ヲタなんかとは付き合わないわ。
.     |   \ ∠イ  ,イイ|    ,`-' |      頭が良くて数学が出来てかっこいい人。それが必要条件よ。
     |     l^,人|  ` `-'     ゝ  |        さらに Ann.of Math に論文書けば十分条件にもなるわよ。
      |      ` -'\       ー'  人          一番嫌いなのは論文数を増やすためにくだらない論文を書いて
    |        /(l     __/  ヽ、           良い論文の出版を遅らせるお馬鹿な人。
     |       (:::::`‐-、__  |::::`、     ヒニニヽ、         あなたの論文が Ann of Math に accept される確率は?
    |      / `‐-、::::::::::`‐-、::::\   /,ニニ、\            それとも最近は Inv. Math. の方が上かしら?
   |      |::::::::::::::::::|` -、:::::::,ヘ ̄|'、  ヒニ二、 \
.   |      /::::::::::::::::::|::::::::\/:::O`、::\   | '、   \
   |      /:::::::::::::::::::/:::::::::::::::::::::::::::::'、::::\ノ  ヽ、  |
  |      |:::::/:::::::::/:::::::::::::::::::::::::::::::::::'、',::::'、  /:\__/‐、
  |      |/:::::::::::/::::::::::::::::::::::::::::::::::O::| '、::| く::::::::::::: ̄|
   |     /_..-'´ ̄`ー-、:::::::::::::::::::::::::::::::::::|/:/`‐'::\;;;;;;;_|
   |    |/::::::::::::::::::::::\:::::::::::::::::::::::::::::|::/::::|::::/:::::::::::/
    |   /:::::::::::::::::::::::::::::::::|:::::::::::::::::::::O::|::|::::::|:::::::::::::::/
440132人目の素数さん:2012/04/24(火) 09:25:29.10
連立微分方程式の問題です
a,b,cは全て時間の関数で
-Db=kb
Da=kb-k'a
Dc=k'a
が成り立っています。ここで、aとcについて解きたいのですが全くわかりません。
答えは書かなくていいのですが、やり方だけ教えてくれませんか。
ちなみに反応速度論の問題です
441132人目の素数さん:2012/04/24(火) 09:29:15.23
k は何の関数だ?
442132人目の素数さん:2012/04/24(火) 09:30:14.14
          __ノ)-'´ ̄ ̄`ー- 、_
        , '´  _. -‐'''"二ニニ=-`ヽ、
      /   /:::::; -‐''"        `ーノ
     /   /:::::/           \
     /    /::::::/          | | |  |
     |   |:::::/ /     |  | | | |  |
      |   |::/ / / |  | ||  | | ,ハ .| ,ハ|
      |   |/ / / /| ,ハノ| /|ノレ,ニ|ル' 
     |   |  | / / レ',二、レ′ ,ィイ|゙/   私は只の数ヲタなんかとは付き合わないわ。
.     |   \ ∠イ  ,イイ|    ,`-' |      頭が良くて数学が出来てかっこいい人。それが必要条件よ。
     |     l^,人|  ` `-'     ゝ  |        さらに Ann.of Math に論文書けば十分条件にもなるわよ。
      |      ` -'\       ー'  人          一番嫌いなのは論文数を増やすためにくだらない論文を書いて
    |        /(l     __/  ヽ、           良い論文の出版を遅らせるお馬鹿な人。
     |       (:::::`‐-、__  |::::`、     ヒニニヽ、         あなたの論文が Ann of Math に accept される確率は?
    |      / `‐-、::::::::::`‐-、::::\   /,ニニ、\            それとも最近は Inv. Math. の方が上かしら?
   |      |::::::::::::::::::|` -、:::::::,ヘ ̄|'、  ヒニ二、 \
.   |      /::::::::::::::::::|::::::::\/:::O`、::\   | '、   \
   |      /:::::::::::::::::::/:::::::::::::::::::::::::::::'、::::\ノ  ヽ、  |
  |      |:::::/:::::::::/:::::::::::::::::::::::::::::::::::'、',::::'、  /:\__/‐、
  |      |/:::::::::::/::::::::::::::::::::::::::::::::::O::| '、::| く::::::::::::: ̄|
   |     /_..-'´ ̄`ー-、:::::::::::::::::::::::::::::::::::|/:/`‐'::\;;;;;;;_|
   |    |/::::::::::::::::::::::\:::::::::::::::::::::::::::::|::/::::|::::/:::::::::::/
    |   /:::::::::::::::::::::::::::::::::|:::::::::::::::::::::O::|::|::::::|:::::::::::::::/
443132人目の素数さん:2012/04/24(火) 09:32:27.94
>>441
ただの定数です。もともとは速度定数です。
444132人目の素数さん:2012/04/24(火) 09:41:14.04
k' は?
445132人目の素数さん:2012/04/24(火) 10:42:42.61
は異なる速度定数です。
446132人目の素数さん:2012/04/24(火) 10:44:53.81
>>440
上から順に解く。
変数分離形、線形、積分。
447132人目の素数さん:2012/04/24(火) 10:46:40.19
素早い回答ありがとうございます!やってみます。ありがとうございました。
448132人目の素数さん:2012/04/24(火) 11:49:52.21
ttp://www.youtube.com/watch?v=vxJ37Pa0VtU
これすげえんだけど一体どういうことなの?
449132人目の素数さん:2012/04/24(火) 12:11:29.20
∫(0→∞)((x^(p-1))/(1+(x^(q))))dx(0<q<p<1)
は収束しますか?
450132人目の素数さん:2012/04/24(火) 13:53:36.01
>>440
a+b+c=const としてやれば少し手間が省けるかも
451132人目の素数さん:2012/04/24(火) 14:39:21.73
>>425
∇(x・M)=x×(rot M)+M×(rot x)+(x・∇)M+(M・∇)x=x×(rot M)+(x・∇)M+(M・∇)x
∴x×(rotM)=∇(x・M)-(x・∇)M-(M・∇)x
∫_v[x×(rotM)]dv =∫_v[∇(x・M)]dv-∫_v[(x・∇)M]dv-∫_v[(M・∇)x]dv
                 (1)        (2)         (3)

(1)∫_v[∇(x・M)]dv=∫_S[n(x・M)]dS

(2)∫_v[(x・∇)M]dv=(∫_v[(x・∇M_1)]dv, ∫_v[(x・∇M_2)]dv, ∫_v[(x・∇M_3)]dv)
∫_v[(x・∇M_i)]dv=∫_v[div(M_i x)]dv-∫_v[M_i div x]dv=∫_S[(n・x)M_i]dS-∫_v[3M_i]dv
∴∫_v[(x・∇)M]dv=∫_S[(n・x)M]dS-3∫_v[M]dv

(3)(M・∇)x=M
∫_v[(M・∇)x]dv=∫_v[M]dv

∴∫_v[x×(rotM)]dv =∫_S [n(x・M)-(n・x)M]dS + 2∫_v [M]dv
452132人目の素数さん:2012/04/24(火) 15:45:33.73
>>451
内容知らねーけど見た目かっこいいな
453132人目の素数さん:2012/04/24(火) 15:49:42.61
半径rの球が4個ある
球の中心全てが互いに球面上になるように重ねる

それぞれの球の中心は、例えば
(0, 0, 0)
(r, 0, 0)
(r/2, r*sin(π/3), 0)
(r/2, r*sin(π/3)/3, r*sin(π/3))
※それぞれの中心を結ぶと正四面体になる

このとき、4個の球の集合体の体積を求めよ
454132人目の素数さん:2012/04/24(火) 15:50:40.17
>>449
しない
455132人目の素数さん:2012/04/24(火) 17:58:44.09
>>451
ご丁寧にありがとうございます。
456132人目の素数さん:2012/04/24(火) 19:23:32.77
>>450
その条件も確かに使えますね。ありがとうございます。
457132人目の素数さん:2012/04/24(火) 19:33:06.09
xlog(1-e^(-x))→0(x→∞)
となる理由を教えて下さい
458132人目の素数さん:2012/04/24(火) 19:40:53.24
m=xy/2(x+y) であるときyを求める式を求めなさい


上記の問題の答えは y=−(2mx/2m−x) なのですが、求め方(途中の考え方)を教えてください
459132人目の素数さん:2012/04/24(火) 19:49:26.24
>>458
分母払って移項してくくる
460132人目の素数さん:2012/04/24(火) 19:51:03.19
>>457
lim[n→∞](log(n))/n=0 に持ち込む。
461132人目の素数さん:2012/04/24(火) 19:59:08.83
>>458
あなたは中学卒業レヴェルにすら達していないと直感致しましたので、
かなり詳しく書きました。

m = xy/2(x+y)
 ↓両辺に2(x+y)を掛ける
m*2(x+y) = xy
 ↓右辺の2を先頭にもっていく
2m(x+y) = xy
 ↓分配する
2mx+2my = xy
 ↓両辺に-2mxを足す
2my = xy - 2mx
 ↓両辺に-xyを足す
2my - xy = -2mx
 ↓右辺をyでくくる
y(2m-x) =-2m
 ↓両辺に1/(2m-x)を掛ける
y =-2m/(2m-x)
462132人目の素数さん:2012/04/24(火) 20:29:25.77
>>461
あなたは幼稚園児レベルの認知能力も無いようですね
463132人目の素数さん:2012/04/24(火) 20:59:13.60
おねがいします


ベジエ曲線を論じるときに出てくるBernstein多項式ですが、
・母関数
・直交性の証明
を教えてください

いちおう、ネットと手持ちの参考書は調べたのですが、見当たりませんでした
464132人目の素数さん:2012/04/24(火) 21:19:51.14
集合Xの濃度をmとすると
"Xから自分自身への全単射の全部からなる集合"の濃度はどう表せるか?
465132人目の素数さん:2012/04/24(火) 21:51:46.96
直感では濃度m
466132人目の素数さん:2012/04/24(火) 21:52:59.74
>>461
ありがとうございます!

中坊でごめんなさい
467132人目の素数さん:2012/04/24(火) 21:54:53.55
>>464
m!
468132人目の素数さん:2012/04/24(火) 22:01:43.40
中A女子です。
数学の宿題教えて下さい。

(1N=5p)
問 0.47Nは何pになるか?

この問題の解き方を教えて下さい。
よろしくお願いします。
469132人目の素数さん:2012/04/24(火) 22:05:57.58
>>468
まず証拠をあっぷしなさい
470132人目の素数さん:2012/04/24(火) 22:07:28.44
約2.5cm
471132人目の素数さん:2012/04/24(火) 22:13:43.70
>>434
1-(2から9までの目がある正八面体のサイコロを17個振ったときにその合計が98以下になる確率)
472132人目の素数さん:2012/04/24(火) 22:15:37.32
>>468
1N=1 kg m/s^2
473132人目の素数さん:2012/04/24(火) 22:38:15.15
2x^2+y^2=C
上の曲線群の微分方程式を求めよと言う問題なのですが


y=-2x^2/y+C/y
y'=-4x/y
yy'=-4x

と言うように計算したのですが、答えをみるとyy'=-2xとなっていました
どのように計算すれば答えのようになるんですか?

何を勘違いしてるかがわからないorz
474132人目の素数さん:2012/04/24(火) 22:39:29.43
          __ノ)-'´ ̄ ̄`ー- 、_
        , '´  _. -‐'''"二ニニ=-`ヽ、
      /   /:::::; -‐''"        `ーノ
     /   /:::::/           \
     /    /::::::/          | | |  |
     |   |:::::/ /     |  | | | |  |
      |   |::/ / / |  | ||  | | ,ハ .| ,ハ|
      |   |/ / / /| ,ハノ| /|ノレ,ニ|ル' 
     |   |  | / / レ',二、レ′ ,ィイ|゙/   私は只の数ヲタなんかとは付き合わないわ。
.     |   \ ∠イ  ,イイ|    ,`-' |      頭が良くて数学が出来てかっこいい人。それが必要条件よ。
     |     l^,人|  ` `-'     ゝ  |        さらに Ann.of Math に論文書けば十分条件にもなるわよ。
      |      ` -'\       ー'  人          一番嫌いなのは論文数を増やすためにくだらない論文を書いて
    |        /(l     __/  ヽ、           良い論文の出版を遅らせるお馬鹿な人。
     |       (:::::`‐-、__  |::::`、     ヒニニヽ、         あなたの論文が Ann of Math に accept される確率は?
    |      / `‐-、::::::::::`‐-、::::\   /,ニニ、\            それとも最近は Inv. Math. の方が上かしら?
   |      |::::::::::::::::::|` -、:::::::,ヘ ̄|'、  ヒニ二、 \
.   |      /::::::::::::::::::|::::::::\/:::O`、::\   | '、   \
   |      /:::::::::::::::::::/:::::::::::::::::::::::::::::'、::::\ノ  ヽ、  |
  |      |:::::/:::::::::/:::::::::::::::::::::::::::::::::::'、',::::'、  /:\__/‐、
  |      |/:::::::::::/::::::::::::::::::::::::::::::::::O::| '、::| く::::::::::::: ̄|
   |     /_..-'´ ̄`ー-、:::::::::::::::::::::::::::::::::::|/:/`‐'::\;;;;;;;_|
   |    |/::::::::::::::::::::::\:::::::::::::::::::::::::::::|::/::::|::::/:::::::::::/
    |   /:::::::::::::::::::::::::::::::::|:::::::::::::::::::::O::|::|::::::|:::::::::::::::/
475132人目の素数さん:2012/04/24(火) 22:42:34.21
じつは
x^2+y^2=C
だとか……
476132人目の素数さん:2012/04/24(火) 22:51:23.60
>>475
何回見ても2x^2+y^2=Cです・・・

もう答えの印刷ミスってことでいいんでしょうか・・・
477132人目の素数さん:2012/04/24(火) 22:58:36.84
>>476
与式の両辺を x で微分しただけでは?
  4x + 2yy’ = 0
478132人目の素数さん:2012/04/24(火) 23:01:42.51
あんた分母のyを微分するのを忘れている(わかってない)
(1/y)' = - y' / y^2



見づらいだろうから、yを変に弄らずそのまま計算しろ

4 x + 2 y y' = 0

2 x = - y y'
479132人目の素数さん:2012/04/24(火) 23:06:44.78
ああ・・・

どうもxだけしかみてなかったです


ご教授ありがとうございました
480132人目の素数さん:2012/04/24(火) 23:07:21.67
質問させてください。

(ab)^2+(bc)^2+(ca)^2≧(ab)(bc)+(bc)(ca)+(ca)(ab)

以上の不等式が成り立つらしいのですが理由がわかりません。
条件はa,b,cは実数であり、
a^2+b^2+c^2≧ab+bc+ca


よろしくお願いします。
481132人目の素数さん:2012/04/24(火) 23:11:38.18
abをαなどと置いてみてはいかがでしょうか
482132人目の素数さん:2012/04/24(火) 23:24:50.81

なるほど!

α^2+β^2+γ^2≧αβ+βγ+γα

として左辺を右辺に移行して両辺に2をかけて整理して出来ますね!

幼稚な質問ですいませんでした。
483132人目の素数さん:2012/04/24(火) 23:29:42.07
dimV 個の要素を持つVの部分集合が1次独立であるか、またはVを生成すればSはVのになる。

問題ではないのですがこの定理の意味がまるで分かりません。どう検索したら参考になるページが出てくるのかわからないレベルです。とりあえずdimVの意味は調べてわかりましたが...
484132人目の素数さん:2012/04/24(火) 23:33:35.81
> またはVを生成すればSはVのになる。
?
485132人目の素数さん:2012/04/24(火) 23:36:15.98
>>482
お前は何を(ry

α、β、γをa、b、cに書き換えればそのまま同じ式だろ
486132人目の素数さん:2012/04/24(火) 23:40:43.36
>>483
問題文が謎だが、要するに

3次元空間の中に、互いに独立な3つのベクトルを考えた時、それらが張る部分空間は元の3次元空間そのもの

を一般的に証明しろということではないだろうか
487132人目の素数さん:2012/04/25(水) 01:16:05.86
関数fn、gnがI上一様に
f、gに収束するならば
fngnはI上一様に fgに収束しますか?
488132人目の素数さん:2012/04/25(水) 01:19:30.74
こういう問題は自分でなっとくのいくまで考えないといけない
489132人目の素数さん:2012/04/25(水) 01:21:14.36
収束します
490132人目の素数さん:2012/04/25(水) 01:23:26.58
>>473と同じような問題ですが

C1x^2+C2y^2=1
この式の微分方程式を求める問題です
(C1,C2は任意定数)

1度微分したもの(2C1x+2C2yy'=0)と2度微分したもの(2C1+2C2y''=0)をひたすらこねくり回してるのですが答えが出ません。

どう計算すれば答えが導けるのでしょうか。過程を教えてください


ちなみに答えはxyy''+2xy'=yy'です


もしかして、また微分間違えてますか?
491132人目の素数さん:2012/04/25(水) 01:28:18.82
>>490
yy’ の微分でミスしている
積,合成関数の導関数公式を用いる
492132人目の素数さん:2012/04/25(水) 02:54:05.29
正規分布の問題です

各クラスの平均は異なるμ1 ≠ μ2 が、分散σ1 = σ2 が等しく、事前確率が等しいとき
各クラスの確率変数が等しくなる変数の値Xを求めよ
493132人目の素数さん:2012/04/25(水) 04:52:17.01
>>487
I=[0,1]
fn(x)=gn(x)={ x=0 なら 1/n, x≠0 なら 1/x+1/n }
としてみろ。
494132人目の素数さん:2012/04/25(水) 05:13:22.80
したらどうなるの?
命題が正しいことはわかるが
495132人目の素数さん:2012/04/25(水) 06:20:07.75
          __ノ)-'´ ̄ ̄`ー- 、_
        , '´  _. -‐'''"二ニニ=-`ヽ、
      /   /:::::; -‐''"        `ーノ
     /   /:::::/           \
     /    /::::::/          | | |  |
     |   |:::::/ /     |  | | | |  |
      |   |::/ / / |  | ||  | | ,ハ .| ,ハ|
      |   |/ / / /| ,ハノ| /|ノレ,ニ|ル' 
     |   |  | / / レ',二、レ′ ,ィイ|゙/   私は只の数ヲタなんかとは付き合わないわ。
.     |   \ ∠イ  ,イイ|    ,`-' |      頭が良くて数学が出来てかっこいい人。それが必要条件よ。
     |     l^,人|  ` `-'     ゝ  |        さらに Ann.of Math に論文書けば十分条件にもなるわよ。
      |      ` -'\       ー'  人          一番嫌いなのは論文数を増やすためにくだらない論文を書いて
    |        /(l     __/  ヽ、           良い論文の出版を遅らせるお馬鹿な人。
     |       (:::::`‐-、__  |::::`、     ヒニニヽ、         あなたの論文が Ann of Math に accept される確率は?
    |      / `‐-、::::::::::`‐-、::::\   /,ニニ、\            それとも最近は Inv. Math. の方が上かしら?
   |      |::::::::::::::::::|` -、:::::::,ヘ ̄|'、  ヒニ二、 \
.   |      /::::::::::::::::::|::::::::\/:::O`、::\   | '、   \
   |      /:::::::::::::::::::/:::::::::::::::::::::::::::::'、::::\ノ  ヽ、  |
  |      |:::::/:::::::::/:::::::::::::::::::::::::::::::::::'、',::::'、  /:\__/‐、
  |      |/:::::::::::/::::::::::::::::::::::::::::::::::O::| '、::| く::::::::::::: ̄|
   |     /_..-'´ ̄`ー-、:::::::::::::::::::::::::::::::::::|/:/`‐'::\;;;;;;;_|
   |    |/::::::::::::::::::::::\:::::::::::::::::::::::::::::|::/::::|::::/:::::::::::/
    |   /:::::::::::::::::::::::::::::::::|:::::::::::::::::::::O::|::|::::::|:::::::::::::::/
496132人目の素数さん:2012/04/25(水) 06:28:16.92
>>490
(y'^2+yy'')x=yy'
497132人目の素数さん:2012/04/25(水) 07:28:34.13
すみません
>>487はfngnがfgに一様収束するかどうか知りたかったです
成り立ちませんか…?
498132人目の素数さん:2012/04/25(水) 08:16:59.69
置換積分だけど
∫(1+x/L)^-2  は
−(1+x/L)^-1 ×1/L
−1/(L+x)
でいいよね?

X=0〜Lだと
1/2Lになるんだけど参考書だとL/2

置換部分の微分が1/LではなくL???
499132人目の素数さん:2012/04/25(水) 08:26:43.97
          __ノ)-'´ ̄ ̄`ー- 、_
        , '´  _. -‐'''"二ニニ=-`ヽ、
      /   /:::::; -‐''"        `ーノ
     /   /:::::/           \
     /    /::::::/          | | |  |
     |   |:::::/ /     |  | | | |  |
      |   |::/ / / |  | ||  | | ,ハ .| ,ハ|
      |   |/ / / /| ,ハノ| /|ノレ,ニ|ル' 
     |   |  | / / レ',二、レ′ ,ィイ|゙/   私は只の数ヲタなんかとは付き合わないわ。
.     |   \ ∠イ  ,イイ|    ,`-' |      頭が良くて数学が出来てかっこいい人。それが必要条件よ。
     |     l^,人|  ` `-'     ゝ  |        さらに Ann.of Math に論文書けば十分条件にもなるわよ。
      |      ` -'\       ー'  人          一番嫌いなのは論文数を増やすためにくだらない論文を書いて
    |        /(l     __/  ヽ、           良い論文の出版を遅らせるお馬鹿な人。
     |       (:::::`‐-、__  |::::`、     ヒニニヽ、         あなたの論文が Ann of Math に accept される確率は?
    |      / `‐-、::::::::::`‐-、::::\   /,ニニ、\            それとも最近は Inv. Math. の方が上かしら?
   |      |::::::::::::::::::|` -、:::::::,ヘ ̄|'、  ヒニ二、 \
.   |      /::::::::::::::::::|::::::::\/:::O`、::\   | '、   \
   |      /:::::::::::::::::::/:::::::::::::::::::::::::::::'、::::\ノ  ヽ、  |
  |      |:::::/:::::::::/:::::::::::::::::::::::::::::::::::'、',::::'、  /:\__/‐、
  |      |/:::::::::::/::::::::::::::::::::::::::::::::::O::| '、::| く::::::::::::: ̄|
   |     /_..-'´ ̄`ー-、:::::::::::::::::::::::::::::::::::|/:/`‐'::\;;;;;;;_|
   |    |/::::::::::::::::::::::\:::::::::::::::::::::::::::::|::/::::|::::/:::::::::::/
    |   /:::::::::::::::::::::::::::::::::|:::::::::::::::::::::O::|::|::::::|:::::::::::::::/
500132人目の素数さん:2012/04/25(水) 08:38:59.81
>置換積分だけど

なら置換しろ

>∫(1+x/L)^-2  は

意味不明
501132人目の素数さん:2012/04/25(水) 08:51:34.91
置換して後で戻すからそのままでいいんじゃね?
502132人目の素数さん:2012/04/25(水) 08:52:28.63
>>498
> ∫(1+x/L)^-2  は
> −(1+x/L)^-1 ×1/L
> −1/(L+x)
> でいいよね?
間違い。-1/(L+x)を微分すると
(-1/(L+x))'=1/((L+x)^2)
=(1/(L^2))*1/((1+x/L)^2)
=(1/(L^2))*(1+x/L)^(-2)

> ∫(1+x/L)^-2  は
そして記法が違う
∫(1+x/L)^(-2)*dx
503132人目の素数さん:2012/04/25(水) 09:03:55.17
激しく勘違いしてたわ
置換部分の微分じゃなく微分の逆数をかけるんだな

orz
504132人目の素数さん:2012/04/25(水) 11:03:25.62
f:R→R で、
aは無理数⇒x=aでfは不連続
a∈Q⇒x=aでfは連続
――なるものは存在しませんよね??逆はあるけど…
結論だけ教えて欲しいです。
505132人目の素数さん:2012/04/25(水) 11:07:19.00
↑証明や反例は書かないで下さい
506132人目の素数さん:2012/04/25(水) 11:32:19.99
↓は書かないで下さい
507132人目の素数さん:2012/04/25(水) 11:49:32.37
>>463
プ板で聞けば
508132人目の素数さん:2012/04/25(水) 12:59:56.20
          __ノ)-'´ ̄ ̄`ー- 、_
        , '´  _. -‐'''"二ニニ=-`ヽ、
      /   /:::::; -‐''"        `ーノ
     /   /:::::/           \
     /    /::::::/          | | |  |
     |   |:::::/ /     |  | | | |  |
      |   |::/ / / |  | ||  | | ,ハ .| ,ハ|
      |   |/ / / /| ,ハノ| /|ノレ,ニ|ル' 
     |   |  | / / レ',二、レ′ ,ィイ|゙/   私は只の数ヲタなんかとは付き合わないわ。
.     |   \ ∠イ  ,イイ|    ,`-' |      頭が良くて数学が出来てかっこいい人。それが必要条件よ。
     |     l^,人|  ` `-'     ゝ  |        さらに Ann.of Math に論文書けば十分条件にもなるわよ。
      |      ` -'\       ー'  人          一番嫌いなのは論文数を増やすためにくだらない論文を書いて
    |        /(l     __/  ヽ、           良い論文の出版を遅らせるお馬鹿な人。
     |       (:::::`‐-、__  |::::`、     ヒニニヽ、         あなたの論文が Ann of Math に accept される確率は?
    |      / `‐-、::::::::::`‐-、::::\   /,ニニ、\            それとも最近は Inv. Math. の方が上かしら?
   |      |::::::::::::::::::|` -、:::::::,ヘ ̄|'、  ヒニ二、 \
.   |      /::::::::::::::::::|::::::::\/:::O`、::\   | '、   \
   |      /:::::::::::::::::::/:::::::::::::::::::::::::::::'、::::\ノ  ヽ、  |
  |      |:::::/:::::::::/:::::::::::::::::::::::::::::::::::'、',::::'、  /:\__/‐、
  |      |/:::::::::::/::::::::::::::::::::::::::::::::::O::| '、::| く::::::::::::: ̄|
   |     /_..-'´ ̄`ー-、:::::::::::::::::::::::::::::::::::|/:/`‐'::\;;;;;;;_|
   |    |/::::::::::::::::::::::\:::::::::::::::::::::::::::::|::/::::|::::/:::::::::::/
    |   /:::::::::::::::::::::::::::::::::|:::::::::::::::::::::O::|::|::::::|:::::::::::::::/
509132人目の素数さん:2012/04/25(水) 13:15:02.87
青チャートで東工大は目指せますか?
510132人目の素数さん:2012/04/25(水) 13:22:14.43
>>504
ある方に100ガバス。
511132人目の素数さん:2012/04/25(水) 14:04:10.78
複素関数の主値積分が収束しない例はありますか?
512132人目の素数さん:2012/04/25(水) 14:16:54.61
地上200階地下101階のビルにエレベーターが設置されています。

このエレベーターには上昇ボタンと下降ボタンしかありません。

上昇ボタンを押すと1〜n階ランダムで上昇し、下降ボタンを押すと1〜n階ランダムで下降します。
1〜nの確率は均等で、nは100以下の自然数とします。

このエレベーターに1階から乗り込み、100階まで向かいます。
行き過ぎた場合には、反対側のボタンを必ず押すことにします。

m回ボタンを押して100階に到着する確率をP(m)とし、P(m)を最大にするmが最小となるnの値を
求めよ。
513132人目の素数さん:2012/04/25(水) 14:31:03.84
>>509
余裕
514132人目の素数さん:2012/04/25(水) 15:16:58.39
          __ノ)-'´ ̄ ̄`ー- 、_
        , '´  _. -‐'''"二ニニ=-`ヽ、
      /   /:::::; -‐''"        `ーノ
     /   /:::::/           \
     /    /::::::/          | | |  |
     |   |:::::/ /     |  | | | |  |
      |   |::/ / / |  | ||  | | ,ハ .| ,ハ|
      |   |/ / / /| ,ハノ| /|ノレ,ニ|ル' 
     |   |  | / / レ',二、レ′ ,ィイ|゙/   私は只の数ヲタなんかとは付き合わないわ。
.     |   \ ∠イ  ,イイ|    ,`-' |      頭が良くて数学が出来てかっこいい人。それが必要条件よ。
     |     l^,人|  ` `-'     ゝ  |        さらに Ann.of Math に論文書けば十分条件にもなるわよ。
      |      ` -'\       ー'  人          一番嫌いなのは論文数を増やすためにくだらない論文を書いて
    |        /(l     __/  ヽ、           良い論文の出版を遅らせるお馬鹿な人。
     |       (:::::`‐-、__  |::::`、     ヒニニヽ、         あなたの論文が Ann of Math に accept される確率は?
    |      / `‐-、::::::::::`‐-、::::\   /,ニニ、\            それとも最近は Inv. Math. の方が上かしら?
   |      |::::::::::::::::::|` -、:::::::,ヘ ̄|'、  ヒニ二、 \
.   |      /::::::::::::::::::|::::::::\/:::O`、::\   | '、   \
   |      /:::::::::::::::::::/:::::::::::::::::::::::::::::'、::::\ノ  ヽ、  |
  |      |:::::/:::::::::/:::::::::::::::::::::::::::::::::::'、',::::'、  /:\__/‐、
  |      |/:::::::::::/::::::::::::::::::::::::::::::::::O::| '、::| く::::::::::::: ̄|
   |     /_..-'´ ̄`ー-、:::::::::::::::::::::::::::::::::::|/:/`‐'::\;;;;;;;_|
   |    |/::::::::::::::::::::::\:::::::::::::::::::::::::::::|::/::::|::::/:::::::::::/
    |   /:::::::::::::::::::::::::::::::::|:::::::::::::::::::::O::|::|::::::|:::::::::::::::/
515132人目の素数さん:2012/04/25(水) 15:17:20.71
A,Bをn次正方行列とし、それぞれ次で定義されるものとする。(ただしn≧3とする。)
Aの(i,j)成分={1(i=j-1のとき),0(その他のとき),Bの(i,j)成分={1(i=j+2のとき),0(その他のとき)}

(1)行列A,Bはそれぞれどのような行列か。
(2)積ABおよびBAを求めよ。

この問題で、(1)は分かったのですが(2)がわかりません。
小行列のところで出された問題なので、おそらくそれを使うと思うのですがいまいち分かりません。
教えてください。よろしくおねがいします。
516132人目の素数さん:2012/04/25(水) 16:04:11.16
>>515
n=10くらいで全部書いてみる
517132人目の素数さん:2012/04/25(水) 16:28:53.36
          __ノ)-'´ ̄ ̄`ー- 、_
        , '´  _. -‐'''"二ニニ=-`ヽ、
      /   /:::::; -‐''"        `ーノ
     /   /:::::/           \
     /    /::::::/          | | |  |
     |   |:::::/ /     |  | | | |  |
      |   |::/ / / |  | ||  | | ,ハ .| ,ハ|
      |   |/ / / /| ,ハノ| /|ノレ,ニ|ル' 
     |   |  | / / レ',二、レ′ ,ィイ|゙/   私は只の数ヲタなんかとは付き合わないわ。
.     |   \ ∠イ  ,イイ|    ,`-' |      頭が良くて数学が出来てかっこいい人。それが必要条件よ。
     |     l^,人|  ` `-'     ゝ  |        さらに Ann.of Math に論文書けば十分条件にもなるわよ。
      |      ` -'\       ー'  人          一番嫌いなのは論文数を増やすためにくだらない論文を書いて
    |        /(l     __/  ヽ、           良い論文の出版を遅らせるお馬鹿な人。
     |       (:::::`‐-、__  |::::`、     ヒニニヽ、         あなたの論文が Ann of Math に accept される確率は?
    |      / `‐-、::::::::::`‐-、::::\   /,ニニ、\            それとも最近は Inv. Math. の方が上かしら?
   |      |::::::::::::::::::|` -、:::::::,ヘ ̄|'、  ヒニ二、 \
.   |      /::::::::::::::::::|::::::::\/:::O`、::\   | '、   \
   |      /:::::::::::::::::::/:::::::::::::::::::::::::::::'、::::\ノ  ヽ、  |
  |      |:::::/:::::::::/:::::::::::::::::::::::::::::::::::'、',::::'、  /:\__/‐、
  |      |/:::::::::::/::::::::::::::::::::::::::::::::::O::| '、::| く::::::::::::: ̄|
   |     /_..-'´ ̄`ー-、:::::::::::::::::::::::::::::::::::|/:/`‐'::\;;;;;;;_|
   |    |/::::::::::::::::::::::\:::::::::::::::::::::::::::::|::/::::|::::/:::::::::::/
    |   /:::::::::::::::::::::::::::::::::|:::::::::::::::::::::O::|::|::::::|:::::::::::::::/
518132人目の素数さん:2012/04/25(水) 16:36:55.96
>>516
画像ですみません。こんな感じになりました。
http://www.dotup.org/uploda/www.dotup.org2901705.jpg
しかし、nとなると途端に分からなくなります。
519132人目の素数さん:2012/04/25(水) 16:55:40.65
>>518
そんな綺麗な形をしていて何が分からないのか分からない
520132人目の素数さん:2012/04/25(水) 17:40:20.95
          __ノ)-'´ ̄ ̄`ー- 、_
        , '´  _. -‐'''"二ニニ=-`ヽ、
      /   /:::::; -‐''"        `ーノ
     /   /:::::/           \
     /    /::::::/          | | |  |
     |   |:::::/ /     |  | | | |  |
      |   |::/ / / |  | ||  | | ,ハ .| ,ハ|
      |   |/ / / /| ,ハノ| /|ノレ,ニ|ル' 
     |   |  | / / レ',二、レ′ ,ィイ|゙/   私は只の数ヲタなんかとは付き合わないわ。
.     |   \ ∠イ  ,イイ|    ,`-' |      頭が良くて数学が出来てかっこいい人。それが必要条件よ。
     |     l^,人|  ` `-'     ゝ  |        さらに Ann.of Math に論文書けば十分条件にもなるわよ。
      |      ` -'\       ー'  人          一番嫌いなのは論文数を増やすためにくだらない論文を書いて
    |        /(l     __/  ヽ、           良い論文の出版を遅らせるお馬鹿な人。
     |       (:::::`‐-、__  |::::`、     ヒニニヽ、         あなたの論文が Ann of Math に accept される確率は?
    |      / `‐-、::::::::::`‐-、::::\   /,ニニ、\            それとも最近は Inv. Math. の方が上かしら?
   |      |::::::::::::::::::|` -、:::::::,ヘ ̄|'、  ヒニ二、 \
.   |      /::::::::::::::::::|::::::::\/:::O`、::\   | '、   \
   |      /:::::::::::::::::::/:::::::::::::::::::::::::::::'、::::\ノ  ヽ、  |
  |      |:::::/:::::::::/:::::::::::::::::::::::::::::::::::'、',::::'、  /:\__/‐、
  |      |/:::::::::::/::::::::::::::::::::::::::::::::::O::| '、::| く::::::::::::: ̄|
   |     /_..-'´ ̄`ー-、:::::::::::::::::::::::::::::::::::|/:/`‐'::\;;;;;;;_|
   |    |/::::::::::::::::::::::\:::::::::::::::::::::::::::::|::/::::|::::/:::::::::::/
    |   /:::::::::::::::::::::::::::::::::|:::::::::::::::::::::O::|::|::::::|:::::::::::::::/
521132人目の素数さん:2012/04/25(水) 20:57:02.02
>>507
工学屋に訊くだけ時間に無駄だからここにした

ぐぐればすぐにわかると思うが、直交分解の方法はすんなりは出てこない模様
プログラマーが必要とするのは幾何学的解釈で、そんなものは別に知りたくない
(とっくに知っている)

ついでに言えば、お前が言いたいのはプログラム板か、プログラマー板だと思うが
それぞれ、ム板、マ板と普通は呼ばれている


さあ、重み関数と母関数を教えろ
522132人目の素数さん:2012/04/25(水) 21:00:20.00
>>521
いやだ
523132人目の素数さん:2012/04/25(水) 21:23:41.84
ぷっ
524132人目の素数さん:2012/04/25(水) 22:00:41.81
fixed algebraically closed fieldってどういう意味ですか?
525132人目の素数さん:2012/04/25(水) 22:03:33.77
algebraically closed fieldをfixedするとき
てな感じ
526132人目の素数さん:2012/04/25(水) 22:08:11.22
-2x^2+6x+1=-2(x-3/2)^2+11/2
どう考えても-2(x-3/2)^2-5/4にしか成らん…
527132人目の素数さん:2012/04/25(水) 22:29:04.91
>>521
意味がよく分からないんですけど、幾何学的解釈って具体的に例えばどういうのですか?
528132人目の素数さん:2012/04/25(水) 22:31:16.98
>>524
fixedな代数的閉体
529132人目の素数さん:2012/04/25(水) 22:34:45.55
>>526
なら、考え方が違う。
はい、終わりさようなら。
530132人目の素数さん:2012/04/25(水) 22:42:34.51
>>526
-2(x-3/2)^2 を展開してみて
531132人目の素数さん:2012/04/25(水) 22:50:10.82
=-2(x^2-3x+9/4)
=-2x^2+6x18/4
532531:2012/04/25(水) 22:53:04.65
ミスった…
533132人目の素数さん:2012/04/25(水) 22:54:11.05
>>531
肝心の所を書き忘れるなよw
534132人目の素数さん:2012/04/25(水) 22:56:37.84
肝心要
535132人目の素数さん:2012/04/25(水) 22:57:38.01
>>530
展開したらあっさり解けたよ
サンクス
536132人目の素数さん:2012/04/26(木) 00:03:52.82

メディアは全部抑えてある。教団名を出したり逆らえばどうなるかわかってるだろうな?
<           _-=≡:: ;;   ヾ\            >
<         /          ヾ:::\           >
<         |            |::::::|          >
<        ミ|-=≡、 ミ≡==- 、 |;;;;;/          >
<         || <●>| ̄| <◎> |── /\         >
<         |ヽ_/  \_/    > /         >
<        / /(    )\      |_/          >
<        | |  ` ´        ) |           >
<        | \/ヽ/\_/  /  |           >
<        \ \ ̄ ̄ /ヽ  /  /           >
<          \  ̄ ̄   /  /       \    >
  / /        ̄ ̄ ̄ ̄ ̄ ̄ ̄     \\ \ \
     ___
   / ー\ ナンミョウホウレンゲッキョウナンミョウホウレンゲッキョウナンミョウホウレンゲッキョウ
 /ノ  (@)\ ナンミョウホウレンゲッキョウナンミョウホウレンゲッキョウナンミョウホウレンゲッキ
.| (@)   ⌒)\ ナンミョウホウレンゲッキョウナンミョウホウレンゲッキョウナンミョウホウレンゲッ
.|   (__ノ ̄|  |   ///;ト,  ナンミョウホウレンゲッキョウナンミョウホウレンゲッキョウナンミョ
 \   |_/  / ////゙l゙l;  ナンミョウホウレンゲッキョウナンミョウホウレンゲッキョウナンミョ
   \     _ノ   l   .i .! |  ナンミョウホウレンゲッキョウナンミョウホウレンゲッキョウナンミョ
   /´ 公明  `\ │   | .|  ナンミョウホウレンゲッキョウナンミョウホウレンゲッキョウナンミョ
537132人目の素数さん:2012/04/26(木) 00:08:14.04
日本って今でもアメリカの植民地なんでしょ?ちがうの?
538132人目の素数さん:2012/04/26(木) 01:58:00.62
          __ノ)-'´ ̄ ̄`ー- 、_
        , '´  _. -‐'''"二ニニ=-`ヽ、
      /   /:::::; -‐''"        `ーノ
     /   /:::::/           \
     /    /::::::/          | | |  |
     |   |:::::/ /     |  | | | |  |
      |   |::/ / / |  | ||  | | ,ハ .| ,ハ|
      |   |/ / / /| ,ハノ| /|ノレ,ニ|ル' 
     |   |  | / / レ',二、レ′ ,ィイ|゙/   私は只の数ヲタなんかとは付き合わないわ。
.     |   \ ∠イ  ,イイ|    ,`-' |      頭が良くて数学が出来てかっこいい人。それが必要条件よ。
     |     l^,人|  ` `-'     ゝ  |        さらに Ann.of Math に論文書けば十分条件にもなるわよ。
      |      ` -'\       ー'  人          一番嫌いなのは論文数を増やすためにくだらない論文を書いて
    |        /(l     __/  ヽ、           良い論文の出版を遅らせるお馬鹿な人。
     |       (:::::`‐-、__  |::::`、     ヒニニヽ、         あなたの論文が Ann of Math に accept される確率は?
    |      / `‐-、::::::::::`‐-、::::\   /,ニニ、\            それとも最近は Inv. Math. の方が上かしら?
   |      |::::::::::::::::::|` -、:::::::,ヘ ̄|'、  ヒニ二、 \
.   |      /::::::::::::::::::|::::::::\/:::O`、::\   | '、   \
   |      /:::::::::::::::::::/:::::::::::::::::::::::::::::'、::::\ノ  ヽ、  |
  |      |:::::/:::::::::/:::::::::::::::::::::::::::::::::::'、',::::'、  /:\__/‐、
  |      |/:::::::::::/::::::::::::::::::::::::::::::::::O::| '、::| く::::::::::::: ̄|
   |     /_..-'´ ̄`ー-、:::::::::::::::::::::::::::::::::::|/:/`‐'::\;;;;;;;_|
   |    |/::::::::::::::::::::::\:::::::::::::::::::::::::::::|::/::::|::::/:::::::::::/
    |   /:::::::::::::::::::::::::::::::::|:::::::::::::::::::::O::|::|::::::|:::::::::::::::/
539132人目の素数さん:2012/04/26(木) 11:07:35.04
>>537
文系板で聞いたら
540片山博文MZ ◆0lBZNi.Q7evd :2012/04/26(木) 12:26:02.83
$S$を集合とする。$n(S) \ge 3$と仮定する。
$S$の互いに異なる3個の元$w_1,w_2,w_3$と、任意の$v,u \in S$について、
\begin{equation}
((v \ne w_1) \land (u \ne w_1)) \lor ((v \ne w_2) \land (u \ne w_2)) \lor ((v \ne w_3) \land (u \ne w_3))
\end{equation}
は常に真であることをエレガントに証明してください。
541132人目の素数さん:2012/04/26(木) 13:20:46.73
((v ≠ w_1) ∧ (u ≠ w_1)) ∨ ((v ≠ w_2) ∧ (u ≠ w_2)) ∨ ((v ≠ w_3) ∧ (u ≠ w_3))
ぐらい書いたらどうだ。
だいたい S なんていらんだろ。
542132人目の素数さん:2012/04/26(木) 14:24:42.53
>>521
ggrないのね、かわいそー
543片山博文MZ ◆0lBZNi.Q7evd :2012/04/26(木) 15:28:23.00
打ち直します。
Sを集合とする。n(S)≧3と仮定する。
Sの互いに異なる3個の元 w_1, w_2, w_3と、任意の v,u∈Sについて
((v≠w_1)∧(u≠w_1)) ∨ ((v≠w_2)∧(u≠w_2)) ∨ ((v≠w_3)∧(u≠w_3))
がトートロジーになることをエレガントに証明してください。
544132人目の素数さん:2012/04/26(木) 15:50:49.30
731 :132人目の素数さん:2012/04/26(木) 04:49:04.94
X=3a+2
X=5b+3
X=11C+9
X<1000
これって、東大理Vの人でも間違える連立方程式(?)みたいなんですけど、
地道に解く場合の最初の方だけでも結構ですので、先生方、師匠方、
ご教授願います。

たけしのコマ大数学科より
545132人目の素数さん:2012/04/26(木) 15:53:41.14
次の2次方程式をとけ
(1)x2-4=0
(2)x2-3x-4-0
(3)x2+8x+16=0
(4)x2+3x-6=0
546132人目の素数さん:2012/04/26(木) 16:05:21.60
>>544
それだけだとx,a,b,cは定まらない
質問下手な人達による伝言ゲームで情報が削られたんじゃないか?
547片山博文MZ ◆0lBZNi.Q7evd :2012/04/26(木) 16:05:54.37
>>545
(1) x=±4 (2) x=-1,4 (3) x=-4(重解) (4) x=(-3±√33)/2
548132人目の素数さん:2012/04/26(木) 16:48:33.89
>>544
うろ覚えだが、abcが自然数てのが抜けてるんじゃないか?
Xも1,000を超えない最大の自然数だったような気もしてるがそのへんはもっとうろ覚え。
549132人目の素数さん:2012/04/26(木) 17:20:38.71
>>537
植民地の定義による
550132人目の素数さん:2012/04/26(木) 17:20:57.48
>>544
x,a,b,cを整数とした場合
3a-5b=1
a=5m+2 b=3m+1 (mは整数)

5b-11c=6
b=11n+10 c=5n+4 (nは整数)

3m=11n+9
m=11s+3 n=3s (sは整数)

a=55s+17 b=33s+10 c=15s+4
551132人目の素数さん:2012/04/26(木) 18:11:25.28
円錐台の形をしたバケツがある
このバケツを傾けて、水を入れてゆく
すると水面がちょうど底の部分とふたの部分に接した
このときバケツに入っている水量はバケツの容量の2分の1といえるか
552544:2012/04/26(木) 18:33:41.67
553132人目の素数さん:2012/04/26(木) 19:01:35.97
554132人目の素数さん:2012/04/26(木) 19:51:44.22
言えない。
ほとんど円錐に近い形の円錐台を
思い浮かべてみる。
555132人目の素数さん:2012/04/26(木) 20:05:51.75
556132人目の素数さん:2012/04/26(木) 20:21:32.69
Gを群、Hを部分群、a∈Gとしたとき
Hの左剰余類と右剰余類の濃度が等しいことの証明についてです。
aHからHa^-1への全単写が存在してその濃度が等しいのはわかるのですが、
Ha^-1と右剰余類Haの濃度がなぜ等しいと言えるのかがわかりません。
最近群論を勉強し始めたのですが、なにか根本的なことを見落としているのでしょうか。
低レベルで申し訳ないのですが、お分かりになる方、教えていただけないしょうか。
557544 552:2012/04/26(木) 20:27:28.66
>>550
初歩的な質問ですが、わからない文字が4つで、3つの等式で求められるのでしょうか?
558132人目の素数さん:2012/04/26(木) 20:59:21.96
>>557
実数であるとか、整数であるとか、他に条件があれば求められる場合もある。
559132人目の素数さん:2012/04/26(木) 20:59:34.52
>>543
pigeonhole principleで済ますのはエレガントなのかどうか
560132人目の素数さん:2012/04/26(木) 21:02:55.54
どうでもいいけどトートロジーって言葉の使い方(少し)間違ってる
561132人目の素数さん:2012/04/26(木) 21:18:50.52
x<(a+5)/3∩4≦x<5
でaの範囲を求める方法の解説キボンヌ
562132人目の素数さん:2012/04/26(木) 21:42:51.08
          ,__
        o'⌒) `ヽ
         (;゙;`゙☆;゙)
          (。・∞・) 願い事を3つまで叶えろ・・・
          ( ∽)
            )ノ         _
          (_          /〜ヽ
          [il=li]       <('A` 。) えっ!俺が叶えんの ?
          )=(_        (  )>
         (-==-)       <^ヾ_
          `ー‐''
563132人目の素数さん:2012/04/26(木) 21:50:12.66
>>556
HaをHa^-1に対応させる写像は右剰余類上の全単射
564132人目の素数さん:2012/04/26(木) 21:56:16.00
>>563
すみません。
もう少し詳しくお教えいただけないでしょうか。
565132人目の素数さん:2012/04/26(木) 22:02:44.44
>>564
・写像はよく定義される
・写像は全単射
を定義に沿って示す
566132人目の素数さん:2012/04/26(木) 22:12:46.64
何度もすみません
写像はよく定義される、というのはどういうことですか?
567132人目の素数さん:2012/04/26(木) 22:13:51.63
代表元のとり方によらないこと
568132人目の素数さん:2012/04/26(木) 22:21:39.10
逆元が存在するから全射、
逆元の一意性から単射…こんな感じで良いのでしょうか
569132人目の素数さん:2012/04/26(木) 22:24:44.17
だと思うけど、右同値類の定義を使って証明しないと
570132人目の素数さん:2012/04/26(木) 22:24:45.14
根本的に基礎が足りない
出直せアホ
571132人目の素数さん:2012/04/26(木) 22:27:37.19
教えていただいた方、ありがとうございます!
なんとなくわかりかけてきたので、もうすこし考えてみます
572132人目の素数さん:2012/04/26(木) 22:37:34.80
誰?
573132人目の素数さん:2012/04/26(木) 22:40:00.02
|aH|=|H|=|Ha|.
574132人目の素数さん:2012/04/27(金) 00:22:17.61
>>555
だからなに?
575132人目の素数さん:2012/04/27(金) 01:43:43.28
          __ノ)-'´ ̄ ̄`ー- 、_
        , '´  _. -‐'''"二ニニ=-`ヽ、
      /   /:::::; -‐''"        `ーノ
     /   /:::::/           \
     /    /::::::/          | | |  |
     |   |:::::/ /     |  | | | |  |
      |   |::/ / / |  | ||  | | ,ハ .| ,ハ|
      |   |/ / / /| ,ハノ| /|ノレ,ニ|ル' 
     |   |  | / / レ',二、レ′ ,ィイ|゙/   私は只の数ヲタなんかとは付き合わないわ。
.     |   \ ∠イ  ,イイ|    ,`-' |      頭が良くて数学が出来てかっこいい人。それが必要条件よ。
     |     l^,人|  ` `-'     ゝ  |        さらに Ann.of Math に論文書けば十分条件にもなるわよ。
      |      ` -'\       ー'  人          一番嫌いなのは論文数を増やすためにくだらない論文を書いて
    |        /(l     __/  ヽ、           良い論文の出版を遅らせるお馬鹿な人。
     |       (:::::`‐-、__  |::::`、     ヒニニヽ、         あなたの論文が Ann of Math に accept される確率は?
    |      / `‐-、::::::::::`‐-、::::\   /,ニニ、\            それとも最近は Inv. Math. の方が上かしら?
   |      |::::::::::::::::::|` -、:::::::,ヘ ̄|'、  ヒニ二、 \
.   |      /::::::::::::::::::|::::::::\/:::O`、::\   | '、   \
   |      /:::::::::::::::::::/:::::::::::::::::::::::::::::'、::::\ノ  ヽ、  |
  |      |:::::/:::::::::/:::::::::::::::::::::::::::::::::::'、',::::'、  /:\__/‐、
  |      |/:::::::::::/::::::::::::::::::::::::::::::::::O::| '、::| く::::::::::::: ̄|
   |     /_..-'´ ̄`ー-、:::::::::::::::::::::::::::::::::::|/:/`‐'::\;;;;;;;_|
   |    |/::::::::::::::::::::::\:::::::::::::::::::::::::::::|::/::::|::::/:::::::::::/
    |   /:::::::::::::::::::::::::::::::::|:::::::::::::::::::::O::|::|::::::|:::::::::::::::/
576132人目の素数さん:2012/04/27(金) 10:53:11.32
476 :あるケミストさん:2012/03/24(土) 20:49:23.84
k3[Fe(CN)6]→3K++ [Fe(CN)6]3-とバラバラにして、この
Feのイオンの価数をxとすると、CN-は−1なので、
[Fe(CN)6]3-:x+(−1)×6=−3 x=+3

黄色い参考書の者です。左辺だけ、何:何になっているのを生まれて
初めて見ました。解説していただけないでしょうか?
478 :476:2012/03/24(土) 21:10:11.45
−3=−3/1=−3:1でよろしいでしょうか?

■■■質問スレッド@化学板102■■■より


>>478の比の定義(?)としては間違いないですか?
577132人目の素数さん:2012/04/27(金) 11:53:27.48
>>576
> [Fe(CN)6]3-:x+(−1)×6=−3 x=+3
の「:」ってコロンとして使われているだけで、比を表してるんじゃないんじゃないか?
x+(−1)×6=−3の等式を解いてx=+3ってなってるだけなのでは?
578576:2012/04/27(金) 12:19:12.56
>>577
いや、その通りですよ。
579132人目の素数さん:2012/04/27(金) 13:35:10.66
Q1 直線OA,OBに接する半径1の円O1、直線OA,OBに接し、円O1に外接する円O2、
円O2に概説する円O3・・・と限りなく作る時、円の面積の和を求めよ
ただし、∠AOB=2θ(0<θ<90)であり、任意のnについては円Onの半径より、円On+1の方が小さいものとする。

Q2 f(x)=1/(1-x)^4 (|x|<1)の時、nCr=n!/r!(n-r)!を使ってΣの形で表す場合、どういう形になるんでしょうか?
580132人目の素数さん:2012/04/27(金) 15:52:03.12
f(x)=3x^2*∫[0,1]f(t)dt+x∫[0,1]{f'(t)}^2dt+∫[0,1]f(t)dt
f(-x)=3x^2*∫[0,1]f(t)dt-x∫[0,1]{f'(t)}^2dt+∫[0,1]f(t)dt
f(x)+f(-x)=6x^2*∫[0,1]f(t)dt+2∫[0,1]f(t)dt
f(x)-f(-x)=2x∫[0,1]{f'(t)}^2dt
f'(x)-f'(-x)=12x∫[0,1]f(t)dt
f'(x)+f'(-x)=2∫[0,1]{f'(t)}^2dt
f(x)-f(-x)=x(f'(x)+f'(-x))
f(x)-f(-x)=Ax
f(x)=3x^2*∫[0,1]f(t)dt+Ax/2+∫[0,1]f(t)dt
f'(x)=6x∫[0,1]f(t)dt+A/2
f''(x)=6∫[0,1]f(t)dt
C1=∫[0,1]f(t)dtとすると
f''(x)=6C1
f'(t)=6C1x+C2
f(t)=3C1x^2+C2x+C3
f(0)=∫[0,1]f(t)dt=C1+C2/2+C3=C3 ∴C2=-2C1
∫[0,1]f(t)dt=2C1+2C3=C1 ∴C3=-C1/2
f(x)=C(6x^2-4x-1)
f(x)=3x^2*∫[0,1]C(6x^2-4x-1)dt+x∫[0,1]{(C(6x^2-4x-1))'}^2dt+∫[0,1]C(6x^2-4x-1)dt
=6Cx^2+16C^2*x-C
C(6x^2-4x-1)=6Cx^2+16C^2*x-C ∴C=-1/4, 0
f(x)=(-6x^2+4x+1)/4, 0
581132人目の素数さん:2012/04/27(金) 15:56:25.20
『平清盛』プロデューサー在日朝鮮人 磯智明(天皇制度廃止論者)のプロデュース作品

@『かんさほうじん (2008)』反体制・反社会
A『最後の戦犯 (2008)』反日・天皇制度廃止・反体制・反社会
B『リミット -刑事の現場2- (2009)』反体制・反社会

大河の画面が汚いのも、役者が大根なのも、衣装がぼろぼろなのも、役者の下品な立ち回りも、
画面が薄暗いのも、役者が汚いのも全ての原因は




NHKが汚れているから
 史実うんぬんの話ではないのですよ。あの大河は役者、セット、演出等が、いまのNHK内部の汚れ具合を見事に反映しているのです。

薄汚れた空間内で繰り広げられる捏造・妄想(=今年の大河)は、反日・在日の脳内を表しているのです
582132人目の素数さん:2012/04/27(金) 16:03:21.88
>>579
n番目の円の半径をr[n]とすると、
r[1]=1
r[n]=r[n-1](1-sin(θ))/(1+sin(θ)) =((1-sin(θ))/(1+sin(θ)))^(n-1)
n番目の円の面積をs[n]とすると、
s[n]=π*((1-sin(θ))/(1+sin(θ)))^(2n-2)

lim[n→∞]Σ[k=1,n]s[k]=π/(1-((1-sin(θ))/(1+sin(θ)))^2)


答えはもっと綺麗な形にできるかもしれない
あんまり自信ない
583132人目の素数さん:2012/04/27(金) 16:22:20.79
おねがいします
無期懲役の百人の囚人に対し、恩赦の賭けが提供されることになった。

部屋の中のテーブルに、この百人の名札が裏を向けて横一列に並べられている。

囚人は一人ずつこの部屋に入り、百個の名札のうち五十個まで自分の好きに選んでは
表に返して名前を確認できる。

この五十個のうちに自分の名札があれば「当たり」で、その囚人は釈放される。

「当たり」でもそうでなくても、囚人は名札を元通りに裏返して別の出口から部屋を立ち去り、
そののち、次の囚人が入室する。

よって、囚人の間で連絡する手段はないし、何か状況を変えたり、情報を残す方法もない。

各囚人が自分の名札を見つける確率は 50/100、つまり二分の一だから、百人全員が釈放
される確率は公平なコイン投げを百回して百回とも表が出る確率に等しく、ほぼゼロである。


では問題。今、確率はほぼゼロと言ったばかりだが、この賭けを始める前に囚人全員で
打ち合わせをして良いなら、百人全員が釈放される確率を30 パーセント以上にできる戦略
がある。

その戦略とはどんなものでしょう?
584132人目の素数さん:2012/04/27(金) 16:53:29.66
札の置かれる場所に1〜100番まで番号を付けて、
1人目は1〜50番を選ぶ、2人目は2〜51番を選ぶ、3人目は・・・100人目は100・1〜49番を選ぶ

この場合に釈放人数の分散は最大になるけど、
全員釈放率(=全員釈放されない率)が3割も行くわけないので多分数学の問題じゃない

答えわかったら教えて
585132人目の素数さん:2012/04/27(金) 17:44:58.19
その手のクソ問題はハフマン符合とかなんぞやとかの
アレだよ!とか言ってるの
前見たぞ
586132人目の素数さん:2012/04/27(金) 19:03:43.92
あああれか
あんたがった タフマンってやつだな
587132人目の素数さん:2012/04/27(金) 19:14:04.16
ベクトル空間を拡大して商体みたいなの作れないの?
588あのこうちやんは始皇帝だった:2012/04/27(金) 19:20:20.59

 お前たちは、定職に就くのが先決だろがああああああ!!!!!!!!!!!

 ニート・無職の、ゴミ・クズ・カスのクソガキどもがあああああ!!!!!!!!!!!
589132人目の素数さん:2012/04/27(金) 19:22:33.91
0以上θ<2πの条件で、方程式con^2+√3sinθconθ=1を満たすθを求めよ。

conかsinに統一するっぽいんだが・・・。
590132人目の素数さん:2012/04/27(金) 19:25:10.06
めちゃくちゃ

やり直し。
591132人目の素数さん:2012/04/27(金) 19:28:20.20
円周と1点ってホモトピー同値?
592132人目の素数さん:2012/04/27(金) 19:29:48.01
ホモトピーの登場する文脈完全無視ですな
593132人目の素数さん:2012/04/27(金) 20:30:09.17
ホモとピーしてえな
594132人目の素数さん:2012/04/27(金) 20:33:04.60
>>580 訂正
f(x)=3x^2*∫[-1,1]f(t)dt+x∫[0,1]{f'(t)}^2dt+∫[0,1]f(t)dt
f(-x)=3x^2*∫[-1,1]f(t)dt-x∫[0,1]{f'(t)}^2dt+∫[0,1]f(t)dt
f(x)+f(-x)=6x^2*∫[-1,1]f(t)dt+2∫[0,1]f(t)dt
f(x)-f(-x)=2x∫[0,1]{f'(t)}^2dt
f'(x)-f'(-x)=12x∫[-1,1]f(t)dt
f'(x)+f'(-x)=2∫[0,1]{f'(t)}^2dt
f(x)-f(-x)=x(f'(x)+f'(-x))
f(x)-f(-x)=Ax
f(x)=3x^2*∫[-1,1]f(t)dt+Ax/2+∫[0,1]f(t)dt
f'(x)=6x∫[-1,1]f(t)dt+A/2
f''(x)=6∫[-1,1]f(t)dt
C1=∫[-1,1]f(t)dtとすると
f''(x)=6C1
f'(t)=6C1x+C2
f(t)=3C1x^2+C2x+C3
f(0)=∫[0,1]f(t)dt=C1+C2/2+C3=C3 ∴C2=-2C1
∫[-1,1]f(t)dt=2C1+2C3=C1 ∴C3=-C1/2
f(x)=C(6x^2-4x-1)
f(x)=3x^2*∫[-1,1]C(6x^2-4x-1)dt+x∫[0,1]{(C(6x^2-4x-1))'}^2dt+∫[0,1]C(6x^2-4x-1)dt
=6Cx^2+16C^2*x-C
C(6x^2-4x-1)=6Cx^2+16C^2*x-C ∴C=-1/4, 0
f(x)=(-6x^2+4x+1)/4, 0
595132人目の素数さん:2012/04/27(金) 20:41:18.94
z=f(x、y)とこれをyについて解いたy=g(x、z)の偏微分がゼロになる
∂g(x、z)/∂z = 0 を連立させた式からzを消去するとx、yだけの曲線がでますが
この包絡線って曲面z=f(x、y)をz軸真上から見た影というか輪郭の曲線って思っていいのでしょうか? 

596132人目の素数さん:2012/04/27(金) 20:54:30.81
>>595
例えば z=y^(1/3) のときどうなるの?
597132人目の素数さん:2012/04/27(金) 21:13:39.09
極限の計算なんですが、

lim_{n→∞} e^(-x√n)(1 + x/√n)^n

の値はどう計算すれば良いんでしょうか。
本にはe^xの定義の拡張である、としか書いておらず、
自分で計算したり調べてみたりしたのですが分かりませんでした。
598132人目の素数さん:2012/04/27(金) 21:29:27.86
>>597
m=√n とおいてみる
599132人目の素数さん:2012/04/27(金) 22:02:50.52
一応それもやってみてはいるのですが、
lim_{m} e^(-mx) (1+x/m)^(m^2)から
指数の型にm^2が出て来るのが思いのほか厄介で進めませんでした。
600132人目の素数さん:2012/04/27(金) 22:38:23.98
1 名無しさんにズームイン! [] Date:2012/03/28(水) 08:28:15.02 ?ID:NWYs/2ZP Be:
 やらなけゃいけない
 電○の各局への圧力が半端ないんです
 昨日、一昨日前田AKB卒業ネタやった情報番組全てが前田AKB卒業ネタ中の毎分で視聴率がダダ下がりしました。
 各局本音では毎分視聴率ダダ下がりするこのネタははやりたくなかったけど原子力村以上に電○からの圧力が凄いんです

ブーム捏造、枕営業、自社買い、サクラの動員そして
AKBの捏造ブームのために税金が大量に使われている証拠がこちら

やっと気付いた「AKBに電通が絡んでる」ではなく「AKBの正体が電通」な件 その127
http://hayabusa3.2ch.net/test/read.cgi/morningcoffee/1335468718/
テレビの捏造ブームに騙されるな
601132人目の素数さん:2012/04/27(金) 22:41:42.61
オメーの書き方だとeの肩に (-x√n)(1 + x/√n)^n が乗っかってるようにみえんだよ
さっさと書きなおせアホ 括弧つけろ
602132人目の素数さん:2012/04/27(金) 22:49:47.13
>>599
-mx + m^2 ln(1+x/m)
= -mx + m^2(x/m - x^2/m^2/2 + O(1/m^3))
= -x^2/2 + O(1/m)
603教えろください:2012/04/27(金) 22:58:44.44
関数y=e^xのグラフに点(1,0)から引いた接線の方程式を求めよw
604132人目の素数さん:2012/04/27(金) 23:23:59.09
求めたまふw
605576 578:2012/04/27(金) 23:46:35.29
476 :あるケミストさん:2012/03/24(土) 20:49:23.84
k3[Fe(CN)6]→3K++ [Fe(CN)6]3-とバラバラにして、この
Feのイオンの価数をxとすると、CN-は−1なので、
[Fe(CN)6]3-:x+(−1)×6=−3 x=+3

黄色い参考書の者です。左辺だけ、何:何になっているのを生まれて
初めて見ました。解説していただけないでしょうか?
478 :476:2012/03/24(土) 21:10:11.45
−3=−3/1=−3:1でよろしいでしょうか?

■■■質問スレッド@化学板102■■■より


>478の比の定義(?)としては間違いないですか?
606132人目の素数さん:2012/04/27(金) 23:46:42.90
>>579,582
lim[n→∞]Σ[k=1,n]s[k]=π*(sinθ+1)^2/(4*sinθ)
答えを変形しただけ
607132人目の素数さん:2012/04/28(土) 00:16:11.97
>>602
ありがとうございます
なるほど、Taylor展開すれば良かったんですね
608132人目の素数さん:2012/04/28(土) 00:42:41.59
>>605
> 476 :あるケミストさん:2012/03/24(土) 20:49:23.84
この:はどういう比を表しているのでしょうか?と尋ねているようなものかと
609605:2012/04/28(土) 04:44:30.85
>>608
そうです。で、化学板の質問も私です。
簡単に申し上げますと、−3=−3/1=−3:1があっているのかと
尋ねたんでした。
610132人目の素数さん:2012/04/28(土) 08:33:18.27
四面体ABCDがあり、AB=AC=8、AD=BD=6√2、cos∠BAC=4/5、∠CAD=45°である。
点ACの中点をMとし、辺AB上にAC⊥PMとなるように点Pを、辺AD上にAC⊥QMとなるように点Qをとる。
(2)
線分PQの長さを求めよ
(3)
点Pから平面ACDに垂線を引き、その交点をHとする。線分PHの長さを求めよ

たのむ(´・ω・`)
611132人目の素数さん:2012/04/28(土) 08:36:34.93
>>609
>>578はどういう意味なんだ?
>>577の言うとおりだとして、君もそう思っているのなら、
> 左辺だけ、何:何になっている
と言うのは一体何のことを言っているんだ?
> −3=−3/1=−3:1
って一体どこから出てきたんだ?

ちなみに、比と比の値を等号で結ぶことは学校教育ではしない。
612132人目の素数さん:2012/04/28(土) 11:52:21.80
lim[x→a]f(x)=A,lim[x→a]g(x)=B
のとき

lim[x→a]f(x)g(x)=AB

を証明せよ


どうやって
|f(x)g(x)-AB|<ε
の式を作るんだか分からないです
613132人目の素数さん:2012/04/28(土) 12:01:12.20
|f(x)g(x)-AB|
=|f(x)g(x)-f(x)B+f(x)B-AB|
≦|f(x)||g(x)-B|+|B||f(x)-A|

∃M>0 |f(x)|<M
614132人目の素数さん:2012/04/28(土) 12:09:20.89
>>613
3行目の式が分かりません
f(x)とBがかけられているのはどう処理しているんですか?
615132人目の素数さん:2012/04/28(土) 12:39:57.18
円周と直線はホモトピー同値ですか?
616132人目の素数さん:2012/04/28(土) 12:44:31.60
ホモトピーの定義を1000000回見なおせ
617132人目の素数さん:2012/04/28(土) 12:59:23.86
(3n)x-2y=1,(n,2)=1
x=(2m+1),y=(3nm+(3n-1)/2)

(3n+1)x-2y=1,(n,2)=2
x=(2m+1),y=(3n+1)m+(3n)/2

(3n+2)x-2y=1,(n,2)=1
x=(2m+1),y=(3n+2)m+(3n+1)/2

連続した合成数が途切れた場所が素数

25-24=(5)(2*2+1)-2(5*2+2)
618132人目の素数さん:2012/04/28(土) 13:11:36.72
京都の事故

44 名前:名無しさん@12周年[] 投稿日:2012/04/12(木) 20:06:17.25 ID:LKXz/REk0
かつてはてんかん患者の自動車免許取得は法的に制限されていたが、
2002年6月の道路交通法改正によって、発作が起きても意識障害を伴わない又は、
発作が就寝中に限るなどの患者は、公安委員会の検査や、医師の診断書を提出するなどの条件付で取得に道が開かれた。
日本てんかん協会の運動で、てんかんでも免許取得が可能になった。
その結果、案の定、2002年以降、てんかん患者のひきおこした重大な事故が急増

2011/04 栃木・鹿沼   6人死亡 (持病を隠蔽、過去に事故2件)
2011/04 島根・松江   1人死亡 (持病を隠蔽、薬飲まず)
2011/05 広島・福山   4人重軽傷 (過去に事故2件)
2011/07 愛知・岩倉   2人死亡 (通院歴なしで不起訴)
2011/10 鹿児島・姶良 1人死亡4人重軽傷 (過去に物損事故、薬飲まず)
2012/02 栃木・宇都宮 6人重軽傷 (昨年7月に事故、運転しないと誓約書)

すると2002年の法改正で可能になった、てんかん患者の自動車免許取得を既得権益とする
共産党が支援している日本てんかん協会は
「てんかん患者の権利を守れ」と法務大臣に要望書を提出
http://uni.2ch.net/test/read.cgi/newsplus/1334225953/

その三日後にまたしても死亡事故発生
2012/04 京都・東山区 7人死亡9人重軽傷     (数日前に発作、姉と相談)

日本てんかん協会 (共産党の支援団体)
住所 東京都新宿区西早稲田2-2-8

>>権利を守れ
>>西早稲田2-2-8    差別利権で食ってる団体だな
619132人目の素数さん:2012/04/28(土) 13:58:17.62
>>614
0=-fB+fB
620132人目の素数さん:2012/04/28(土) 14:42:19.11
          __ノ)-'´ ̄ ̄`ー- 、_
        , '´  _. -‐'''"二ニニ=-`ヽ、
      /   /:::::; -‐''"        `ーノ
     /   /:::::/           \
     /    /::::::/          | | |  |
     |   |:::::/ /     |  | | | |  |
      |   |::/ / / |  | ||  | | ,ハ .| ,ハ|
      |   |/ / / /| ,ハノ| /|ノレ,ニ|ル' 
     |   |  | / / レ',二、レ′ ,ィイ|゙/   私は只の数ヲタなんかとは付き合わないわ。
.     |   \ ∠イ  ,イイ|    ,`-' |      頭が良くて数学が出来てかっこいい人。それが必要条件よ。
     |     l^,人|  ` `-'     ゝ  |        さらに Ann.of Math に論文書けば十分条件にもなるわよ。
      |      ` -'\       ー'  人          一番嫌いなのは論文数を増やすためにくだらない論文を書いて
    |        /(l     __/  ヽ、           良い論文の出版を遅らせるお馬鹿な人。
     |       (:::::`‐-、__  |::::`、     ヒニニヽ、         あなたの論文が Ann of Math に accept される確率は?
    |      / `‐-、::::::::::`‐-、::::\   /,ニニ、\            それとも最近は Inv. Math. の方が上かしら?
   |      |::::::::::::::::::|` -、:::::::,ヘ ̄|'、  ヒニ二、 \
.   |      /::::::::::::::::::|::::::::\/:::O`、::\   | '、   \
   |      /:::::::::::::::::::/:::::::::::::::::::::::::::::'、::::\ノ  ヽ、  |
  |      |:::::/:::::::::/:::::::::::::::::::::::::::::::::::'、',::::'、  /:\__/‐、
  |      |/:::::::::::/::::::::::::::::::::::::::::::::::O::| '、::| く::::::::::::: ̄|
   |     /_..-'´ ̄`ー-、:::::::::::::::::::::::::::::::::::|/:/`‐'::\;;;;;;;_|
   |    |/::::::::::::::::::::::\:::::::::::::::::::::::::::::|::/::::|::::/:::::::::::/
    |   /:::::::::::::::::::::::::::::::::|:::::::::::::::::::::O::|::|::::::|:::::::::::::::/
621132人目の素数さん:2012/04/28(土) 14:50:39.35
中学のときに重度のてんかん持ちの同級生がいて常に親と付き添いで登校していました。
そんなとき登校中に一度だけてんかんの発作が起きているのを目撃しましたことがありますが何もしてあげられませんでした。
622132人目の素数さん:2012/04/28(土) 15:54:30.49
「z=x^2+y^2とz=-(x-2)^2-(y-4)^2+16で囲まれた領域の体積を積分で求めろ」

上記の問題ですが、図を書いてみたらz=-(x-2)^2-(y-4)^2+16が上になることがわかったので
このことよりzから積分をはじめ、
∫[x^2+y^2≦z≦-(x-2)^2-(y-4)^2+16]dz∫[2-√(x^2-2x+2)≦y≦2+√(x^2-2x+2)]dy∫[1-√3≦x≦1+√3]dx
と積分範囲を指定しました。
これで合っていますか?
またこれを計算するにあたって3次元極座標を用いたほうがよいなど簡単にする方法はありますか?
623132人目の素数さん:2012/04/28(土) 16:16:20.14
>>622
3次元極座標ってのは球面を考えた座標系なのだから
今回の回転放物面には合わないだろう。
やるとしても円筒座標。

それで、定石通り z = kでの切断面は
2つの円の共通部分なのだから
それで計算できて、zで積分するといったところだろう。
624132人目の素数さん:2012/04/28(土) 17:52:12.28
>>609
>>577>>608が全く通じてないな。両者が言ってるのは
>[Fe(CN)6]3-:x+(−1)×6=−3
の「[Fe(CN)6]3-:」の部分はそもそも式の一部ではなく
「[Fe(CN)6]3-について立式すると、」くらいの意味で、このコロン記号は
比を表す数学記号ではなく文中の句読点の一種だということ。
「476 :あるケミストさん」は何かの比を表す数式ではないでしょ?
625132人目の素数さん:2012/04/28(土) 19:19:15.64
>>615
お好みのホモトピー不変量を計算してみる
626132人目の素数さん:2012/04/28(土) 19:24:57.28
>>623
ではx,y,z座標でそのまま計算しても大丈夫なようですね。
積分の順序によって計算結果は異なりませんが、
私のやったように、まずzに関して積分し、
あるzの値kにおいて二つの円の重なっているところを計算すればいいんですよね。
627576 578 605 609:2012/04/28(土) 21:00:17.77
すいません。ありがとうございます。

>>578は、
×いや、その通りですよ。
○はい、その通りですよ。
です。
628132人目の素数さん:2012/04/28(土) 21:22:53.04
>>627
同じ意味だろ、それ。
629132人目の素数さん:2012/04/28(土) 21:25:18.74
>>627
そんな言葉はどうでもいいよ。意味同じじゃん。
で、そうすると、
> 左辺だけ、何:何になっている
ってのは一体何のことを言っているんだ?
630132人目の素数さん:2012/04/28(土) 21:26:32.89
だれもマルチと指摘しなかったな
631132人目の素数さん:2012/04/28(土) 23:00:59.65
>>629

k3[Fe(CN)6]→3K++ [Fe(CN)6]3-とバラバラにして、この
Feのイオンの価数をxとすると、CN-は−1なので、
[Fe(CN)6]3-:x+(−1)×6=−3 x=+3

です。この「:」は「対」ではないことはすでにわかっている状況です。
632132人目の素数さん:2012/04/28(土) 23:09:24.41
          __ノ)-'´ ̄ ̄`ー- 、_
        , '´  _. -‐'''"二ニニ=-`ヽ、
      /   /:::::; -‐''"        `ーノ
     /   /:::::/           \
     /    /::::::/          | | |  |
     |   |:::::/ /     |  | | | |  |
      |   |::/ / / |  | ||  | | ,ハ .| ,ハ|
      |   |/ / / /| ,ハノ| /|ノレ,ニ|ル' 
     |   |  | / / レ',二、レ′ ,ィイ|゙/   私は只の数ヲタなんかとは付き合わないわ。
.     |   \ ∠イ  ,イイ|    ,`-' |      頭が良くて数学が出来てかっこいい人。それが必要条件よ。
     |     l^,人|  ` `-'     ゝ  |        さらに Ann.of Math に論文書けば十分条件にもなるわよ。
      |      ` -'\       ー'  人          一番嫌いなのは論文数を増やすためにくだらない論文を書いて
    |        /(l     __/  ヽ、           良い論文の出版を遅らせるお馬鹿な人。
     |       (:::::`‐-、__  |::::`、     ヒニニヽ、         あなたの論文が Ann of Math に accept される確率は?
    |      / `‐-、::::::::::`‐-、::::\   /,ニニ、\            それとも最近は Inv. Math. の方が上かしら?
   |      |::::::::::::::::::|` -、:::::::,ヘ ̄|'、  ヒニ二、 \
.   |      /::::::::::::::::::|::::::::\/:::O`、::\   | '、   \
   |      /:::::::::::::::::::/:::::::::::::::::::::::::::::'、::::\ノ  ヽ、  |
  |      |:::::/:::::::::/:::::::::::::::::::::::::::::::::::'、',::::'、  /:\__/‐、
  |      |/:::::::::::/::::::::::::::::::::::::::::::::::O::| '、::| く::::::::::::: ̄|
   |     /_..-'´ ̄`ー-、:::::::::::::::::::::::::::::::::::|/:/`‐'::\;;;;;;;_|
   |    |/::::::::::::::::::::::\:::::::::::::::::::::::::::::|::/::::|::::/:::::::::::/
    |   /:::::::::::::::::::::::::::::::::|:::::::::::::::::::::O::|::|::::::|:::::::::::::::/
633132人目の素数さん:2012/04/28(土) 23:26:09.23
>>631
>>605のコロンが比を表すものではないということが分かったのならもう疑問はないってことでいいのか?
634132人目の素数さん:2012/04/28(土) 23:28:59.44
生きることの解の公式を教えて下さい
635132人目の素数さん:2012/04/28(土) 23:36:03.61
次の場合の時の、{f(a+h)-f(a)}/hを求めよ
1. a≠2かつa+h≠2のとき、f(x)=1/(x-2)
2. a≧4かつa+h≧4のとき、f(x)=√(x-4)
3. a≠-1かつa+h≠-1のとき、f(x)=x/(x+1)
分からないのでどうかお願いします
636132人目の素数さん:2012/04/29(日) 00:13:14.46
a^2=(bsin(θ))^2
をb=の形にする方法を知りたいのですが、
下記の計算であっているでしょうか?

a^2=b^2*sin^2(θ)
b^2=(a^2) / (sin^2(θ) )
 b=√( (a^2) / (sin^2(θ) )

回答おねがいします。
637132人目の素数さん:2012/04/29(日) 00:15:28.36
ルートはもうとっぱらってもいいんじゃないかな
二乗と一緒に
638132人目の素数さん:2012/04/29(日) 00:32:20.53
>>637
確かにそうですね・・・
気がつきませんでした。
ありがとうございます。
639132人目の素数さん:2012/04/29(日) 01:24:15.07
>>626
z 方向への積分は最後だぞ。
640132人目の素数さん:2012/04/29(日) 01:31:28.72
なんでっ?
641132人目の素数さん:2012/04/29(日) 01:36:17.69
>>638
符号に注意
642132人目の素数さん:2012/04/29(日) 01:36:32.80
そう決まってるから
643132人目の素数さん:2012/04/29(日) 01:41:45.30
F(a+b+h) = F(a) + F(b) + F(h)
644132人目の素数さん:2012/04/29(日) 01:42:36.92
>>643
は?
645132人目の素数さん:2012/04/29(日) 02:02:27.73
>>639
よろしければ、それぞれの積分範囲を記載していただいても大丈夫ですか?
646132人目の素数さん:2012/04/29(日) 04:16:10.01
>>633
いえ、−3=ー3/1=ー3:1というにことになるのかどうかです。
647132人目の素数さん:2012/04/29(日) 04:38:43.02
ならない
648132人目の素数さん:2012/04/29(日) 08:12:56.96
          __ノ)-'´ ̄ ̄`ー- 、_
        , '´  _. -‐'''"二ニニ=-`ヽ、
      /   /:::::; -‐''"        `ーノ
     /   /:::::/           \
     /    /::::::/          | | |  |
     |   |:::::/ /     |  | | | |  |
      |   |::/ / / |  | ||  | | ,ハ .| ,ハ|
      |   |/ / / /| ,ハノ| /|ノレ,ニ|ル' 
     |   |  | / / レ',二、レ′ ,ィイ|゙/   私は只の数ヲタなんかとは付き合わないわ。
.     |   \ ∠イ  ,イイ|    ,`-' |      頭が良くて数学が出来てかっこいい人。それが必要条件よ。
     |     l^,人|  ` `-'     ゝ  |        さらに Ann.of Math に論文書けば十分条件にもなるわよ。
      |      ` -'\       ー'  人          一番嫌いなのは論文数を増やすためにくだらない論文を書いて
    |        /(l     __/  ヽ、           良い論文の出版を遅らせるお馬鹿な人。
     |       (:::::`‐-、__  |::::`、     ヒニニヽ、         あなたの論文が Ann of Math に accept される確率は?
    |      / `‐-、::::::::::`‐-、::::\   /,ニニ、\            それとも最近は Inv. Math. の方が上かしら?
   |      |::::::::::::::::::|` -、:::::::,ヘ ̄|'、  ヒニ二、 \
.   |      /::::::::::::::::::|::::::::\/:::O`、::\   | '、   \
   |      /:::::::::::::::::::/:::::::::::::::::::::::::::::'、::::\ノ  ヽ、  |
  |      |:::::/:::::::::/:::::::::::::::::::::::::::::::::::'、',::::'、  /:\__/‐、
  |      |/:::::::::::/::::::::::::::::::::::::::::::::::O::| '、::| く::::::::::::: ̄|
   |     /_..-'´ ̄`ー-、:::::::::::::::::::::::::::::::::::|/:/`‐'::\;;;;;;;_|
   |    |/::::::::::::::::::::::\:::::::::::::::::::::::::::::|::/::::|::::/:::::::::::/
    |   /:::::::::::::::::::::::::::::::::|:::::::::::::::::::::O::|::|::::::|:::::::::::::::/
649132人目の素数さん:2012/04/29(日) 08:21:39.22
>>631
全く意味がわからない。
「:」が単なるコロンだと理解しているのなら、
> 左辺だけ、何:何になっている
は何を言ってるんだ?って聞いているのだが。
その式の左辺は「x+(−1)×6」だぞ。一体左辺のどこに「:」があるんだよ。

>>646
それはおまえが勝手に作った式だろう?
> 476 :あるケミストさん:2012/03/24(土) 20:49:23.84
> k3[Fe(CN)6]→3K++ [Fe(CN)6]3-とバラバラにして、この
> Feのイオンの価数をxとすると、CN-は−1なので、
> [Fe(CN)6]3-:x+(−1)×6=−3 x=+3
は関係ないじゃないか。
650132人目の素数さん:2012/04/29(日) 13:07:17.08
イミフ
説明へたくそ杉ここの回答者は
651132人目の素数さん:2012/04/29(日) 13:15:53.26
↑なにコイツw
652132人目の素数さん:2012/04/29(日) 13:16:40.59
質問スレはここですか?

「正定値対称行列[[A,B],[tB,C]]に対し,|[A,B],[tB,C]|≦|A||C|を示せ」
よろしくお願いします。
653132人目の素数さん:2012/04/29(日) 13:59:08.34
ちょっと記号の意味がよくわかんない
654652:2012/04/29(日) 14:04:05.92
ごめんなさい。
[[A,B],[tB,C]]は左上がA,右上がB,左下がtB(Bの転置),右下がCのブロック行列です。
AとCは正方行列です。
|A|は行列式det(A)です。

数学板って以前は記号の書き方の例がどこかにあった気がするのですが,今は無いんですか?
655132人目の素数さん:2012/04/29(日) 14:10:42.66
いつも「高校生のための数学の質問スレ」のテンプレで確認してる>記号の書き方
他にもあるのかもしれない
656132人目の素数さん:2012/04/29(日) 14:14:05.37
657132人目の素数さん:2012/04/29(日) 14:46:14.59
        , '´  _. -‐'''"二ニニ=-`ヽ、
      /   /:::::; -‐''"        `ーノ
     /   /:::::/           \
     /    /::::::/          | | |  |
     |   |:::::/ /     |  | | | |  |
      |   |::/ / / |  | ||  | | ,ハ .| ,ハ|
      |   |/ / / /| ,ハノ| /|ノレ,ニ|ル' 
     |   |  | / / レ',二、レ′ ,ィイ|゙/   私は只の数ヲタなんかとは付き合わないわ。
.     |   \ ∠イ  ,イイ|    ,`-' |      頭が良くて数学が出来てかっこいい人。それが必要条件よ。
     |     l^,人|  ` `-'     ゝ  |        さらに Ann.of Math に論文書けば十分条件にもなるわよ。
      |      ` -'\       ー'  人          一番嫌いなのは論文数を増やすためにくだらない論文を書いて
    |        /(l     __/  ヽ、           良い論文の出版を遅らせるお馬鹿な人。
     |       (:::::`‐-、__  |::::`、     ヒニニヽ、         あなたの論文が Ann of Math に accept される確率は?
    |      / `‐-、::::::::::`‐-、::::\   /,ニニ、\            それとも最近は Inv. Math. の方が上かしら?
   |      |::::::::::::::::::|` -、:::::::,ヘ ̄|'、  ヒニ二、 \
.   |      /::::::::::::::::::|::::::::\/:::O`、::\   | '、   \
   |      /:::::::::::::::::::/:::::::::::::::::::::::::::::'、::::\ノ  ヽ、  |
  |      |:::::/:::::::::/:::::::::::::::::::::::::::::::::::'、',::::'、  /:\__/‐、
  |      |/:::::::::::/::::::::::::::::::::::::::::::::::O::| '、::| く::::::::::::: ̄|
   |     /_..-'´ ̄`ー-、:::::::::::::::::::::::::::::::::::|/:/`‐'::\;;;;;;;_|
   |    |/::::::::::::::::::::::\:::::::::::::::::::::::::::::|::/::::|::::/:::::::::::/
    |   /:::::::::::::::::::::::::::::::::|:::::::::::::::::::::O::|::|::::::|:::::::::::::::/
658132人目の素数さん:2012/04/29(日) 15:14:58.23
1 名無しさんにズームイン! [] Date:2012/03/28(水) 08:28:15.02 ?ID:NWYs/2ZP Be:
 やらなけゃいけない
 電○の各局への圧力が半端ないんです
 昨日、一昨日前田AKB卒業ネタやった情報番組全てが前田AKB卒業ネタ中の毎分で視聴率がダダ下がりしました。
 各局本音では毎分視聴率ダダ下がりするこのネタははやりたくなかったけど原子力村以上に電○からの圧力が凄いんです

ブーム捏造、枕営業、自社買い、サクラの動員そして
AKBの捏造ブームのために税金が大量に使われている証拠がこちら

やっと気付いた「AKBに電通が絡んでる」ではなく「AKBの正体が電通」な件 その127
http://hayabusa3.2ch.net/test/read.cgi/mor●ningcoffee/1335468718/

AKBの宣伝に税金を使い、その税金が民主党に流れている
テレビの捏造に騙されるな
659132人目の素数さん:2012/04/29(日) 16:24:32.17
次の問題はおかしいと思いますがどうでしょうか?

2曲線C1:y=e^(x+α) +β ,C2: y=logxについて点(1, 0)における
C2の接線がC1にも接している。

(1) βをαで表せ
(2) C1,C2に共通する接線をすべて求めαを求めよ。

(1)の答は β=-α-2 となりますが
C1,C2に共通する接線は
C2のx=1における接線y=x-1と  C1のx=1における接線y=e^(1+α) x-α-2
となってαの値は出ないと思えるのですが,どうなんでしょう?
660132人目の素数さん:2012/04/29(日) 17:18:25.42
α(x)のような関数
661すうさん:2012/04/29(日) 19:01:45.98
外径D₀=350o、内径D=300oの短い円筒にP=100kNの物体を乗せるときに
生じる応力σはいくつですか?
662132人目の素数さん:2012/04/29(日) 19:38:53.59
          __ノ)-'´ ̄ ̄`ー- 、_
        , '´  _. -‐'''"二ニニ=-`ヽ、
      /   /:::::; -‐''"        `ーノ
     /   /:::::/           \
     /    /::::::/          | | |  |
     |   |:::::/ /     |  | | | |  |
      |   |::/ / / |  | ||  | | ,ハ .| ,ハ|
      |   |/ / / /| ,ハノ| /|ノレ,ニ|ル' 
     |   |  | / / レ',二、レ′ ,ィイ|゙/   私は只の数ヲタなんかとは付き合わないわ。
.     |   \ ∠イ  ,イイ|    ,`-' |      頭が良くて数学が出来てかっこいい人。それが必要条件よ。
     |     l^,人|  ` `-'     ゝ  |        さらに Ann.of Math に論文書けば十分条件にもなるわよ。
      |      ` -'\       ー'  人          一番嫌いなのは論文数を増やすためにくだらない論文を書いて
    |        /(l     __/  ヽ、           良い論文の出版を遅らせるお馬鹿な人。
     |       (:::::`‐-、__  |::::`、     ヒニニヽ、         あなたの論文が Ann of Math に accept される確率は?
    |      / `‐-、::::::::::`‐-、::::\   /,ニニ、\            それとも最近は Inv. Math. の方が上かしら?
   |      |::::::::::::::::::|` -、:::::::,ヘ ̄|'、  ヒニ二、 \
.   |      /::::::::::::::::::|::::::::\/:::O`、::\   | '、   \
   |      /:::::::::::::::::::/:::::::::::::::::::::::::::::'、::::\ノ  ヽ、  |
  |      |:::::/:::::::::/:::::::::::::::::::::::::::::::::::'、',::::'、  /:\__/‐、
  |      |/:::::::::::/::::::::::::::::::::::::::::::::::O::| '、::| く::::::::::::: ̄|
   |     /_..-'´ ̄`ー-、:::::::::::::::::::::::::::::::::::|/:/`‐'::\;;;;;;;_|
   |    |/::::::::::::::::::::::\:::::::::::::::::::::::::::::|::/::::|::::/:::::::::::/
    |   /:::::::::::::::::::::::::::::::::|:::::::::::::::::::::O::|::|::::::|:::::::::::::::/
663132人目の素数さん:2012/04/29(日) 21:00:00.11
>>661
ぶ つ り い た で き け
664132人目の素数さん:2012/04/29(日) 22:23:50.87
ゲーデルの不完全性定理
http://ja.wikipedia.org/wiki/%E3%82%B2%E3%83%BC%E3%83%87%E3%83%AB%E3%81%AE%E4%B8%8D%E5%AE%8C%E5%85%A8%E6%80%A7%E5%AE%9A%E7%90%86#.E6.A6.82.E8.A6.81


の文中に出てくる、Gが証明可能であれば、のところの、
記号「¬」の意味がわかりません。

記号の意味を教えてください。
665132人目の素数さん:2012/04/29(日) 22:31:24.67
どうせwikiの解説読んだって誤解するだけなので、気にする必要ありません
(その記事書いたのは明らかにシロートと思われるし)
666132人目の素数さん:2012/04/29(日) 23:21:14.81
          __ノ)-'´ ̄ ̄`ー- 、_
        , '´  _. -‐'''"二ニニ=-`ヽ、
      /   /:::::; -‐''"        `ーノ
     /   /:::::/           \
     /    /::::::/          | | |  |
     |   |:::::/ /     |  | | | |  |
      |   |::/ / / |  | ||  | | ,ハ .| ,ハ|
      |   |/ / / /| ,ハノ| /|ノレ,ニ|ル' 
     |   |  | / / レ',二、レ′ ,ィイ|゙/   私は只の数ヲタなんかとは付き合わないわ。
.     |   \ ∠イ  ,イイ|    ,`-' |      頭が良くて数学が出来てかっこいい人。それが必要条件よ。
     |     l^,人|  ` `-'     ゝ  |        さらに Ann.of Math に論文書けば十分条件にもなるわよ。
      |      ` -'\       ー'  人          一番嫌いなのは論文数を増やすためにくだらない論文を書いて
    |        /(l     __/  ヽ、           良い論文の出版を遅らせるお馬鹿な人。
     |       (:::::`‐-、__  |::::`、     ヒニニヽ、         あなたの論文が Ann of Math に accept される確率は?
    |      / `‐-、::::::::::`‐-、::::\   /,ニニ、\            それとも最近は Inv. Math. の方が上かしら?
   |      |::::::::::::::::::|` -、:::::::,ヘ ̄|'、  ヒニ二、 \
.   |      /::::::::::::::::::|::::::::\/:::O`、::\   | '、   \
   |      /:::::::::::::::::::/:::::::::::::::::::::::::::::'、::::\ノ  ヽ、  |
  |      |:::::/:::::::::/:::::::::::::::::::::::::::::::::::'、',::::'、  /:\__/‐、
  |      |/:::::::::::/::::::::::::::::::::::::::::::::::O::| '、::| く::::::::::::: ̄|
   |     /_..-'´ ̄`ー-、:::::::::::::::::::::::::::::::::::|/:/`‐'::\;;;;;;;_|
   |    |/::::::::::::::::::::::\:::::::::::::::::::::::::::::|::/::::|::::/:::::::::::/
    |   /:::::::::::::::::::::::::::::::::|:::::::::::::::::::::O::|::|::::::|:::::::::::::::/
667132人目の素数さん:2012/04/29(日) 23:38:08.68
≫664
「引っ掛け」ですね。
668132人目の素数さん:2012/04/29(日) 23:42:12.05
disるな けなすな 否定するな
否定から始まる人間は全てをダメにする
669132人目の素数さん:2012/04/30(月) 00:11:20.05
どうして?
670132人目の素数さん:2012/04/30(月) 04:30:55.23
>>622 解けたかたいますか?
671132人目の素数さん:2012/04/30(月) 05:26:51.69
>>652
直感的イメージだけなら、
大きい行列を M として、A と C を単独で対角化して更にスケール変更すれば、A, C は単位行列としてもかまわない。
次にベクトル x に対する方程式 tx M x=1 を考えると、これは楕円体を表し体積は det(M) に比例する。
M が A,C=単位行列を対角成分に持つ行列なら、これは立方体に内接する楕円体で、球の時が体積最大となる。
球の時は M が単位行列になるから、B≠0 の場合は det(M) 最大じゃない。
これを厳密な証明にするのは自分でやってくれ。
672132人目の素数さん:2012/04/30(月) 10:46:58.83
代数幾何学は遊びじゃありませんか?
673132人目の素数さん:2012/04/30(月) 10:58:54.09
>>672
学問に対する信仰はあるか?
674132人目の素数さん:2012/04/30(月) 14:16:55.51
>>646
そうなるとしたら、3:0や1:2:3はどうなるの?
675132人目の素数さん:2012/04/30(月) 14:21:48.73
>>670
>>623の後段に従えばいいんじゃないの?
676132人目の素数さん:2012/04/30(月) 15:25:31.84
 やらなけゃいけない
 電○の各局への圧力が半端ないんです
 昨日、一昨日前田AKB卒業ネタやった情報番組全てが前田AKB卒業ネタ中の毎分で視聴率がダダ下がりしました。
 各局本音では毎分視聴率ダダ下がりするこのネタははやりたくなかったけど原子力村以上に電○からの圧力が凄いんです

AKBの宣伝に税金が使われ、その税金が民主党に流れている。
ブーム捏造、枕営業、自社買い、サクラの動員そして
AKBの捏造ブームのために税金が大量に使われている証拠がこちら

やっと気付いた「AKBに電通が絡んでる」ではなく「AKBの正体が電通」な件 その127
http://hayabusa3.2ch.net/test/read.cgi/morningcoffee/133∵5468718/     ∵を外して貼り付け

テレビのやらせブームに騙されるな
677132人目の素数さん:2012/04/30(月) 15:32:19.73
          __ノ)-'´ ̄ ̄`ー- 、_
        , '´  _. -‐'''"二ニニ=-`ヽ、
      /   /:::::; -‐''"        `ーノ
     /   /:::::/           \
     /    /::::::/          | | |  |
     |   |:::::/ /     |  | | | |  |
      |   |::/ / / |  | ||  | | ,ハ .| ,ハ|
      |   |/ / / /| ,ハノ| /|ノレ,ニ|ル' 
     |   |  | / / レ',二、レ′ ,ィイ|゙/   私は只の数ヲタなんかとは付き合わないわ。
.     |   \ ∠イ  ,イイ|    ,`-' |      頭が良くて数学が出来てかっこいい人。それが必要条件よ。
     |     l^,人|  ` `-'     ゝ  |        さらに Ann.of Math に論文書けば十分条件にもなるわよ。
      |      ` -'\       ー'  人          一番嫌いなのは論文数を増やすためにくだらない論文を書いて
    |        /(l     __/  ヽ、           良い論文の出版を遅らせるお馬鹿な人。
     |       (:::::`‐-、__  |::::`、     ヒニニヽ、         あなたの論文が Ann of Math に accept される確率は?
    |      / `‐-、::::::::::`‐-、::::\   /,ニニ、\            それとも最近は Inv. Math. の方が上かしら?
   |      |::::::::::::::::::|` -、:::::::,ヘ ̄|'、  ヒニ二、 \
.   |      /::::::::::::::::::|::::::::\/:::O`、::\   | '、   \
   |      /:::::::::::::::::::/:::::::::::::::::::::::::::::'、::::\ノ  ヽ、  |
  |      |:::::/:::::::::/:::::::::::::::::::::::::::::::::::'、',::::'、  /:\__/‐、
  |      |/:::::::::::/::::::::::::::::::::::::::::::::::O::| '、::| く::::::::::::: ̄|
   |     /_..-'´ ̄`ー-、:::::::::::::::::::::::::::::::::::|/:/`‐'::\;;;;;;;_|
   |    |/::::::::::::::::::::::\:::::::::::::::::::::::::::::|::/::::|::::/:::::::::::/
    |   /:::::::::::::::::::::::::::::::::|:::::::::::::::::::::O::|::|::::::|:::::::::::::::/
678132人目の素数さん:2012/04/30(月) 15:36:20.02
おれもう40歳過ぎてんだけどさ 最近気になることがあるんだ
高校数学ぐらいしかわからんけどさ
微積分とかの解析学? と 空間図形とか扱う幾何学? って
おんなじことをやってるんじゃないの?
つまり、対応がある気がするんだけど やっぱ違うの?
679132人目の素数さん:2012/04/30(月) 16:42:52.52
対応はある
デカルト座標とか解析幾何とか
普通の高校生レベルの図形を解析するのが普通の方程式とかなら
もっと小難しい方程式は 現実にはあり得ない抽象的な図形に対応するかもしれない
680132人目の素数さん:2012/04/30(月) 18:22:19.04
結合法則 (ab)c=a(bc)が成り立つなら
a[1]a[2]…a[n]が括弧のつけ方によらず定まることは
どうやって証明するの?
681132人目の素数さん:2012/04/30(月) 18:25:30.02
数学的帰納法
682132人目の素数さん:2012/04/30(月) 18:26:28.70
帰納法でいけそうな気がするけど
683132人目の素数さん:2012/04/30(月) 18:46:50.64
          __ノ)-'´ ̄ ̄`ー- 、_
        , '´  _. -‐'''"二ニニ=-`ヽ、
      /   /:::::; -‐''"        `ーノ
     /   /:::::/           \
     /    /::::::/          | | |  |
     |   |:::::/ /     |  | | | |  |
      |   |::/ / / |  | ||  | | ,ハ .| ,ハ|
      |   |/ / / /| ,ハノ| /|ノレ,ニ|ル' 
     |   |  | / / レ',二、レ′ ,ィイ|゙/   私は只の数ヲタなんかとは付き合わないわ。
.     |   \ ∠イ  ,イイ|    ,`-' |      頭が良くて数学が出来てかっこいい人。それが必要条件よ。
     |     l^,人|  ` `-'     ゝ  |        さらに Ann.of Math に論文書けば十分条件にもなるわよ。
      |      ` -'\       ー'  人          一番嫌いなのは論文数を増やすためにくだらない論文を書いて
    |        /(l     __/  ヽ、           良い論文の出版を遅らせるお馬鹿な人。
     |       (:::::`‐-、__  |::::`、     ヒニニヽ、         あなたの論文が Ann of Math に accept される確率は?
    |      / `‐-、::::::::::`‐-、::::\   /,ニニ、\            それとも最近は Inv. Math. の方が上かしら?
   |      |::::::::::::::::::|` -、:::::::,ヘ ̄|'、  ヒニ二、 \
.   |      /::::::::::::::::::|::::::::\/:::O`、::\   | '、   \
   |      /:::::::::::::::::::/:::::::::::::::::::::::::::::'、::::\ノ  ヽ、  |
  |      |:::::/:::::::::/:::::::::::::::::::::::::::::::::::'、',::::'、  /:\__/‐、
  |      |/:::::::::::/::::::::::::::::::::::::::::::::::O::| '、::| く::::::::::::: ̄|
   |     /_..-'´ ̄`ー-、:::::::::::::::::::::::::::::::::::|/:/`‐'::\;;;;;;;_|
   |    |/::::::::::::::::::::::\:::::::::::::::::::::::::::::|::/::::|::::/:::::::::::/
    |   /:::::::::::::::::::::::::::::::::|:::::::::::::::::::::O::|::|::::::|:::::::::::::::/
684132人目の素数さん:2012/04/30(月) 18:48:08.42
          __ノ)-'´ ̄ ̄`ー- 、_
        , '´  _. -‐'''"二ニニ=-`ヽ、
      /   /:::::; -‐''"        `ーノ
     /   /:::::/           \
     /    /::::::/          | | |  |
     |   |:::::/ /     |  | | | |  |
      |   |::/ / / |  | ||  | | ,ハ .| ,ハ|
      |   |/ / / /| ,ハノ| /|ノレ,ニ|ル' 
     |   |  | / / レ',二、レ′ ,ィイ|゙/   私は只の数ヲタなんかとは付き合わないわ。
.     |   \ ∠イ  ,イイ|    ,`-' |      頭が良くて数学が出来てかっこいい人。それが必要条件よ。
     |     l^,人|  ` `-'     ゝ  |        さらに Ann.of Math に論文書けば十分条件にもなるわよ。
      |      ` -'\       ー'  人          一番嫌いなのは論文数を増やすためにくだらない論文を書いて
    |        /(l     __/  ヽ、           良い論文の出版を遅らせるお馬鹿な人。
     |       (:::::`‐-、__  |::::`、     ヒニニヽ、         あなたの論文が Ann of Math に accept される確率は?
    |      / `‐-、::::::::::`‐-、::::\   /,ニニ、\            それとも最近は Inv. Math. の方が上かしら?
   |      |::::::::::::::::::|` -、:::::::,ヘ ̄|'、  ヒニ二、 \
.   |      /::::::::::::::::::|::::::::\/:::O`、::\   | '、   \
   |      /:::::::::::::::::::/:::::::::::::::::::::::::::::'、::::\ノ  ヽ、  |
  |      |:::::/:::::::::/:::::::::::::::::::::::::::::::::::'、',::::'、  /:\__/‐、
  |      |/:::::::::::/::::::::::::::::::::::::::::::::::O::| '、::| く::::::::::::: ̄|
   |     /_..-'´ ̄`ー-、:::::::::::::::::::::::::::::::::::|/:/`‐'::\;;;;;;;_|
   |    |/::::::::::::::::::::::\:::::::::::::::::::::::::::::|::/::::|::::/:::::::::::/
    |   /:::::::::::::::::::::::::::::::::|:::::::::::::::::::::O::|::|::::::|:::::::::::::::/
685132人目の素数さん:2012/04/30(月) 21:42:38.02
a^3+6ab-8b^3+1を因数分解するとどうなりますか?
686132人目の素数さん:2012/04/30(月) 21:45:29.92
>>685
x^3+y^3+z^3-3xyz の応用。
687132人目の素数さん:2012/04/30(月) 21:51:41.52
>>686
どう応用すればいいのかわかりません・・・
688132人目の素数さん:2012/04/30(月) 22:02:28.31
>>685
マルチ
689132人目の素数さん:2012/04/30(月) 23:37:29.47
某国政府著「民間防衛」より転載


某国元首:費用のかからない方法で敵(国)を滅ぼすことができる。
魅力でひきつける宣伝は効果的な武器だ。我々の意図を美しい装飾で包み隠そう。
文化は立派な隠れ蓑になる。音楽・芸術・旅行などの口実で仲間をつくり、一方的な文化交流(聞こえは良いが実際は押し付け→韓流)をしよう。
彼らは徐々に罠にはまっていく。


韓流、AKB商法の正体
もちろん、韓流、AKBは捏造ブームである。

ヨン様ファン?
もちろん在日ババアの動員ですよ。
テレビは愚民の思考を止めるために存在し、そのためにテレビ業界が役人、政治家により手厚く保護されていることを忘れないでください。
690132人目の素数さん:2012/05/01(火) 00:27:34.15
>>681-682
どうやって帰納法使ったらいいのか分からない
元がn個かけられてる時に、カッコのつけ方によらず
全部n-1個以下の積に帰着できることをどうやって示すの?
691132人目の素数さん:2012/05/01(火) 00:34:49.30
>>690
最も外側の結合がk番目にあるとする。すなわち
(a[1]…a[k]) ・ (a[k+1]…a[n])
の形である。帰納法の仮定より、これの左側・右側それぞれは括弧の付け方に依らないから、
(((a[1]a[2])a[3])…a[k]) ・ (a[k+1]…(a[n-2](a[n-1]a[n])))
としてよい。あとは (ab)c = a(bc) を順々に適用すると
(((a[1]a[2])a[3])…a[n])
の形に変形できる。
692132人目の素数さん:2012/05/01(火) 00:43:25.26
>>691
これは明快ですね
ありがとうございます
693132人目の素数さん:2012/05/01(火) 00:48:08.87
関数とは比例と同じ意味ですか?
694132人目の素数さん:2012/05/01(火) 00:49:34.08
>>693
No.
695132人目の素数さん:2012/05/01(火) 00:49:43.80
fish!
696132人目の素数さん:2012/05/01(火) 00:56:47.93
xy^2+y+1-x の因数分解のやり方誰か教えてください
697132人目の素数さん:2012/05/01(火) 01:10:01.12
マルチするんじゃないですよアホ野郎、
てな具合に優しく教えてもらったことが無いのですか?
698132人目の素数さん:2012/05/01(火) 01:13:55.89
(y-(-1+|2x-1|)/2x)(y-(-1-|2x-1|)/2x)
699132人目の素数さん:2012/05/01(火) 01:14:20.96
俺も両方のスレ見てるからなあ、丸わかり
700132人目の素数さん:2012/05/01(火) 01:19:16.29
>>697
すいません。向こうに書いた後こっちのスレに書くのが良かったのかなと思いまして
マルチしてしまいました。
701132人目の素数さん:2012/05/01(火) 01:19:32.19
∞!=?
702132人目の素数さん:2012/05/01(火) 01:21:00.03
>>696
解の公式は知ってるか?
703132人目の素数さん:2012/05/01(火) 01:21:05.37
どうして俺に彼女ができないのか数学的に教えろクズども
704132人目の素数さん:2012/05/01(火) 01:24:31.31
>>700
じゃあ向こうは取り下げてこい
705132人目の素数さん:2012/05/01(火) 02:12:04.23
>>703
君の彼女になりえる女性の集合をXとする
君の顔性格体型雰囲気を考慮するとXは空集合であると言える。
つまり君が変わらない限り未来永劫彼女はできない。
706132人目の素数さん:2012/05/01(火) 02:25:32.94
>>703
1. 君の彼女になり得る女性の集合をXとする。
2. Xに元があると仮定して矛盾を導く。
707132人目の素数さん:2012/05/01(火) 04:01:05.78
708132人目の素数さん:2012/05/01(火) 04:02:33.56
709132人目の素数さん:2012/05/01(火) 04:14:18.11
>>703
彼女が出来ない男の集合をΩとおく。
人間xに対し、x∈Ωであるための十分条件の集合をΣとおく。
xがお前を表すとき、∃C∈Σ such that <xはCを満たす> が証明できればx∈Ωである。
710132人目の素数さん:2012/05/01(火) 04:26:54.30
Σの元としては次のようなものが知られている
(a)ハゲ∨デブ
(b)インポテンツ∨ホモ
(c)腋臭∧チビ

さあ、キミが知っているΣの元を掲げてくれ。
711片山博文MZ ◆0lBZNi.Q7evd :2012/05/01(火) 13:26:55.44
Sを3個以上の元を持つ集合とする。
Sから互いに異なるある3個の元x,y,zを選ぶと、任意のa,b∈Sについて、
((a ≠ x) ∧ (b ≠ x)) ∨ ((a ≠ y) ∧ (b ≠ y)) ∨ ((a ≠ z) ∧ (b ≠ z))
が常に成り立つことの証明か判例を示せ。
この問題を最初に解いた人にWebMoney2000ptあげるよ(トリップつけてね)。
712 ◆4xEIYAYq9c :2012/05/01(火) 14:03:04.21
>>711
x, y, z, a, bがすべて異なるときは成り立つのは明らか
同じものが1つのときでは
x=aのとき
y≠b, z≠b
このときY、zはxと異なるのでy≠a, z≠a
よって成り立つ
x=bも同様
y=a
y=b
z=a
z=b
も同じようにして示せる
同じとなるものが2つのときでは
y=a, z=bのとき
xはy, zと異なるので(a ≠ x) ∧ (b ≠ x)
y=b, z=aのときも同様
x=a, y=b
x=b, y=a
x=a, z=b
x=b, z=aも同様に示せる
713片山博文MZ ◆0lBZNi.Q7evd :2012/05/01(火) 14:32:36.94
>>712 正解。対称性に関する言及が欲しかった。
メールアドレスを教えてください。後日WebMoneyを送ります。
714132人目の素数さん:2012/05/01(火) 14:40:15.52
すごい基本的なこと聞くけど誰か答えてorz
cosの求め方って、aA+bA−cA/2ab みたいなやつ以外にあるの?
√なんかA−なんかA ってやって求められるの?
715 ◆4xEIYAYq9c :2012/05/01(火) 14:41:23.85
716132人目の素数さん:2012/05/01(火) 14:42:52.11
なんか/(√(なんかA+なんかA))で求まるよ
717132人目の素数さん:2012/05/01(火) 14:42:54.64
>>714
sinを求める
ベクトルの内積を利用する
三角形の面積から考える
718132人目の素数さん:2012/05/01(火) 14:50:37.12
sinから求めてるっぽいんだけど…
問題→http://iup.2ch-library.com/i/i0623004-1335851252.jpg
解答→http://iup.2ch-library.com/i/i0623005-1335851252.jpg
見れるかな
719132人目の素数さん:2012/05/01(火) 15:09:52.27
例えばとして

(m+2)(m-2)≧0

の定数mを求める場合

m≦-2,2≦m

みたいにして書くのはどうしてですか?

m+2≧0
m≧-2

m-2≧0
m≧2

みたいな書き方ではダメなんですか?
初歩的な質問ですみません・・
720132人目の素数さん:2012/05/01(火) 15:14:02.45
>>718
画像小さくね?
>>719
別にいいよ
721132人目の素数さん:2012/05/01(火) 15:14:56.74
>m≦-2,2≦m
これは正確には m≦-2 または 2≦m の意。
そう了解する慣習がある。


貴方が
>m+2≧0
>m≧-2

>m-2≧0
>m≧2
これを一体どういう意味で書いたのか。
疑問の余地がある時点で好ましくないと思う。
722132人目の素数さん:2012/05/01(火) 15:22:08.26
>>720
アップローダー使ったことなくて
見れないか…
723片山博文MZ ◆0lBZNi.Q7evd :2012/05/01(火) 15:23:56.92
WebMoneyを送りました。
724132人目の素数さん:2012/05/01(火) 15:29:10.94
>>719
以下の(1)または(2)が成り立つときに、(m+2)(m-2)≧0が成り立つ
(1)m+2≧0かつm-2≧0 ⇒ m≧2
(2)m+2≦0かつm-2≦0 ⇒ m≦-2
725132人目の素数さん:2012/05/01(火) 15:35:05.20
>>722
アップローダーによっては自動で画像を縮小する
iPhoneならこれが便利
http://itunes.apple.com/jp/app/idotuploader/id474724266?mt=8
726132人目の素数さん:2012/05/01(火) 15:43:48.67
727132人目の素数さん:2012/05/01(火) 15:48:01.01
>>726
まず
sin^2θ+cos^2θ=1
という基本的な関係は大丈夫?
cosθ=±√(1-sin^2θ)
θが鋭角ならcosθは正
θが鈍角ならcosθは負
728 ◆4xEIYAYq9c :2012/05/01(火) 15:48:22.26
>>723
ありがとう
729132人目の素数さん:2012/05/01(火) 15:49:02.21
数列{2・4・6・8・10・12・16・18・24・・・・}と
フィボナッチ級数の関係について説明してください。

よろしくお願いします。
730132人目の素数さん:2012/05/01(火) 15:49:19.73
>>727
あ!それだ!
ありがとうございます解いてみます
731132人目の素数さん:2012/05/01(火) 15:52:56.38
R^nで、loglog(1+1/|x|)が原点中心で半径1の開球上でW^(1,n)に属することってどうやって証明したらいいですか??
W^(1,n)はソボレフ空間です。
どなたか教えてくださいお願いします。
732132人目の素数さん:2012/05/01(火) 16:06:50.66
>>721
もっとわかりやすく
733132人目の素数さん:2012/05/01(火) 19:39:19.28
2x^2+xy-y^2+10x+4y+12
=2x^2+x(y+10)-(y^2-4y-12)
=2x^2+x(y+10)-(y+2)(y-6)  ここまではわかるんだけど

={x+(y+2)}{2x-(y-6)} 次にこうなる所がよくわからないので教えてください
734132人目の素数さん:2012/05/01(火) 19:47:48.88
>>733
たすきがけ
735132人目の素数さん:2012/05/01(火) 20:38:55.04
それでつじつまが合うからとしかいいようがない
736132人目の素数さん:2012/05/01(火) 20:58:34.35
それを言っちゃスレ終了するだろ
737132人目の素数さん:2012/05/01(火) 21:06:26.93
>>734
たすきがけで調べたらわかりました^^
738132人目の素数さん:2012/05/01(火) 22:03:14.13
>>731
定義に沿ってノルムを計算
739132人目の素数さん:2012/05/01(火) 23:04:17.68
おいさっさと教えろ
レポート間に合わねぇじゃねぇかお願いしますよ
740132人目の素数さん:2012/05/01(火) 23:05:28.81
まっててね
741132人目の素数さん:2012/05/01(火) 23:10:44.32
残りゼロか

ふ〜
742132人目の素数さん:2012/05/01(火) 23:12:00.46
正三角形ABCにおいて、辺BC、CA、ABを3:(n-3)に内分する点をそれぞれD,E,Fとする(ただしn>6)。
線分AD,BE,CFの交点の作る三角形の面積が元の三角形の4/49のとき、nを求めろ。
743132人目の素数さん:2012/05/01(火) 23:18:18.04
マルチおっつ〜^^
744132人目の素数さん:2012/05/01(火) 23:20:01.98
ごめんね
急ぎだから
745132人目の素数さん:2012/05/01(火) 23:23:08.65
>>742
おいクソ
どっちか取り下げろよアホ
んで半年ROMれよカス
746132人目の素数さん:2012/05/01(火) 23:32:06.72
息ぬけよ
747132人目の素数さん:2012/05/01(火) 23:43:02.74
俺、高校生〜スレも巡回してるからなあ
他にも多いんじゃないの、複数スレ巡回してる、よく回答する人
>ごめんね
>急ぎだから
と言うなら同時に、回答募集するスレ指定を
添えればまだマシなんだが
748132人目の素数さん:2012/05/02(水) 00:46:30.81
>>742
君があちこちの質問スレに藁にも縋る思いで問題を書き込んでいるとき
数学ヲタ達もまたあちこちの質問スレにどんな質問が並んでいるかを見回っている。
だから、マルチは真っ先に発見されて、以後それへの回答は期待できなくなる。
749132人目の素数さん:2012/05/02(水) 00:48:25.98
          __ノ)-'´ ̄ ̄`ー- 、_
        , '´  _. -‐'''"二ニニ=-`ヽ、
      /   /:::::; -‐''"        `ーノ
     /   /:::::/           \
     /    /::::::/          | | |  |
     |   |:::::/ /     |  | | | |  |
      |   |::/ / / |  | ||  | | ,ハ .| ,ハ|
      |   |/ / / /| ,ハノ| /|ノレ,ニ|ル' 
     |   |  | / / レ',二、レ′ ,ィイ|゙/   私は只の数ヲタなんかとは付き合わないわ。
.     |   \ ∠イ  ,イイ|    ,`-' |      頭が良くて数学が出来てかっこいい人。それが必要条件よ。
     |     l^,人|  ` `-'     ゝ  |        さらに Ann.of Math に論文書けば十分条件にもなるわよ。
      |      ` -'\       ー'  人          一番嫌いなのは論文数を増やすためにくだらない論文を書いて
    |        /(l     __/  ヽ、           良い論文の出版を遅らせるお馬鹿な人。
     |       (:::::`‐-、__  |::::`、     ヒニニヽ、         あなたの論文が Ann of Math に accept される確率は?
    |      / `‐-、::::::::::`‐-、::::\   /,ニニ、\            それとも最近は Inv. Math. の方が上かしら?
   |      |::::::::::::::::::|` -、:::::::,ヘ ̄|'、  ヒニ二、 \
.   |      /::::::::::::::::::|::::::::\/:::O`、::\   | '、   \
   |      /:::::::::::::::::::/:::::::::::::::::::::::::::::'、::::\ノ  ヽ、  |
  |      |:::::/:::::::::/:::::::::::::::::::::::::::::::::::'、',::::'、  /:\__/‐、
  |      |/:::::::::::/::::::::::::::::::::::::::::::::::O::| '、::| く::::::::::::: ̄|
   |     /_..-'´ ̄`ー-、:::::::::::::::::::::::::::::::::::|/:/`‐'::\;;;;;;;_|
   |    |/::::::::::::::::::::::\:::::::::::::::::::::::::::::|::/::::|::::/:::::::::::/
    |   /:::::::::::::::::::::::::::::::::|:::::::::::::::::::::O::|::|::::::|:::::::::::::::/
750132人目の素数さん:2012/05/02(水) 01:52:59.45
GL(n,C)の位数が有限個の元は対角化可能であることを示せってどうやるんですか?
方針だけでもいいので教えてください
751132人目の素数さん:2012/05/02(水) 01:54:13.33
僕には分かりましぇん
752132人目の素数さん:2012/05/02(水) 05:18:57.97
a^m=0->a=a1e1+a2e2+...+anen
753132人目の素数さん:2012/05/02(水) 07:09:04.31
ベクトル空間V Uについて VからUへの線形変換が対角化可能というのはどういう定義ですか?
754132人目の素数さん:2012/05/02(水) 08:46:06.61
>>753
線形変換 f:V →U
は,VとUにそれぞれ基底(なんでもいい)を定めるとfの表現行列が対角行列になるとき
「fは対角化可能」と言う.

「行列が対角化可能」という概念を少しだけ一般化したものだと考えれば良い.

任意の正方行列はある線形変換R^n→R^nを表していると考えられる.この行列をAとし,Aが表す線形変換
R^n→R^nをaとするとき,

「行列Aが対角化可能」 ⇔ 「線形写像 a が対角化可能」

であることは良いか.これによって
「対角化可能な線形変換の例」
「対角化不可能な線形変換の例」
を,正方行列から容易に得られるであろう.すなわち,2×2実行列を例に取れば

「可能な例」は det(A) ≠0 かつ全ての固有値が実数である任意の行列から得られるし
「不可能な例」は固有値が実数とならない行列,例えば回転行列から得られる.
755132人目の素数さん:2012/05/02(水) 08:56:28.13
a x + b y
756132人目の素数さん:2012/05/02(水) 10:16:46.13
40-32÷2=?
小学生「4!」
理系「よくわかってんじゃん」
文系「やっぱわかんないか〜w」

そうなんだけど答え方としてどうかと思うぞ


ツイッターで流れてきましたが意味がわかりません
()がついてないのに答えは4になるんですか?
ちなみに発言者は塾で講師のバイトをしています
757132人目の素数さん:2012/05/02(水) 10:20:02.27
>>756
答えは4じゃなく4!
758132人目の素数さん:2012/05/02(水) 10:27:13.29
>>757
なるほど・・・軽い謎かけのようなものだったのか・・・
ありがとうございます
759132人目の素数さん:2012/05/02(水) 10:36:29.20
0行列も対角行列なんだけど。
760132人目の素数さん:2012/05/02(水) 12:49:09.38
>>757
4!ってどういう意味?
文系の俺には数字に!がつくとどう変わるのかわからないです。
761132人目の素数さん:2012/05/02(水) 12:53:51.00
ゴルゴ松本 「よんっ」
762132人目の素数さん:2012/05/02(水) 12:54:15.82
4が驚く
763132人目の素数さん:2012/05/02(水) 12:58:08.41
もしかして4の階乗ってことですか?
!は階乗の記号みたいですね
それなら24であってますね。
764132人目の素数さん:2012/05/02(水) 13:02:52.65
>>760
つまり、「11000」でもいいってこと
765132人目の素数さん:2012/05/02(水) 13:09:20.73
(2)てつけとけよ
766132人目の素数さん:2012/05/02(水) 13:48:11.35
今、リーマンの1859年の論文を読んでるんですが、数学素人なので、出だしから躓いてます。
∫[0,∞](e^(-nx))x^(s-1)dx=Π(s-1)/n^s
っていう等式が出てくるんですが、このΠ(s-1)って何です?
Πは総積記号ですよね?Π(s-1)ってどんな積なんですか?
PDFを見る限り、Π{(s-1)/n^s} とも違うようです。
767132人目の素数さん:2012/05/02(水) 14:09:27.44
> 元は階乗の一般化としてオイラーが得たもので、Γという記号は、ルジャンドルが用いたものである。
> それ以前はΠ(x)などと表記していた(ただしΠ(x)=Γ(x+1))。
768132人目の素数さん:2012/05/02(水) 14:36:18.70
おお、ガンマ関数のことでしたか、有難う
769132人目の素数さん:2012/05/02(水) 14:38:02.55
>>766
元ネタは
770132人目の素数さん:2012/05/02(水) 14:50:24.53
771132人目の素数さん:2012/05/02(水) 14:54:38.37
>>770
ありとん、しかしドイツ語がわからん
772132人目の素数さん:2012/05/02(水) 15:06:17.32
>>754
V=R^2 基底(1,0),(0,1)
U=R^2 基底(0,1),(-1,0)
f(x,y)=(-y,x)は対角化可能

R^2->R^2 f(x,y)=(-y,x) が対角化可能だから
2×2実行列A=
(0 -1)
(1 0)
は対角化可能
773132人目の素数さん:2012/05/02(水) 15:16:17.82
>>771
>>770は英訳
http://www.amazon.co.jp/dp/4535781818
に全文の和訳と解説があったと思う
774132人目の素数さん:2012/05/02(水) 15:51:14.41
>>773
これはうっかり八平
http://www.emis.de/classics/Riemann/
と勘違い
775132人目の素数さん:2012/05/02(水) 17:39:04.37
22 :可愛い奥様:2012/02/19(日) 20:10:14.60 ID:BAiCWuwH0

AKBを全面に出したバスが走ってて
http://www.akb-sg.net/akbus01.jpg

バスに書かれたリンクに行ったらAKBのページがあって
クールジャパンとは関係ないショップやカフェの広告が出てて
http://www.cooljapan.com.sg/akb48_live.html

クールジャパンとは関係ない現地のAKB企業HPへのリンクが貼ってあり
経済産業省の文字の真上で是非リンク先に訪れて下さいねと書いてある Please visit the official AKB48 Singapore website for the latest schedule of highlights!
www.akb48.com.sg

これのどこが日本の利益になるの?
税金使ってAKBと秋元康のバックにいる民主党への利益誘導をしてるだけでしょ
776132人目の素数さん:2012/05/02(水) 18:50:14.97
5個の玉を無作為に5個の箱に入れるとき箱が1つだけ空になる確立がわかりません
777132人目の素数さん:2012/05/02(水) 18:52:45.61
>>776
それぐらいなら、箱を玉に番号つけて区別して、総当たりで解けるんじゃないか?
778132人目の素数さん:2012/05/02(水) 19:08:07.35
>>777
全通りが5^5
空箱の選び方が5通り

1回目に4つの箱に玉を4つ入れて2回目に残った1つの玉をいれるってやり方を考えました。
2回目まで残る玉の選び方5通り
今選ばれなかった4つの玉を1回目の操作で4つの箱に入れる入れ方4!通り
2回目に玉を入れる箱の選び方4通り

で5*5*4!*4/5^5=96/125
ってだしたんですけどなんか間違っていそうなきがしまして
779132人目の素数さん:2012/05/02(水) 19:44:12.66
空箱以外の4つの箱に5個の玉を入れるのが4^5通り
780132人目の素数さん:2012/05/02(水) 19:57:45.29
【2012】クソゲーオブザイヤー part28【携帯】
http://toro.2ch.net/test/read.cgi/handygame/1335765313/
781132人目の素数さん:2012/05/02(水) 20:03:19.15
>>779
それだと1つの箱に5個入れる等の場合も数えているのではないでしょうか?
782132人目の素数さん:2012/05/02(水) 20:07:42.57
>>781
いいえ
783132人目の素数さん:2012/05/02(水) 20:09:47.07
>>778
・1回目に玉abcdを箱ABCDにそれぞれ入れて2回目に玉eを箱Aに入れる
・1回目に玉ebcdを箱ABCDにそれぞれ入れて2回目に玉aを箱Aに入れる
を重複して数えてる
784132人目の素数さん:2012/05/03(木) 01:12:39.18
a,b,cを(a-b)/cが自然数となる自然数として
       ↓階乗
( (a-b)/c ) ! = a - b/c

となるa,b,cの組を全てあげよ

例: a = 40 , b = 32 , c = 2
785132人目の素数さん:2012/05/03(木) 01:37:56.51
>>764
(40-32)÷2が4になるってとこが意味あるんじゃん。野暮上等。
786132人目の素数さん:2012/05/03(木) 01:38:02.11
>>784
a=b+1, c=1 の組が無限にあるが
787132人目の素数さん:2012/05/03(木) 02:14:09.46
>>785
間違った計算順序で出た数の階乗が
正しい計算順序の数になる

ってことじゃないの?
788132人目の素数さん:2012/05/03(木) 02:16:00.10
787は784へのレスかと思ったごめん

あと c > 1 で
789132人目の素数さん:2012/05/03(木) 02:16:32.23
ミス 787 は 785 が 784へのレスかと思った
790132人目の素数さん:2012/05/03(木) 03:00:00.01
(a−b)/c=x。
a−b/c=x!。
(c−1)a/c=x!−x。
(c−1)|(x!−x)。
a=(x!−x)c/(c−1)。
b=(x!−cx)c/(c−1)。
791132人目の素数さん:2012/05/03(木) 07:43:31.67
          __ノ)-'´ ̄ ̄`ー- 、_
        , '´  _. -‐'''"二ニニ=-`ヽ、
      /   /:::::; -‐''"        `ーノ
     /   /:::::/           \
     /    /::::::/          | | |  |
     |   |:::::/ /     |  | | | |  |
      |   |::/ / / |  | ||  | | ,ハ .| ,ハ|
      |   |/ / / /| ,ハノ| /|ノレ,ニ|ル' 
     |   |  | / / レ',二、レ′ ,ィイ|゙/   私は只の数ヲタなんかとは付き合わないわ。
.     |   \ ∠イ  ,イイ|    ,`-' |      頭が良くて数学が出来てかっこいい人。それが必要条件よ。
     |     l^,人|  ` `-'     ゝ  |        さらに Ann.of Math に論文書けば十分条件にもなるわよ。
      |      ` -'\       ー'  人          一番嫌いなのは論文数を増やすためにくだらない論文を書いて
    |        /(l     __/  ヽ、           良い論文の出版を遅らせるお馬鹿な人。
     |       (:::::`‐-、__  |::::`、     ヒニニヽ、         あなたの論文が Ann of Math に accept される確率は?
    |      / `‐-、::::::::::`‐-、::::\   /,ニニ、\            それとも最近は Inv. Math. の方が上かしら?
   |      |::::::::::::::::::|` -、:::::::,ヘ ̄|'、  ヒニ二、 \
.   |      /::::::::::::::::::|::::::::\/:::O`、::\   | '、   \
   |      /:::::::::::::::::::/:::::::::::::::::::::::::::::'、::::\ノ  ヽ、  |
  |      |:::::/:::::::::/:::::::::::::::::::::::::::::::::::'、',::::'、  /:\__/‐、
  |      |/:::::::::::/::::::::::::::::::::::::::::::::::O::| '、::| く::::::::::::: ̄|
   |     /_..-'´ ̄`ー-、:::::::::::::::::::::::::::::::::::|/:/`‐'::\;;;;;;;_|
   |    |/::::::::::::::::::::::\:::::::::::::::::::::::::::::|::/::::|::::/:::::::::::/
    |   /:::::::::::::::::::::::::::::::::|:::::::::::::::::::::O::|::|::::::|:::::::::::::::/
792132人目の素数さん:2012/05/03(木) 10:01:20.83
複2次式(x^4) -11(x^2)(y^2) + (y^4)を因数分解する際

答えでは((x^2)-(y^2))^2=(x^4) -2(x^2)(y^2) + (y^4)を使い
(与式) = ((x^2)-(y^2)+3xy)((x^2)+(y^2) -3xy)となっています。

僕は
((x^2)+(y^2))^2=(x^4) +2(x^2)(y^2) + (y^4)より
(与式)= ((x^2)+(y^2))^2 -13(x^2)(y^2)
(与式)= ((x^2)+(y^2))^2 -(√(13)xy)^2

(与式) = ((x^2)+(y^2)+(√(13)xy))((x^2)+(y^2) -(√(13)xy))
としたのですがこれではだめなんでしょうか?
793132人目の素数さん:2012/05/03(木) 10:12:55.94
>>772
R^nの標準基底についての言及が抜けていると素直に書けばよろしい。
794132人目の素数さん:2012/05/03(木) 10:25:13.29
>>793
いや違うし
795132人目の素数さん:2012/05/03(木) 10:30:35.50
>>792
君が正解
796132人目の素数さん:2012/05/03(木) 10:33:28.28
>>792
↑の答え符号違うと思うけどどっちも良いんじゃない
797132人目の素数さん:2012/05/03(木) 10:34:12.63
>>784
a<1000, b<1000, c<1000での(a,b,c)
(40,32,2), (230,220,2), (30,18,3), (952,928,4),
(25,5,5), (138,108,6), (833,791,7), (816,768,8),
(765,675,15), (756,648,18), (748,616,22), (735,525,35),
(731,473,43), (728,416,52), (721,103,103)
798132人目の素数さん:2012/05/03(木) 10:37:45.89
>>792
君のは不正解
799132人目の素数さん:2012/05/03(木) 10:43:25.82
          __ノ)-'´ ̄ ̄`ー- 、_
        , '´  _. -‐'''"二ニニ=-`ヽ、
      /   /:::::; -‐''"        `ーノ
     /   /:::::/           \
     /    /::::::/          | | |  |
     |   |:::::/ /     |  | | | |  |
      |   |::/ / / |  | ||  | | ,ハ .| ,ハ|
      |   |/ / / /| ,ハノ| /|ノレ,ニ|ル' 
     |   |  | / / レ',二、レ′ ,ィイ|゙/   私は只の数ヲタなんかとは付き合わないわ。
.     |   \ ∠イ  ,イイ|    ,`-' |      頭が良くて数学が出来てかっこいい人。それが必要条件よ。
     |     l^,人|  ` `-'     ゝ  |        さらに Ann.of Math に論文書けば十分条件にもなるわよ。
      |      ` -'\       ー'  人          一番嫌いなのは論文数を増やすためにくだらない論文を書いて
    |        /(l     __/  ヽ、           良い論文の出版を遅らせるお馬鹿な人。
     |       (:::::`‐-、__  |::::`、     ヒニニヽ、         あなたの論文が Ann of Math に accept される確率は?
    |      / `‐-、::::::::::`‐-、::::\   /,ニニ、\            それとも最近は Inv. Math. の方が上かしら?
   |      |::::::::::::::::::|` -、:::::::,ヘ ̄|'、  ヒニ二、 \
.   |      /::::::::::::::::::|::::::::\/:::O`、::\   | '、   \
   |      /:::::::::::::::::::/:::::::::::::::::::::::::::::'、::::\ノ  ヽ、  |
  |      |:::::/:::::::::/:::::::::::::::::::::::::::::::::::'、',::::'、  /:\__/‐、
  |      |/:::::::::::/::::::::::::::::::::::::::::::::::O::| '、::| く::::::::::::: ̄|
   |     /_..-'´ ̄`ー-、:::::::::::::::::::::::::::::::::::|/:/`‐'::\;;;;;;;_|
   |    |/::::::::::::::::::::::\:::::::::::::::::::::::::::::|::/::::|::::/:::::::::::/
    |   /:::::::::::::::::::::::::::::::::|:::::::::::::::::::::O::|::|::::::|:::::::::::::::/
800132人目の素数さん:2012/05/03(木) 10:52:59.91
確率の問題です。
同じ形をした3個の箱A、B、Cがある。
箱Aの中には赤玉1個と青玉1個が入っている。箱Bの中には赤玉1個と青玉3個が、
箱Cの中には赤玉2個と青玉3個が入っている。
3つの箱の中から1つの箱を選び、選んだその箱から玉を1個無作為に取り出すとき、次の確率を求めよ
@取りだした玉が青玉である確率
A取りだした玉が青玉である時、箱Aが選ばれる確率

ベイズの定理を使うようですが、文系の私には難しいです。
どなたか解き方を教えてください。
801132人目の素数さん:2012/05/03(木) 11:12:26.51
Aの箱から赤玉、青玉を取り出す確率をそれぞれPa(R)、Pa(B)と表すと
@P(B)=Pa(B)+Pb(B)+Pc(B)=1/3*1/2+1/3*3/4+1/3*3/5=37/60
APa(B)/P(B)=1/3*1/2/(37/60)=10/37
802132人目の素数さん:2012/05/03(木) 11:13:20.95
>>800
(1)もわからんのなら論外すぎる。文系とか関係ない。
中学生レベル。教科書読め。
803792:2012/05/03(木) 12:19:26.00
>>795,798
ありがとうございます。ですが、どっちなんでしょう?

>>796
ありがとうございます。すみませんその通りで
(与式) = ((x^2)-(y^2)+3xy)((x^2)-(y^2) -3xy)が答えでした。


答えは下のどちらでもいいけど
(与式) = ((x^2)+(y^2)+(√(13)xy))((x^2)+(y^2) -(√(13)xy))
(与式) = ((x^2)-(y^2)+3xy)((x^2)-(y^2) -3xy)

ここからさらに続く問題の時、下に因数分解したほうが後が楽という事ですか?
804132人目の素数さん:2012/05/03(木) 12:53:42.45
>>803
整数係数の範囲で因数分解するなら ((x^2)-(y^2)+3xy)((x^2)-(y^2) -3xy) で終わり。
整数係数に限らないなら (x+((3+√13)/2)y)(x+((3-√13)/2)y)(x-((3+√13)/2)y)(x-((3-√13)/2)y) まで分解する。
805132人目の素数さん:2012/05/03(木) 13:50:44.65
A1,A2,A3…An∈Bが互いに排反であり、A1∪A2∪…∪An=Aとすると
任意のC∈Bに対して、P(C|Aa)=P(C|A)である事を示せ。(a=1,2,…n)

条件付確率の証明ですが解き方を教えてください。
806132人目の素数さん:2012/05/03(木) 14:11:57.49
自分の頭で考えない奴が多いな
807132人目の素数さん:2012/05/03(木) 14:19:31.54
考える気がないから質問してるんじゃないか
教育スレと勘違いしているバカが大杉
どうみても質問者のニーズに応えているとはいえない糞スレ
808132人目の素数さん:2012/05/03(木) 14:21:57.45
>>807
教育すれてあるの?
質問者は偉いの?
809132人目の素数さん:2012/05/03(木) 14:22:17.10
東大にも前に出てた論証とかの問題によくある

xに0とか1とか−1とか代入して
その式を満たす必要条件であることを確認したあと
十分性を考慮して代入する

みたいな手法があるじゃん
確かにそれは理解できるんだけど、もしそのxにもっと大きな他の数を代入したら
式が成立するかもしれないってことにはならないのかって(実際にはならないんだろうけど)疑問が拭えないんだが…

要は代入して必要十分を満たす値が唯一の解になる理由を教えてください
810132人目の素数さん:2012/05/03(木) 14:26:15.36
          __ノ)-'´ ̄ ̄`ー- 、_
        , '´  _. -‐'''"二ニニ=-`ヽ、
      /   /:::::; -‐''"        `ーノ
     /   /:::::/           \
     /    /::::::/          | | |  |
     |   |:::::/ /     |  | | | |  |
      |   |::/ / / |  | ||  | | ,ハ .| ,ハ|
      |   |/ / / /| ,ハノ| /|ノレ,ニ|ル' 
     |   |  | / / レ',二、レ′ ,ィイ|゙/   私は只の数ヲタなんかとは付き合わないわ。
.     |   \ ∠イ  ,イイ|    ,`-' |      頭が良くて数学が出来てかっこいい人。それが必要条件よ。
     |     l^,人|  ` `-'     ゝ  |        さらに Ann.of Math に論文書けば十分条件にもなるわよ。
      |      ` -'\       ー'  人          一番嫌いなのは論文数を増やすためにくだらない論文を書いて
    |        /(l     __/  ヽ、           良い論文の出版を遅らせるお馬鹿な人。
     |       (:::::`‐-、__  |::::`、     ヒニニヽ、         あなたの論文が Ann of Math に accept される確率は?
    |      / `‐-、::::::::::`‐-、::::\   /,ニニ、\            それとも最近は Inv. Math. の方が上かしら?
   |      |::::::::::::::::::|` -、:::::::,ヘ ̄|'、  ヒニ二、 \
.   |      /::::::::::::::::::|::::::::\/:::O`、::\   | '、   \
   |      /:::::::::::::::::::/:::::::::::::::::::::::::::::'、::::\ノ  ヽ、  |
  |      |:::::/:::::::::/:::::::::::::::::::::::::::::::::::'、',::::'、  /:\__/‐、
  |      |/:::::::::::/::::::::::::::::::::::::::::::::::O::| '、::| く::::::::::::: ̄|
   |     /_..-'´ ̄`ー-、:::::::::::::::::::::::::::::::::::|/:/`‐'::\;;;;;;;_|
   |    |/::::::::::::::::::::::\:::::::::::::::::::::::::::::|::/::::|::::/:::::::::::/
    |   /:::::::::::::::::::::::::::::::::|:::::::::::::::::::::O::|::|::::::|:::::::::::::::/
811132人目の素数さん:2012/05/03(木) 14:30:10.09
>>809
つまり、必要十分が何を意味するかもわからずに、機械的に「ヒツヨウジュウブンジョウケン」をチェックしているわけだ
812132人目の素数さん:2012/05/03(木) 14:37:26.30
だってある意味で適当にえらんだ数値(例:x=0,1)を代入して成り立つときの(m,n)=(a,b)を元の式に代入しなおしてx=0,1が成り立つのは当たり前なんじゃないんですか?
他に解があることはないんですか?

ホントによくわからないんでわかりやすく教えてほしいです
813132人目の素数さん:2012/05/03(木) 14:42:14.90
>>793
基底を揃えろってんだよバカ
814132人目の素数さん:2012/05/03(木) 14:48:43.04
>>809
> xに0とか1とか−1とか代入して
これで必要十分条件がでてくるように選んでいる(選べる)から
815132人目の素数さん:2012/05/03(木) 16:48:37.93
809には無理だということが理解できたようだな
816132人目の素数さん:2012/05/03(木) 16:57:39.97
>>812
元の問題を省略するから訳わからんけど、
xに0と1のいずれもで成り立つとき、(m,n)が(a,b)しかないとなったのなら、
その他の(m,n)ではx=0、1の少なくともどちらかで成り立たんことになるだろ。
817132人目の素数さん:2012/05/03(木) 17:01:38.14
lim n!/n^n=0ってどうやって示せばいいですか?
1/n×2/n×3/n×4/n×…×n/nと分けてみたんですがどうすればいいかよく分かりませんでした
818132人目の素数さん:2012/05/03(木) 17:08:45.25
>>817
n>2 のとき 1/n×2/n×3/n×4/n×…×n/n<1/n×n/n×n/n×n/n×…×n/n
819132人目の素数さん:2012/05/03(木) 19:31:45.15
これの1ページ目と2ページ目がなに言ってるのかさっぱりわからないんだ。
http://www.math.leidenuniv.nl/~hwl/PUBLICATIONS/1992d/art.pdf
解説してほしい
820132人目の素数さん:2012/05/03(木) 19:56:10.66
本文全部じゃん >1ページ目と2ページ目
821132人目の素数さん:2012/05/03(木) 20:07:00.68
また論文丸投げか
822132人目の素数さん:2012/05/03(木) 20:08:01.73
たぶん言ってることは中高生レベルの数学なんじゃないかと思うんだけど
おれやっぱ頭悪いのかな・・・

ついでに、わかる人いたら
フェルマーの最終定理を少し改変して
x,y,z,nが有理数のとき、どういう結果になるのか知りたいんだわ
823132人目の素数さん:2012/05/03(木) 22:37:50.72
1^(1/3)+1^(1/3)=8^(1/3)
(a/b)^n+(c/d)^n=(e/f)^n ⇔ (adf)^n+(bcf)^n=(bde)^n
824132人目の素数さん:2012/05/04(金) 04:21:56.00
分からないので、どなたか教えてください。
標準正規分布のモーメント母関数を計算し、3次モーメントと4次モーメントを求めなさい。
マクローリン展開するのがヒントなんですが…。
825132人目の素数さん:2012/05/04(金) 08:17:01.50
826132人目の素数さん:2012/05/04(金) 09:46:12.51
          __ノ)-'´ ̄ ̄`ー- 、_
        , '´  _. -‐'''"二ニニ=-`ヽ、
      /   /:::::; -‐''"        `ーノ
     /   /:::::/           \
     /    /::::::/          | | |  |
     |   |:::::/ /     |  | | | |  |
      |   |::/ / / |  | ||  | | ,ハ .| ,ハ|
      |   |/ / / /| ,ハノ| /|ノレ,ニ|ル' 
     |   |  | / / レ',二、レ′ ,ィイ|゙/   私は只の数ヲタなんかとは付き合わないわ。
.     |   \ ∠イ  ,イイ|    ,`-' |      頭が良くて数学が出来てかっこいい人。それが必要条件よ。
     |     l^,人|  ` `-'     ゝ  |        さらに Ann.of Math に論文書けば十分条件にもなるわよ。
      |      ` -'\       ー'  人          一番嫌いなのは論文数を増やすためにくだらない論文を書いて
    |        /(l     __/  ヽ、           良い論文の出版を遅らせるお馬鹿な人。
     |       (:::::`‐-、__  |::::`、     ヒニニヽ、         あなたの論文が Ann of Math に accept される確率は?
    |      / `‐-、::::::::::`‐-、::::\   /,ニニ、\            それとも最近は Inv. Math. の方が上かしら?
   |      |::::::::::::::::::|` -、:::::::,ヘ ̄|'、  ヒニ二、 \
.   |      /::::::::::::::::::|::::::::\/:::O`、::\   | '、   \
   |      /:::::::::::::::::::/:::::::::::::::::::::::::::::'、::::\ノ  ヽ、  |
  |      |:::::/:::::::::/:::::::::::::::::::::::::::::::::::'、',::::'、  /:\__/‐、
  |      |/:::::::::::/::::::::::::::::::::::::::::::::::O::| '、::| く::::::::::::: ̄|
   |     /_..-'´ ̄`ー-、:::::::::::::::::::::::::::::::::::|/:/`‐'::\;;;;;;;_|
   |    |/::::::::::::::::::::::\:::::::::::::::::::::::::::::|::/::::|::::/:::::::::::/
    |   /:::::::::::::::::::::::::::::::::|:::::::::::::::::::::O::|::|::::::|:::::::::::::::/
827132人目の素数さん:2012/05/04(金) 14:11:29.43
問 lim_[n→∞]b[n]=β≠0, b[n]≠0であれば、{1/b[n]}が有界であることを示せ。

解答
ε=|β|/2に対して、番号n[0]が定まって次が成り立つ。
n≧n[0] ⇒ |b[n]-β|<|β|/2 ⇒ |b[n]|>|β|/2
...

解答の|b[n]-β|<|β|/2 ⇒ |b[n]|>|β|/2ってどうやって出てきたんでしょうか?
828132人目の素数さん:2012/05/04(金) 14:57:53.02
>>827
数直線をイメージすれば分かる
βとの差が|β|/2未満ってことは、βが正の場合

---0---β/2---β---3β/2---

のβ/2〜3β/2の間にb[n]があるってこと
βが負の場合も同様
829132人目の素数さん:2012/05/04(金) 18:38:59.56
|b[n]|<|β|/2ー>|b[n]-β|>||b[n]|-|β||>|β|/2
830132人目の素数さん:2012/05/04(金) 19:21:06.63
          __ノ)-'´ ̄ ̄`ー- 、_
        , '´  _. -‐'''"二ニニ=-`ヽ、
      /   /:::::; -‐''"        `ーノ
     /   /:::::/           \
     /    /::::::/          | | |  |
     |   |:::::/ /     |  | | | |  |
      |   |::/ / / |  | ||  | | ,ハ .| ,ハ|
      |   |/ / / /| ,ハノ| /|ノレ,ニ|ル' 
     |   |  | / / レ',二、レ′ ,ィイ|゙/   私は只の数ヲタなんかとは付き合わないわ。
.     |   \ ∠イ  ,イイ|    ,`-' |      頭が良くて数学が出来てかっこいい人。それが必要条件よ。
     |     l^,人|  ` `-'     ゝ  |        さらに Ann.of Math に論文書けば十分条件にもなるわよ。
      |      ` -'\       ー'  人          一番嫌いなのは論文数を増やすためにくだらない論文を書いて
    |        /(l     __/  ヽ、           良い論文の出版を遅らせるお馬鹿な人。
     |       (:::::`‐-、__  |::::`、     ヒニニヽ、         あなたの論文が Ann of Math に accept される確率は?
    |      / `‐-、::::::::::`‐-、::::\   /,ニニ、\            それとも最近は Inv. Math. の方が上かしら?
   |      |::::::::::::::::::|` -、:::::::,ヘ ̄|'、  ヒニ二、 \
.   |      /::::::::::::::::::|::::::::\/:::O`、::\   | '、   \
   |      /:::::::::::::::::::/:::::::::::::::::::::::::::::'、::::\ノ  ヽ、  |
  |      |:::::/:::::::::/:::::::::::::::::::::::::::::::::::'、',::::'、  /:\__/‐、
  |      |/:::::::::::/::::::::::::::::::::::::::::::::::O::| '、::| く::::::::::::: ̄|
   |     /_..-'´ ̄`ー-、:::::::::::::::::::::::::::::::::::|/:/`‐'::\;;;;;;;_|
   |    |/::::::::::::::::::::::\:::::::::::::::::::::::::::::|::/::::|::::/:::::::::::/
    |   /:::::::::::::::::::::::::::::::::|:::::::::::::::::::::O::|::|::::::|:::::::::::::::/
831132人目の素数さん:2012/05/04(金) 19:25:32.37
          __ノ)-'´ ̄ ̄`ー- 、_
        , '´  _. -‐'''"二ニニ=-`ヽ、
      /   /:::::; -‐''"        `ーノ
     /   /:::::/           \
     /    /::::::/          | | |  |
     |   |:::::/ /     |  | | | |  |
      |   |::/ / / |  | ||  | | ,ハ .| ,ハ|
      |   |/ / / /| ,ハノ| /|ノレ,ニ|ル' 
     |   |  | / / レ',二、レ′ ,ィイ|゙/   私は只の数ヲタなんかとは付き合わないわ。
.     |   \ ∠イ  ,イイ|    ,`-' |      頭が良くて数学が出来てかっこいい人。それが必要条件よ。
     |     l^,人|  ` `-'     ゝ  |        さらに Ann.of Math に論文書けば十分条件にもなるわよ。
      |      ` -'\       ー'  人          一番嫌いなのは論文数を増やすためにくだらない論文を書いて
    |        /(l     __/  ヽ、           良い論文の出版を遅らせるお馬鹿な人。
     |       (:::::`‐-、__  |::::`、     ヒニニヽ、         あなたの論文が Ann of Math に accept される確率は?
    |      / `‐-、::::::::::`‐-、::::\   /,ニニ、\            それとも最近は Inv. Math. の方が上かしら?
   |      |::::::::::::::::::|` -、:::::::,ヘ ̄|'、  ヒニ二、 \
.   |      /::::::::::::::::::|::::::::\/:::O`、::\   | '、   \
   |      /:::::::::::::::::::/:::::::::::::::::::::::::::::'、::::\ノ  ヽ、  |
  |      |:::::/:::::::::/:::::::::::::::::::::::::::::::::::'、',::::'、  /:\__/‐、
  |      |/:::::::::::/::::::::::::::::::::::::::::::::::O::| '、::| く::::::::::::: ̄|
   |     /_..-'´ ̄`ー-、:::::::::::::::::::::::::::::::::::|/:/`‐'::\;;;;;;;_|
   |    |/::::::::::::::::::::::\:::::::::::::::::::::::::::::|::/::::|::::/:::::::::::/
    |   /:::::::::::::::::::::::::::::::::|:::::::::::::::::::::O::|::|::::::|:::::::::::::::/
832132人目の素数さん:2012/05/04(金) 19:28:11.50
          __ノ)-'´ ̄ ̄`ー- 、_
        , '´  _. -‐'''"二ニニ=-`ヽ、
      /   /:::::; -‐''"        `ーノ
     /   /:::::/           \
     /    /::::::/          | | |  |
     |   |:::::/ /     |  | | | |  |
      |   |::/ / / |  | ||  | | ,ハ .| ,ハ|
      |   |/ / / /| ,ハノ| /|ノレ,ニ|ル' 
     |   |  | / / レ',二、レ′ ,ィイ|゙/   私は只の数ヲタなんかとは付き合わないわ。
.     |   \ ∠イ  ,イイ|    ,`-' |      頭が良くて数学が出来てかっこいい人。それが必要条件よ。
     |     l^,人|  ` `-'     ゝ  |        さらに Ann.of Math に論文書けば十分条件にもなるわよ。
      |      ` -'\       ー'  人          一番嫌いなのは論文数を増やすためにくだらない論文を書いて
    |        /(l     __/  ヽ、           良い論文の出版を遅らせるお馬鹿な人。
     |       (:::::`‐-、__  |::::`、     ヒニニヽ、         あなたの論文が Ann of Math に accept される確率は?
    |      / `‐-、::::::::::`‐-、::::\   /,ニニ、\            それとも最近は Inv. Math. の方が上かしら?
   |      |::::::::::::::::::|` -、:::::::,ヘ ̄|'、  ヒニ二、 \
.   |      /::::::::::::::::::|::::::::\/:::O`、::\   | '、   \
   |      /:::::::::::::::::::/:::::::::::::::::::::::::::::'、::::\ノ  ヽ、  |
  |      |:::::/:::::::::/:::::::::::::::::::::::::::::::::::'、',::::'、  /:\__/‐、
  |      |/:::::::::::/::::::::::::::::::::::::::::::::::O::| '、::| く::::::::::::: ̄|
   |     /_..-'´ ̄`ー-、:::::::::::::::::::::::::::::::::::|/:/`‐'::\;;;;;;;_|
   |    |/::::::::::::::::::::::\:::::::::::::::::::::::::::::|::/::::|::::/:::::::::::/
    |   /:::::::::::::::::::::::::::::::::|:::::::::::::::::::::O::|::|::::::|:::::::::::::::/
833132人目の素数さん:2012/05/04(金) 19:44:03.44
          __ノ)-'´ ̄ ̄`ー- 、_
        , '´  _. -‐'''"二ニニ=-`ヽ、
      /   /:::::; -‐''"        `ーノ
     /   /:::::/           \
     /    /::::::/          | | |  |
     |   |:::::/ /     |  | | | |  |
      |   |::/ / / |  | ||  | | ,ハ .| ,ハ|
      |   |/ / / /| ,ハノ| /|ノレ,ニ|ル' 
     |   |  | / / レ',二、レ′ ,ィイ|゙/   私は只の数ヲタなんかとは付き合わないわ。
.     |   \ ∠イ  ,イイ|    ,`-' |      頭が良くて数学が出来てかっこいい人。それが必要条件よ。
     |     l^,人|  ` `-'     ゝ  |        さらに Ann.of Math に論文書けば十分条件にもなるわよ。
      |      ` -'\       ー'  人          一番嫌いなのは論文数を増やすためにくだらない論文を書いて
    |        /(l     __/  ヽ、           良い論文の出版を遅らせるお馬鹿な人。
     |       (:::::`‐-、__  |::::`、     ヒニニヽ、         あなたの論文が Ann of Math に accept される確率は?
    |      / `‐-、::::::::::`‐-、::::\   /,ニニ、\            それとも最近は Inv. Math. の方が上かしら?
   |      |::::::::::::::::::|` -、:::::::,ヘ ̄|'、  ヒニ二、 \
.   |      /::::::::::::::::::|::::::::\/:::O`、::\   | '、   \
   |      /:::::::::::::::::::/:::::::::::::::::::::::::::::'、::::\ノ  ヽ、  |
  |      |:::::/:::::::::/:::::::::::::::::::::::::::::::::::'、',::::'、  /:\__/‐、
  |      |/:::::::::::/::::::::::::::::::::::::::::::::::O::| '、::| く::::::::::::: ̄|
   |     /_..-'´ ̄`ー-、:::::::::::::::::::::::::::::::::::|/:/`‐'::\;;;;;;;_|
   |    |/::::::::::::::::::::::\:::::::::::::::::::::::::::::|::/::::|::::/:::::::::::/
    |   /:::::::::::::::::::::::::::::::::|:::::::::::::::::::::O::|::|::::::|:::::::::::::::/
834132人目の素数さん:2012/05/04(金) 20:13:22.42
X3乗-2X2乗-4X-1
の因数分解の解説をお願いします
835132人目の素数さん:2012/05/04(金) 20:16:16.96
文字列「www」を「・・・」に置き換える操作をほどこしたとき
文章「wwwww」はどうなりますか?
836132人目の素数さん:2012/05/04(金) 20:28:20.39
知らね
837132人目の素数さん:2012/05/04(金) 20:29:40.27
>>835
自分で考えた問題か?
まったく面白くないぞ
838132人目の素数さん:2012/05/04(金) 20:42:15.71
>>835
「wwwww」が定義されるとは限らない
839132人目の素数さん:2012/05/04(金) 21:16:17.31
>>835
その手の正規表現は有限オートマトンになったりして大変クソになる
計算工学の基礎の方

例えば
・ ある文字列が I で終わっている場合、U をそれにつなげることができる(xI → xIU)
・ Mの後の文字列は全体を複製できる(Mx → Mxx)
・ 3つの I が連続している場合(III)、それらを1つの U に置換できる(xIIIy → xUy)
・ 2つの U が連続している場合、それらを削除できる(xUUy → xy)
この規則だけからMIからMUを作り出すことはできないことが証明されている

文字入れ替えの規則が厳密に与えられていれば
作れる文字列変換できる文字列と どうしても到達できない文字列が出てくる
840132人目の素数さん:2012/05/04(金) 21:46:57.99
$ echo wwwww | sed -e s/www/・・・/g
・・・ww
841132人目の素数さん:2012/05/04(金) 22:10:14.70
小学校の算数質問スレがなくなっていたのでここで質問させてください

小学五年の小数のかけ算わり算、割合の考え方がわかりません

小数のかけ算やわり算をするとき数直線を使った図がありますが、
数直線の図がよくわかりません
以下参考書転載
かけ算、1mの重さが4gの針金があるとき、3mにあたる重さを求める
重さ 0  4  8 □(g)
   ├─┼─┼─┤
長さ 0  1  2  3(m)
(↑のかけ算は関数の考え方?)
かけ算、〇×□とは○を1としたとき□にあたる数を求めること、という解説の解説をお願いします

わり算
  A=4  C
├─┼──┼── 
  1  B
わり算、A×B=Cのとき、Aを1としたときのBにあたるところを「B倍」といいます。わり算とは、AやBを求める計算
転載以上
とかどういう意味でしょう?もっとわかりやすい考え方や例えはありますか?
そもそもかけ算とわり算の原理的な性質ってどういうものですか?
数直線の例えがわからないのです

842続きます:2012/05/04(金) 22:10:43.95

割合もわかりません
以下、転載
「今日、全児童の12%にあたる48人が欠席しました。全児童は何人か」という問題の解説で
割合  0.12   1
├─┼────┼──┤
児童の数 48   □
     ↑   ↑
   比べる量 もとにする量

これは小数を含む計算だから難しそうに見えるだけです。
「何かの3倍は18です。何かはいくつですか」というとき18÷3の式で求めますよね。
つまり、1にあたる、もとにする量を求めたいときは、わり算で求めるのです。 一人分を求めたときと同じです。
転載以上

やはり数直線になると意味がわからないです

文章題では、どのようなときにかけ算わり算を使いわけるとかコツはありますか?


小学校の質問をしてごめんなさい。本当にわからないのです。よろしくお願いします
843132人目の素数さん:2012/05/04(金) 22:15:37.91
          __ノ)-'´ ̄ ̄`ー- 、_
        , '´  _. -‐'''"二ニニ=-`ヽ、
      /   /:::::; -‐''"        `ーノ
     /   /:::::/           \
     /    /::::::/          | | |  |
     |   |:::::/ /     |  | | | |  |
      |   |::/ / / |  | ||  | | ,ハ .| ,ハ|
      |   |/ / / /| ,ハノ| /|ノレ,ニ|ル' 
     |   |  | / / レ',二、レ′ ,ィイ|゙/   私は只の数ヲタなんかとは付き合わないわ。
.     |   \ ∠イ  ,イイ|    ,`-' |      頭が良くて数学が出来てかっこいい人。それが必要条件よ。
     |     l^,人|  ` `-'     ゝ  |        さらに Ann.of Math に論文書けば十分条件にもなるわよ。
      |      ` -'\       ー'  人          一番嫌いなのは論文数を増やすためにくだらない論文を書いて
    |        /(l     __/  ヽ、           良い論文の出版を遅らせるお馬鹿な人。
     |       (:::::`‐-、__  |::::`、     ヒニニヽ、         あなたの論文が Ann of Math に accept される確率は?
    |      / `‐-、::::::::::`‐-、::::\   /,ニニ、\            それとも最近は Inv. Math. の方が上かしら?
   |      |::::::::::::::::::|` -、:::::::,ヘ ̄|'、  ヒニ二、 \
.   |      /::::::::::::::::::|::::::::\/:::O`、::\   | '、   \
   |      /:::::::::::::::::::/:::::::::::::::::::::::::::::'、::::\ノ  ヽ、  |
  |      |:::::/:::::::::/:::::::::::::::::::::::::::::::::::'、',::::'、  /:\__/‐、
  |      |/:::::::::::/::::::::::::::::::::::::::::::::::O::| '、::| く::::::::::::: ̄|
   |     /_..-'´ ̄`ー-、:::::::::::::::::::::::::::::::::::|/:/`‐'::\;;;;;;;_|
   |    |/::::::::::::::::::::::\:::::::::::::::::::::::::::::|::/::::|::::/:::::::::::/
    |   /:::::::::::::::::::::::::::::::::|:::::::::::::::::::::O::|::|::::::|:::::::::::::::/
844132人目の素数さん:2012/05/04(金) 22:35:15.64
>>841
転載するならそっくりそのまま改変せずに。
キャプチャーしてもらった方がいいかも知れない。
845132人目の素数さん:2012/05/05(土) 02:59:16.06
>>834
X^3−2X^2−4X−1
まず試しにX=−1を代入してみると
(−1)^3−2(−1)^2−4(−1)−1=−1−2+4−1=0 となって X+1 の因数があることが分かる。
以下省略。
846132人目の素数さん:2012/05/05(土) 09:51:10.25
初歩的な質問で申し訳ないのですが
ベクトルの内積が(a,c)・(b,d)=ac+bd
なのに対して、関数の内積が∫f(x)・g(x)dx
になるのはなぜですか?
∫が和に対応するとして、f(x)が(f(-∞) ,.......,f(∞))の無限次元のベクトルだとして、
dxは何に対応するのですか?
847132人目の素数さん:2012/05/05(土) 10:14:14.80
内積とは何か?
満たしていてほしい性質を考えて抽出しろ
848841:2012/05/05(土) 11:13:41.07
1mの重さが4gのとき、2.4gにあたる重さは、4×2.4=9.6g

重さ 4 8 □ 12(g)
 ──┼-┼-┼-┼─
長さ 1 2 2.4 3(m)


という小数のかけ算の考え方で、かけ算の概念がわからなくなりました

この数直線を使うかけ算の考え方って、どういう考え方ですか?そして今までの同数累加とはどういう考え方ですか?


初歩の初歩でごめんなさい。よろしくお願いします
849132人目の素数さん:2012/05/05(土) 11:20:37.67
>>848
2.4mにあたる重さだろ?

長さと重さの値が比例することが前提条件にある。
長さが2.4倍になったのなら重さも2.4倍。
4の2.4倍を式にしたものが4*2.4ってだけ。
850132人目の素数さん:2012/05/05(土) 11:20:43.64
>1mの重さが4gのとき、2.4gにあたる重さは、4×2.4=9.6g
www
まずは、国語からだな

じょーだんでなかったら、2.4gにあたる重さは9.6gから説明してくれ
851132人目の素数さん:2012/05/05(土) 11:46:22.48
>>848
マジレスしてやるよ
2.4mは0.1mで分ければ24個分
じゃあ、0.1mは重さに換算すると、0.4g
つまり、0.4gが24個分つまり、0.4gを24回足すと9.6g

その数直線をもっとたくさん分割するんだ
852132人目の素数さん:2012/05/05(土) 12:03:52.85
          __ノ)-'´ ̄ ̄`ー- 、_
        , '´  _. -‐'''"二ニニ=-`ヽ、
      /   /:::::; -‐''"        `ーノ
     /   /:::::/           \
     /    /::::::/          | | |  |
     |   |:::::/ /     |  | | | |  |
      |   |::/ / / |  | ||  | | ,ハ .| ,ハ|
      |   |/ / / /| ,ハノ| /|ノレ,ニ|ル' 
     |   |  | / / レ',二、レ′ ,ィイ|゙/   私は只の数ヲタなんかとは付き合わないわ。
.     |   \ ∠イ  ,イイ|    ,`-' |      頭が良くて数学が出来てかっこいい人。それが必要条件よ。
     |     l^,人|  ` `-'     ゝ  |        さらに Ann.of Math に論文書けば十分条件にもなるわよ。
      |      ` -'\       ー'  人          一番嫌いなのは論文数を増やすためにくだらない論文を書いて
    |        /(l     __/  ヽ、           良い論文の出版を遅らせるお馬鹿な人。
     |       (:::::`‐-、__  |::::`、     ヒニニヽ、         あなたの論文が Ann of Math に accept される確率は?
    |      / `‐-、::::::::::`‐-、::::\   /,ニニ、\            それとも最近は Inv. Math. の方が上かしら?
   |      |::::::::::::::::::|` -、:::::::,ヘ ̄|'、  ヒニ二、 \
.   |      /::::::::::::::::::|::::::::\/:::O`、::\   | '、   \
   |      /:::::::::::::::::::/:::::::::::::::::::::::::::::'、::::\ノ  ヽ、  |
  |      |:::::/:::::::::/:::::::::::::::::::::::::::::::::::'、',::::'、  /:\__/‐、
  |      |/:::::::::::/::::::::::::::::::::::::::::::::::O::| '、::| く::::::::::::: ̄|
   |     /_..-'´ ̄`ー-、:::::::::::::::::::::::::::::::::::|/:/`‐'::\;;;;;;;_|
   |    |/::::::::::::::::::::::\:::::::::::::::::::::::::::::|::/::::|::::/:::::::::::/
    |   /:::::::::::::::::::::::::::::::::|:::::::::::::::::::::O::|::|::::::|:::::::::::::::/
853132人目の素数さん:2012/05/05(土) 13:32:37.44
          __ノ)-'´ ̄ ̄`ー- 、_
        , '´  _. -‐'''"二ニニ=-`ヽ、
      /   /:::::; -‐''"        `ーノ
     /   /:::::/           \
     /    /::::::/          | | |  |
     |   |:::::/ /     |  | | | |  |
      |   |::/ / / |  | ||  | | ,ハ .| ,ハ|
      |   |/ / / /| ,ハノ| /|ノレ,ニ|ル' 
     |   |  | / / レ',二、レ′ ,ィイ|゙/   私は只の数ヲタなんかとは付き合わないわ。
.     |   \ ∠イ  ,イイ|    ,`-' |      頭が良くて数学が出来てかっこいい人。それが必要条件よ。
     |     l^,人|  ` `-'     ゝ  |        さらに Ann.of Math に論文書けば十分条件にもなるわよ。
      |      ` -'\       ー'  人          一番嫌いなのは論文数を増やすためにくだらない論文を書いて
    |        /(l     __/  ヽ、           良い論文の出版を遅らせるお馬鹿な人。
     |       (:::::`‐-、__  |::::`、     ヒニニヽ、         あなたの論文が Ann of Math に accept される確率は?
    |      / `‐-、::::::::::`‐-、::::\   /,ニニ、\            それとも最近は Inv. Math. の方が上かしら?
   |      |::::::::::::::::::|` -、:::::::,ヘ ̄|'、  ヒニ二、 \
.   |      /::::::::::::::::::|::::::::\/:::O`、::\   | '、   \
   |      /:::::::::::::::::::/:::::::::::::::::::::::::::::'、::::\ノ  ヽ、  |
  |      |:::::/:::::::::/:::::::::::::::::::::::::::::::::::'、',::::'、  /:\__/‐、
  |      |/:::::::::::/::::::::::::::::::::::::::::::::::O::| '、::| く::::::::::::: ̄|
   |     /_..-'´ ̄`ー-、:::::::::::::::::::::::::::::::::::|/:/`‐'::\;;;;;;;_|
   |    |/::::::::::::::::::::::\:::::::::::::::::::::::::::::|::/::::|::::/:::::::::::/
    |   /:::::::::::::::::::::::::::::::::|:::::::::::::::::::::O::|::|::::::|:::::::::::::::/
854841:2012/05/05(土) 13:42:44.36
回答ありがとうございます

わかったようなわからないような…です
リアルで自閉症なので頭が固いです


このAAの女の子に声をかけることができるまでどのくらい勉強すればいいかすらわかりません
855132人目の素数さん:2012/05/05(土) 14:15:02.58
日本語の問題なのかもしれませんが、混乱しています。

「ある病気Aであるかどうかを判定するのに、Bという検査が行われるとする。
 Bという検査が陽性であれば、病気Aにかかっている確率は90%であるという・・・」

A:病気Aにかかっている事象
B:検査Bで陽性が出る事象

P(B|A)=0.9となってるのですが、
どうしてもP(A|B)=0.9じゃないかと思ってしまいます。

P(B|A)は、病気Aにかかってる人が、検査Bを受けて陽性となる確率であり
上記問題文に対応するのは、検査Bが陽性だった人が病気Aにかかっている確率
P(A|B)ではないのでしょうか?

宜しくお願いします。
856132人目の素数さん:2012/05/05(土) 14:17:09.84
          __ノ)-'´ ̄ ̄`ー- 、_
        , '´  _. -‐'''"二ニニ=-`ヽ、
      /   /:::::; -‐''"        `ーノ
     /   /:::::/           \
     /    /::::::/          | | |  |
     |   |:::::/ /     |  | | | |  |
      |   |::/ / / |  | ||  | | ,ハ .| ,ハ|
      |   |/ / / /| ,ハノ| /|ノレ,ニ|ル' 
     |   |  | / / レ',二、レ′ ,ィイ|゙/   私は只の数ヲタなんかとは付き合わないわ。
.     |   \ ∠イ  ,イイ|    ,`-' |      頭が良くて数学が出来てかっこいい人。それが必要条件よ。
     |     l^,人|  ` `-'     ゝ  |        さらに Ann.of Math に論文書けば十分条件にもなるわよ。
      |      ` -'\       ー'  人          一番嫌いなのは論文数を増やすためにくだらない論文を書いて
    |        /(l     __/  ヽ、           良い論文の出版を遅らせるお馬鹿な人。
     |       (:::::`‐-、__  |::::`、     ヒニニヽ、         あなたの論文が Ann of Math に accept される確率は?
    |      / `‐-、::::::::::`‐-、::::\   /,ニニ、\            それとも最近は Inv. Math. の方が上かしら?
   |      |::::::::::::::::::|` -、:::::::,ヘ ̄|'、  ヒニ二、 \
.   |      /::::::::::::::::::|::::::::\/:::O`、::\   | '、   \
   |      /:::::::::::::::::::/:::::::::::::::::::::::::::::'、::::\ノ  ヽ、  |
  |      |:::::/:::::::::/:::::::::::::::::::::::::::::::::::'、',::::'、  /:\__/‐、
  |      |/:::::::::::/::::::::::::::::::::::::::::::::::O::| '、::| く::::::::::::: ̄|
   |     /_..-'´ ̄`ー-、:::::::::::::::::::::::::::::::::::|/:/`‐'::\;;;;;;;_|
   |    |/::::::::::::::::::::::\:::::::::::::::::::::::::::::|::/::::|::::/:::::::::::/
    |   /:::::::::::::::::::::::::::::::::|:::::::::::::::::::::O::|::|::::::|:::::::::::::::/
857132人目の素数さん:2012/05/05(土) 14:26:46.65
無知な質問ですいませんが……

二乗して-1になる数は実数の範囲では定義できなくて、
それを定義した時に、虚数にまで数の範囲は拡大したということでいいんですよね?
では、三乗して-1になる数は-1だけですか?
四乗して-1になる数はないんですか?
 iだと二乗して-1、四乗だと1に戻ってしまいますよね……
一般にn乗して-1になる数の集合に名前がついていたりするんですか?

858132人目の素数さん:2012/05/05(土) 14:28:59.75
B on A とか B in A とか
そういう意味をもつ記法だろうな 
西洋由来だし
集合の B in A も B∈A の順で書くし

「Aという状況内部での(in)、 B」てな解釈あるいは覚え方でどうでしょう
859132人目の素数さん:2012/05/05(土) 14:45:19.03
>>858

>>855への回答でしょうか?
860132人目の素数さん:2012/05/05(土) 14:45:27.31
>>857
3乗して-1になる数をxと置くと、x^3=-1。
x^3+1=0
(x+1)(x^2-x+1)=0
x+1=0またはx^2-x+1=0
前者からx=-1。後者からは実数範囲では解がないが複素数まで範囲を拡げれば2つ出てくる。

x^4=-1
x^4+1=0
(x^2+i)(x^2-i)=0
あとはゴリゴリ
861132人目の素数さん:2012/05/05(土) 14:47:12.65
>>857
どのような代数方程式も複素数の範囲で解けるという定理がある
だからx^n+1=0の解も複素数で表される
つまり-1のn乗根も複素数で表される
862132人目の素数さん:2012/05/05(土) 15:08:52.13
>>857
以下の説明で、で虚数をiとする。
3乗と4乗はノー
2乗はその他に-i

http://ja.wikipedia.org/wiki/オイラーの公式
の図を参照のこと。Reは実数で、Imは虚数。

例えば)
wikiの図の90度の位置の値が2乗の答え=>cos(180/2)+i・sin(180/2)=i
wikiの図の60度の位置の値が3乗の答え=>cos(180/3)+i・sin(180/3)
wikiの図の45度の位置の値が4乗の答え=>cos(180/4)+i・sin(180/4)=1/√2+i/√2 

2乗と同様に3乗、4乗にも満たす答えはある。
他の答えは、
e^i・x=cos(x)+ i・sin(x)
e^i・n・x=cos(n・x)+ i・sin(n・x)
から考えてみると良い。
863132人目の素数さん:2012/05/05(土) 15:12:35.23
そして美しい
e^iπ=-1が導かれるのである。
864132人目の素数さん:2012/05/05(土) 15:21:39.07
>>863
それのどこが美しいの?
865132人目の素数さん:2012/05/05(土) 15:28:46.99
つ博士の愛した数式
866132人目の素数さん:2012/05/05(土) 15:29:49.12
それがどうした?
867132人目の素数さん:2012/05/05(土) 15:32:25.72
つゲバゲバ
868132人目の素数さん:2012/05/05(土) 15:45:51.18
          __ノ)-'´ ̄ ̄`ー- 、_
        , '´  _. -‐'''"二ニニ=-`ヽ、
      /   /:::::; -‐''"        `ーノ
     /   /:::::/           \
     /    /::::::/          | | |  |
     |   |:::::/ /     |  | | | |  |
      |   |::/ / / |  | ||  | | ,ハ .| ,ハ|
      |   |/ / / /| ,ハノ| /|ノレ,ニ|ル' 
     |   |  | / / レ',二、レ′ ,ィイ|゙/   私は只の数ヲタなんかとは付き合わないわ。
.     |   \ ∠イ  ,イイ|    ,`-' |      頭が良くて数学が出来てかっこいい人。それが必要条件よ。
     |     l^,人|  ` `-'     ゝ  |        さらに Ann.of Math に論文書けば十分条件にもなるわよ。
      |      ` -'\       ー'  人          一番嫌いなのは論文数を増やすためにくだらない論文を書いて
    |        /(l     __/  ヽ、           良い論文の出版を遅らせるお馬鹿な人。
     |       (:::::`‐-、__  |::::`、     ヒニニヽ、         あなたの論文が Ann of Math に accept される確率は?
    |      / `‐-、::::::::::`‐-、::::\   /,ニニ、\            それとも最近は Inv. Math. の方が上かしら?
   |      |::::::::::::::::::|` -、:::::::,ヘ ̄|'、  ヒニ二、 \
.   |      /::::::::::::::::::|::::::::\/:::O`、::\   | '、   \
   |      /:::::::::::::::::::/:::::::::::::::::::::::::::::'、::::\ノ  ヽ、  |
  |      |:::::/:::::::::/:::::::::::::::::::::::::::::::::::'、',::::'、  /:\__/‐、
  |      |/:::::::::::/::::::::::::::::::::::::::::::::::O::| '、::| く::::::::::::: ̄|
   |     /_..-'´ ̄`ー-、:::::::::::::::::::::::::::::::::::|/:/`‐'::\;;;;;;;_|
   |    |/::::::::::::::::::::::\:::::::::::::::::::::::::::::|::/::::|::::/:::::::::::/
    |   /:::::::::::::::::::::::::::::::::|:::::::::::::::::::::O::|::|::::::|:::::::::::::::/
869132人目の素数さん:2012/05/05(土) 15:54:18.91
>>864
つ鏡
870132人目の素数さん:2012/05/05(土) 16:16:09.29
それまで
指数、円(三角関数)、複素数の間に関係はほとんど無いだろう
と思われていたところに
実はそれらが深く強くつながっていると判明したからねえ
871132人目の素数さん:2012/05/05(土) 16:33:40.98
>>870
嘘だな。
そもそもオイラーの時代は
複素数も対数もせいぜい100年
まだ黎明期で、どこにどう関係がありそうとか
断じられる程に深くはない。
オイラー自身も切り開いていくわけだが。

百歩譲ってそんな嘘が、本当だったとしても
指数と円と複素数の間に関係が無いと思ってはいない
現代人にとってそれはたいして美しく無いということにもなる。
872132人目の素数さん:2012/05/05(土) 16:43:56.76
「関係はほとんど無いだろう」の意味が>>870>>871とで違う気がする
873132人目の素数さん:2012/05/05(土) 16:57:09.92
>>871の読解力が割と本気で心配だ
874132人目の素数さん:2012/05/05(土) 17:00:54.63
じゃ
>>870
>思われていた

の思うの主語は誰なの?
そんなこと思ってる人なんて
そんな時代にいないだろうと
でっちあげただけの架空の人物かい?
875132人目の素数さん:2012/05/05(土) 17:03:44.64
オイラーの公式が美しいと言ってる奴のほとんどは
単なる受け売りで自分じゃ綺麗だとは思ってもない
博士の愛した数式という電波が強すぎる映画見てもよくわかる
876132人目の素数さん:2012/05/05(土) 17:04:33.07
積極的・能動的に思っていることだけが思っていることじゃないんだぜ
877132人目の素数さん:2012/05/05(土) 17:12:28.33
複素数が市民権を得るのはガウス以後だから
あんま驚かなかったかもな
878132人目の素数さん:2012/05/05(土) 17:16:32.05
ガウスってホント天才だよなwwww
何やったか全く知らないケドみんなが天才って言ってるから天才なんだよwwww

オイラーの公式ってホントめちゃくちゃ美しいと思わね?????
よくわかんねーけど
みんなが美しいって言ってるし
強く結びついてるから美しいってことだywwwwwwwwww


そんなのりww
879132人目の素数さん:2012/05/05(土) 17:36:52.90
いいえコーシーのおかげです
880132人目の素数さん:2012/05/05(土) 17:39:02.58
3次と4次方程式の解の公式は?
881132人目の素数さん:2012/05/05(土) 17:41:15.36
>>880
それが何?
882132人目の素数さん:2012/05/05(土) 17:47:44.48
ノイマンは?
883132人目の素数さん:2012/05/05(土) 17:56:16.75
アレは計算が早いだけw
884NAS6 ◆n3AmnVhjwc :2012/05/05(土) 19:22:19.10
グラフのプロットデータから関数を推測するのはどうやるんだっけ?
多分指数関数
885132人目の素数さん:2012/05/05(土) 19:50:25.75
怪奇
886132人目の素数さん:2012/05/05(土) 19:56:41.21
          __ノ)-'´ ̄ ̄`ー- 、_
        , '´  _. -‐'''"二ニニ=-`ヽ、
      /   /:::::; -‐''"        `ーノ
     /   /:::::/           \
     /    /::::::/          | | |  |
     |   |:::::/ /     |  | | | |  |
      |   |::/ / / |  | ||  | | ,ハ .| ,ハ|
      |   |/ / / /| ,ハノ| /|ノレ,ニ|ル' 
     |   |  | / / レ',二、レ′ ,ィイ|゙/   私は只の数ヲタなんかとは付き合わないわ。
.     |   \ ∠イ  ,イイ|    ,`-' |      頭が良くて数学が出来てかっこいい人。それが必要条件よ。
     |     l^,人|  ` `-'     ゝ  |        さらに Ann.of Math に論文書けば十分条件にもなるわよ。
      |      ` -'\       ー'  人          一番嫌いなのは論文数を増やすためにくだらない論文を書いて
    |        /(l     __/  ヽ、           良い論文の出版を遅らせるお馬鹿な人。
     |       (:::::`‐-、__  |::::`、     ヒニニヽ、         あなたの論文が Ann of Math に accept される確率は?
    |      / `‐-、::::::::::`‐-、::::\   /,ニニ、\            それとも最近は Inv. Math. の方が上かしら?
   |      |::::::::::::::::::|` -、:::::::,ヘ ̄|'、  ヒニ二、 \
.   |      /::::::::::::::::::|::::::::\/:::O`、::\   | '、   \
   |      /:::::::::::::::::::/:::::::::::::::::::::::::::::'、::::\ノ  ヽ、  |
  |      |:::::/:::::::::/:::::::::::::::::::::::::::::::::::'、',::::'、  /:\__/‐、
  |      |/:::::::::::/::::::::::::::::::::::::::::::::::O::| '、::| く::::::::::::: ̄|
   |     /_..-'´ ̄`ー-、:::::::::::::::::::::::::::::::::::|/:/`‐'::\;;;;;;;_|
   |    |/::::::::::::::::::::::\:::::::::::::::::::::::::::::|::/::::|::::/:::::::::::/
    |   /:::::::::::::::::::::::::::::::::|:::::::::::::::::::::O::|::|::::::|:::::::::::::::/
887132人目の素数さん:2012/05/05(土) 20:24:46.27
          __ノ)-'´ ̄ ̄`ー- 、_
        , '´  _. -‐'''"二ニニ=-`ヽ、
      /   /:::::; -‐''"        `ーノ
     /   /:::::/           \
     /    /::::::/          | | |  |
     |   |:::::/ /     |  | | | |  |
      |   |::/ / / |  | ||  | | ,ハ .| ,ハ|
      |   |/ / / /| ,ハノ| /|ノレ,ニ|ル' 
     |   |  | / / レ',二、レ′ ,ィイ|゙/   私は只の数ヲタなんかとは付き合わないわ。
.     |   \ ∠イ  ,イイ|    ,`-' |      頭が良くて数学が出来てかっこいい人。それが必要条件よ。
     |     l^,人|  ` `-'     ゝ  |        さらに Ann.of Math に論文書けば十分条件にもなるわよ。
      |      ` -'\       ー'  人          一番嫌いなのは論文数を増やすためにくだらない論文を書いて
    |        /(l     __/  ヽ、           良い論文の出版を遅らせるお馬鹿な人。
     |       (:::::`‐-、__  |::::`、     ヒニニヽ、         あなたの論文が Ann of Math に accept される確率は?
    |      / `‐-、::::::::::`‐-、::::\   /,ニニ、\            それとも最近は Inv. Math. の方が上かしら?
   |      |::::::::::::::::::|` -、:::::::,ヘ ̄|'、  ヒニ二、 \
.   |      /::::::::::::::::::|::::::::\/:::O`、::\   | '、   \
   |      /:::::::::::::::::::/:::::::::::::::::::::::::::::'、::::\ノ  ヽ、  |
  |      |:::::/:::::::::/:::::::::::::::::::::::::::::::::::'、',::::'、  /:\__/‐、
  |      |/:::::::::::/::::::::::::::::::::::::::::::::::O::| '、::| く::::::::::::: ̄|
   |     /_..-'´ ̄`ー-、:::::::::::::::::::::::::::::::::::|/:/`‐'::\;;;;;;;_|
   |    |/::::::::::::::::::::::\:::::::::::::::::::::::::::::|::/::::|::::/:::::::::::/
    |   /:::::::::::::::::::::::::::::::::|:::::::::::::::::::::O::|::|::::::|:::::::::::::::/
888132人目の素数さん:2012/05/05(土) 20:31:49.76
( ) parentheses
{ } braces
[ ] brackets

では、< >は何と呼ばれているのでしょうか?
889132人目の素数さん:2012/05/05(土) 20:34:10.26
greater than
less than
890132人目の素数さん:2012/05/05(土) 20:34:46.62
電気工学や量子力学の文脈なら
<>こそがブラケット
891132人目の素数さん:2012/05/05(土) 21:00:10.80
>>855
君が正しいように思う。
892132人目の素数さん:2012/05/05(土) 21:11:18.65
ベクトル空間VからVへの線形写像fとVの部分空間Uについて
fをU上で制限して得られる写像の固有値はfの固有値に含まれてますか?
893132人目の素数さん:2012/05/05(土) 21:39:37.09
スピヴァックの「多変数の解析学」が難しいのか
そもそも多変数の解析学の勉強が難しいのか
894132人目の素数さん:2012/05/05(土) 21:41:48.66
それとも学習者の頭がムツカしいのか
895132人目の素数さん:2012/05/05(土) 21:44:48.36
          __ノ)-'´ ̄ ̄`ー- 、_
        , '´  _. -‐'''"二ニニ=-`ヽ、
      /   /:::::; -‐''"        `ーノ
     /   /:::::/           \
     /    /::::::/          | | |  |
     |   |:::::/ /     |  | | | |  |
      |   |::/ / / |  | ||  | | ,ハ .| ,ハ|
      |   |/ / / /| ,ハノ| /|ノレ,ニ|ル' 
     |   |  | / / レ',二、レ′ ,ィイ|゙/   私は只の数ヲタなんかとは付き合わないわ。
.     |   \ ∠イ  ,イイ|    ,`-' |      頭が良くて数学が出来てかっこいい人。それが必要条件よ。
     |     l^,人|  ` `-'     ゝ  |        さらに Ann.of Math に論文書けば十分条件にもなるわよ。
      |      ` -'\       ー'  人          一番嫌いなのは論文数を増やすためにくだらない論文を書いて
    |        /(l     __/  ヽ、           良い論文の出版を遅らせるお馬鹿な人。
     |       (:::::`‐-、__  |::::`、     ヒニニヽ、         あなたの論文が Ann of Math に accept される確率は?
    |      / `‐-、::::::::::`‐-、::::\   /,ニニ、\            それとも最近は Inv. Math. の方が上かしら?
   |      |::::::::::::::::::|` -、:::::::,ヘ ̄|'、  ヒニ二、 \
.   |      /::::::::::::::::::|::::::::\/:::O`、::\   | '、   \
   |      /:::::::::::::::::::/:::::::::::::::::::::::::::::'、::::\ノ  ヽ、  |
  |      |:::::/:::::::::/:::::::::::::::::::::::::::::::::::'、',::::'、  /:\__/‐、
  |      |/:::::::::::/::::::::::::::::::::::::::::::::::O::| '、::| く::::::::::::: ̄|
   |     /_..-'´ ̄`ー-、:::::::::::::::::::::::::::::::::::|/:/`‐'::\;;;;;;;_|
   |    |/::::::::::::::::::::::\:::::::::::::::::::::::::::::|::/::::|::::/:::::::::::/
    |   /:::::::::::::::::::::::::::::::::|:::::::::::::::::::::O::|::|::::::|:::::::::::::::/
896132人目の素数さん:2012/05/05(土) 21:45:11.44
          __ノ)-'´ ̄ ̄`ー- 、_
        , '´  _. -‐'''"二ニニ=-`ヽ、
      /   /:::::; -‐''"        `ーノ
     /   /:::::/           \
     /    /::::::/          | | |  |
     |   |:::::/ /     |  | | | |  |
      |   |::/ / / |  | ||  | | ,ハ .| ,ハ|
      |   |/ / / /| ,ハノ| /|ノレ,ニ|ル' 
     |   |  | / / レ',二、レ′ ,ィイ|゙/   私は只の数ヲタなんかとは付き合わないわ。
.     |   \ ∠イ  ,イイ|    ,`-' |      頭が良くて数学が出来てかっこいい人。それが必要条件よ。
     |     l^,人|  ` `-'     ゝ  |        さらに Ann.of Math に論文書けば十分条件にもなるわよ。
      |      ` -'\       ー'  人          一番嫌いなのは論文数を増やすためにくだらない論文を書いて
    |        /(l     __/  ヽ、           良い論文の出版を遅らせるお馬鹿な人。
     |       (:::::`‐-、__  |::::`、     ヒニニヽ、         あなたの論文が Ann of Math に accept される確率は?
    |      / `‐-、::::::::::`‐-、::::\   /,ニニ、\            それとも最近は Inv. Math. の方が上かしら?
   |      |::::::::::::::::::|` -、:::::::,ヘ ̄|'、  ヒニ二、 \
.   |      /::::::::::::::::::|::::::::\/:::O`、::\   | '、   \
   |      /:::::::::::::::::::/:::::::::::::::::::::::::::::'、::::\ノ  ヽ、  |
  |      |:::::/:::::::::/:::::::::::::::::::::::::::::::::::'、',::::'、  /:\__/‐、
  |      |/:::::::::::/::::::::::::::::::::::::::::::::::O::| '、::| く::::::::::::: ̄|
   |     /_..-'´ ̄`ー-、:::::::::::::::::::::::::::::::::::|/:/`‐'::\;;;;;;;_|
   |    |/::::::::::::::::::::::\:::::::::::::::::::::::::::::|::/::::|::::/:::::::::::/
    |   /:::::::::::::::::::::::::::::::::|:::::::::::::::::::::O::|::|::::::|:::::::::::::::/
897132人目の素数さん:2012/05/05(土) 22:07:32.89
線形写像の同時対角化可能に関する証明がのってるサイトありますかね?
898132人目の素数さん:2012/05/05(土) 22:17:18.95
>>892
V=ℂ²、A=〔〔0,1〕,〔1,0〕〕、f:V∋x→Ax∈V 、U=ℂ×{0} で考えてみよう。
899132人目の素数さん:2012/05/05(土) 22:18:26.08
>>893
スヒヴァックを難しく感じる様だと、先はない
900132人目の素数さん:2012/05/05(土) 22:25:41.11
          __ノ)-'´ ̄ ̄`ー- 、_
        , '´  _. -‐'''"二ニニ=-`ヽ、
      /   /:::::; -‐''"        `ーノ
     /   /:::::/           \
     /    /::::::/          | | |  |
     |   |:::::/ /     |  | | | |  |
      |   |::/ / / |  | ||  | | ,ハ .| ,ハ|
      |   |/ / / /| ,ハノ| /|ノレ,ニ|ル' 
     |   |  | / / レ',二、レ′ ,ィイ|゙/   私は只の数ヲタなんかとは付き合わないわ。
.     |   \ ∠イ  ,イイ|    ,`-' |      頭が良くて数学が出来てかっこいい人。それが必要条件よ。
     |     l^,人|  ` `-'     ゝ  |        さらに Ann.of Math に論文書けば十分条件にもなるわよ。
      |      ` -'\       ー'  人          一番嫌いなのは論文数を増やすためにくだらない論文を書いて
    |        /(l     __/  ヽ、           良い論文の出版を遅らせるお馬鹿な人。
     |       (:::::`‐-、__  |::::`、     ヒニニヽ、         あなたの論文が Ann of Math に accept される確率は?
    |      / `‐-、::::::::::`‐-、::::\   /,ニニ、\            それとも最近は Inv. Math. の方が上かしら?
   |      |::::::::::::::::::|` -、:::::::,ヘ ̄|'、  ヒニ二、 \
.   |      /::::::::::::::::::|::::::::\/:::O`、::\   | '、   \
   |      /:::::::::::::::::::/:::::::::::::::::::::::::::::'、::::\ノ  ヽ、  |
  |      |:::::/:::::::::/:::::::::::::::::::::::::::::::::::'、',::::'、  /:\__/‐、
  |      |/:::::::::::/::::::::::::::::::::::::::::::::::O::| '、::| く::::::::::::: ̄|
   |     /_..-'´ ̄`ー-、:::::::::::::::::::::::::::::::::::|/:/`‐'::\;;;;;;;_|
   |    |/::::::::::::::::::::::\:::::::::::::::::::::::::::::|::/::::|::::/:::::::::::/
    |   /:::::::::::::::::::::::::::::::::|:::::::::::::::::::::O::|::|::::::|:::::::::::::::/
901132人目の素数さん:2012/05/05(土) 22:49:02.00
>>898
! って何ですか?
902132人目の素数さん:2012/05/05(土) 23:09:48.86
z=x^2-y^2

これのグラフの求め方を教えてください!
^2って二乗って意味であってますよね…!?
903132人目の素数さん:2012/05/05(土) 23:14:36.11
方眼紙には書けないので、
まず、粘土を買い求める。
904132人目の素数さん:2012/05/05(土) 23:16:42.02
>>903
xyzのグラフで書く問題です
905132人目の素数さん:2012/05/05(土) 23:21:16.14
ここにゃあテンプレがないのな

高校生のための数学の質問スレPART330
http://uni.2ch.net/test/read.cgi/math/1334845283/

の2あたりに、この手のスレでの記法一覧がある
^はプログラミングではシフト演算を指したりするけど、数学スレじゃあそんな意味は稀

あとは双曲線で調べる
906NAS6 ◆n3AmnVhjwc :2012/05/05(土) 23:38:20.05
n番目の素数の概算値Paは
Pa=2.32nlogn+2n
で大体出せた
907132人目の素数さん:2012/05/05(土) 23:38:33.99

 | |l ̄|
 | |l民|
 | |l主|
 | |l党|
 | |l_|
 |   .|_∧  
 |   .|`∀´>
 |   .⊂ ノ
 |   .| ノ
 ̄ ̄ ̄ ̄ ̄ ̄
民主党とAKBの関係が遂に明かされようとしている
秋元康の後ろには蓮訪
蓮訪の後ろに前原、枝野 そして民主党全体

税金を使ってAKBを宣伝し、その税金が民主党に流れている
AKB自体が、税金目当ての捏造ブーム
908132人目の素数さん:2012/05/06(日) 06:35:54.75
>>902
それは双曲線の基本形
ググると詳しく載ってるサイトが沢山出るよ
909132人目の素数さん:2012/05/06(日) 06:40:03.62
>>902
xyz空間のグラフを紙に投影しろという問題だとしたら絵のセンスが要るかもしれないが,
どんな形か想像するのは難しくない.平面z=c(cは定数)とグラフの共通部分を考えるとわかりやすい.
x^2-y^2=cという方程式が表す曲線が,cを動かしたときどのように変化するかを考えればいい.
x,yを緯度と経度,zを標高と考えれば,この各cに対する曲線はいわゆる等高線を表すことになる.
910132人目の素数さん:2012/05/06(日) 10:10:30.76
一葉双曲面
911132人目の素数さん:2012/05/06(日) 10:53:00.69
関数ι,τ,ρ,α,β,γを
ι=x,τ=1/(1-x),ρ=(x-1)/x,α=1/x,β=1-x,γ=x/(x-1)
とする。
このうち任意の2つの関数の合成関数を考えると、その合成関数は元の関数のいずれかになる。
(1)実際に計算して確かめよ。
(2)どうしてそうなるのか。

(2)がさっぱりです。
τの逆関数がρで、α,β,γの逆関数がそれ自身ってことくらいしか・・・
912132人目の素数さん:2012/05/06(日) 11:14:14.00
          __ノ)-'´ ̄ ̄`ー- 、_
        , '´  _. -‐'''"二ニニ=-`ヽ、
      /   /:::::; -‐''"        `ーノ
     /   /:::::/           \
     /    /::::::/          | | |  |
     |   |:::::/ /     |  | | | |  |
      |   |::/ / / |  | ||  | | ,ハ .| ,ハ|
      |   |/ / / /| ,ハノ| /|ノレ,ニ|ル' 
     |   |  | / / レ',二、レ′ ,ィイ|゙/   私は只の数ヲタなんかとは付き合わないわ。
.     |   \ ∠イ  ,イイ|    ,`-' |      頭が良くて数学が出来てかっこいい人。それが必要条件よ。
     |     l^,人|  ` `-'     ゝ  |        さらに Ann.of Math に論文書けば十分条件にもなるわよ。
      |      ` -'\       ー'  人          一番嫌いなのは論文数を増やすためにくだらない論文を書いて
    |        /(l     __/  ヽ、           良い論文の出版を遅らせるお馬鹿な人。
     |       (:::::`‐-、__  |::::`、     ヒニニヽ、         あなたの論文が Ann of Math に accept される確率は?
    |      / `‐-、::::::::::`‐-、::::\   /,ニニ、\            それとも最近は Inv. Math. の方が上かしら?
   |      |::::::::::::::::::|` -、:::::::,ヘ ̄|'、  ヒニ二、 \
.   |      /::::::::::::::::::|::::::::\/:::O`、::\   | '、   \
   |      /:::::::::::::::::::/:::::::::::::::::::::::::::::'、::::\ノ  ヽ、  |
  |      |:::::/:::::::::/:::::::::::::::::::::::::::::::::::'、',::::'、  /:\__/‐、
  |      |/:::::::::::/::::::::::::::::::::::::::::::::::O::| '、::| く::::::::::::: ̄|
   |     /_..-'´ ̄`ー-、:::::::::::::::::::::::::::::::::::|/:/`‐'::\;;;;;;;_|
   |    |/::::::::::::::::::::::\:::::::::::::::::::::::::::::|::/::::|::::/:::::::::::/
    |   /:::::::::::::::::::::::::::::::::|:::::::::::::::::::::O::|::|::::::|:::::::::::::::/
913仙石18:2012/05/06(日) 11:47:55.57
(あ) 計算したら(2)になった。
(ほ) f(x)=(a x+b)/(c x+d) for ad-bc != 0
  g(x)=(p x+q)/(rx+s) for ps-qr != 0
f(g(x))=((sp+br)x+aq + bs)/((cp+dr)x+cq+ds)
    .....
演算で閉じている


914132人目の素数さん:2012/05/06(日) 12:00:06.73
>>781, >>779 訂正
5個の玉を5個の箱に入れる場合の数は、5^5=3125
このとき、1個の箱だけ空になる場合の数は
空になる箱がどれになるかで、5通り
残り4個の箱のうちどれに2個の玉を入れるかで、4通り
5個の玉を空になる箱ができないように4個の箱に入れる場合の数は
1,1,2,3,4の番号の書いてある5個の玉の並べ方と同じだから、5!/2通り
となるから、5*4*5!/2=1200
求める確率は、1200/3125=48/125
915132人目の素数さん:2012/05/06(日) 12:17:25.60
>>855 あなたが正しいです。
916132人目の素数さん:2012/05/06(日) 12:28:14.28
          __ノ)-'´ ̄ ̄`ー- 、_
        , '´  _. -‐'''"二ニニ=-`ヽ、
      /   /:::::; -‐''"        `ーノ
     /   /:::::/           \
     /    /::::::/          | | |  |
     |   |:::::/ /     |  | | | |  |
      |   |::/ / / |  | ||  | | ,ハ .| ,ハ|
      |   |/ / / /| ,ハノ| /|ノレ,ニ|ル' 
     |   |  | / / レ',二、レ′ ,ィイ|゙/   私は只の数ヲタなんかとは付き合わないわ。
.     |   \ ∠イ  ,イイ|    ,`-' |      頭が良くて数学が出来てかっこいい人。それが必要条件よ。
     |     l^,人|  ` `-'     ゝ  |        さらに Ann.of Math に論文書けば十分条件にもなるわよ。
      |      ` -'\       ー'  人          一番嫌いなのは論文数を増やすためにくだらない論文を書いて
    |        /(l     __/  ヽ、           良い論文の出版を遅らせるお馬鹿な人。
     |       (:::::`‐-、__  |::::`、     ヒニニヽ、         あなたの論文が Ann of Math に accept される確率は?
    |      / `‐-、::::::::::`‐-、::::\   /,ニニ、\            それとも最近は Inv. Math. の方が上かしら?
   |      |::::::::::::::::::|` -、:::::::,ヘ ̄|'、  ヒニ二、 \
.   |      /::::::::::::::::::|::::::::\/:::O`、::\   | '、   \
   |      /:::::::::::::::::::/:::::::::::::::::::::::::::::'、::::\ノ  ヽ、  |
  |      |:::::/:::::::::/:::::::::::::::::::::::::::::::::::'、',::::'、  /:\__/‐、
  |      |/:::::::::::/::::::::::::::::::::::::::::::::::O::| '、::| く::::::::::::: ̄|
   |     /_..-'´ ̄`ー-、:::::::::::::::::::::::::::::::::::|/:/`‐'::\;;;;;;;_|
   |    |/::::::::::::::::::::::\:::::::::::::::::::::::::::::|::/::::|::::/:::::::::::/
    |   /:::::::::::::::::::::::::::::::::|:::::::::::::::::::::O::|::|::::::|:::::::::::::::/
917132人目の素数さん:2012/05/06(日) 12:41:18.96
P(A|B)=P(A∩B)/P(B)
P(B|A)=P(A∩B)/P(A)
P(A)≠P(B)のとき、P(A|B)≠P(B|A)
918132人目の素数さん:2012/05/06(日) 12:47:44.09
          __ノ)-'´ ̄ ̄`ー- 、_
        , '´  _. -‐'''"二ニニ=-`ヽ、
      /   /:::::; -‐''"        `ーノ
     /   /:::::/           \
     /    /::::::/          | | |  |
     |   |:::::/ /     |  | | | |  |
      |   |::/ / / |  | ||  | | ,ハ .| ,ハ|
      |   |/ / / /| ,ハノ| /|ノレ,ニ|ル' 
     |   |  | / / レ',二、レ′ ,ィイ|゙/   私は只の数ヲタなんかとは付き合わないわ。
.     |   \ ∠イ  ,イイ|    ,`-' |      頭が良くて数学が出来てかっこいい人。それが必要条件よ。
     |     l^,人|  ` `-'     ゝ  |        さらに Ann.of Math に論文書けば十分条件にもなるわよ。
      |      ` -'\       ー'  人          一番嫌いなのは論文数を増やすためにくだらない論文を書いて
    |        /(l     __/  ヽ、           良い論文の出版を遅らせるお馬鹿な人。
     |       (:::::`‐-、__  |::::`、     ヒニニヽ、         あなたの論文が Ann of Math に accept される確率は?
    |      / `‐-、::::::::::`‐-、::::\   /,ニニ、\            それとも最近は Inv. Math. の方が上かしら?
   |      |::::::::::::::::::|` -、:::::::,ヘ ̄|'、  ヒニ二、 \
.   |      /::::::::::::::::::|::::::::\/:::O`、::\   | '、   \
   |      /:::::::::::::::::::/:::::::::::::::::::::::::::::'、::::\ノ  ヽ、  |
  |      |:::::/:::::::::/:::::::::::::::::::::::::::::::::::'、',::::'、  /:\__/‐、
  |      |/:::::::::::/::::::::::::::::::::::::::::::::::O::| '、::| く::::::::::::: ̄|
   |     /_..-'´ ̄`ー-、:::::::::::::::::::::::::::::::::::|/:/`‐'::\;;;;;;;_|
   |    |/::::::::::::::::::::::\:::::::::::::::::::::::::::::|::/::::|::::/:::::::::::/
    |   /:::::::::::::::::::::::::::::::::|:::::::::::::::::::::O::|::|::::::|:::::::::::::::/
919132人目の素数さん:2012/05/06(日) 12:48:37.75
「supE=α<∞ とする。このとき、Eが最大値を持つための条件は、α∈Eである。」

どなたか証明して下さい。
920132人目の素数さん:2012/05/06(日) 13:19:11.55
αがEの最大値とは次の(1)と(2)が成立すること。(定義)
(1)∀x∈Eに対してx≦α
(2)α∈E

(証明)
仮定よりsupE=α<∞だから上限の定義より(1)が成立。
よってα∈EならばEは最大値αをもつ。
921132人目の素数さん:2012/05/06(日) 13:30:41.66
集積点が含まれてなきゃコンプリートじゃないので
922132人目の素数さん:2012/05/06(日) 13:32:43.63
反例
E<1、0。99999。。。<1、supE=1
0。9999。。。=1
923132人目の素数さん:2012/05/06(日) 15:04:47.08
雑魚の分際で何を言うw
924132人目の素数さん:2012/05/06(日) 15:55:22.45
線形台数って何台の事ですかね
925132人目の素数さん:2012/05/06(日) 16:00:05.20
1M
926132人目の素数さん:2012/05/06(日) 16:03:50.34
単連結が基準点に依らないことを示せってどうやるんですか?
927132人目の素数さん:2012/05/06(日) 16:06:10.06
ミス 基準点の取り方に依らないこと
928132人目の素数さん:2012/05/06(日) 16:28:52.95
          __ノ)-'´ ̄ ̄`ー- 、_
        , '´  _. -‐'''"二ニニ=-`ヽ、
      /   /:::::; -‐''"        `ーノ
     /   /:::::/           \
     /    /::::::/          | | |  |
     |   |:::::/ /     |  | | | |  |
      |   |::/ / / |  | ||  | | ,ハ .| ,ハ|
      |   |/ / / /| ,ハノ| /|ノレ,ニ|ル' 
     |   |  | / / レ',二、レ′ ,ィイ|゙/   私は只の数ヲタなんかとは付き合わないわ。
.     |   \ ∠イ  ,イイ|    ,`-' |      頭が良くて数学が出来てかっこいい人。それが必要条件よ。
     |     l^,人|  ` `-'     ゝ  |        さらに Ann.of Math に論文書けば十分条件にもなるわよ。
      |      ` -'\       ー'  人          一番嫌いなのは論文数を増やすためにくだらない論文を書いて
    |        /(l     __/  ヽ、           良い論文の出版を遅らせるお馬鹿な人。
     |       (:::::`‐-、__  |::::`、     ヒニニヽ、         あなたの論文が Ann of Math に accept される確率は?
    |      / `‐-、::::::::::`‐-、::::\   /,ニニ、\            それとも最近は Inv. Math. の方が上かしら?
   |      |::::::::::::::::::|` -、:::::::,ヘ ̄|'、  ヒニ二、 \
.   |      /::::::::::::::::::|::::::::\/:::O`、::\   | '、   \
   |      /:::::::::::::::::::/:::::::::::::::::::::::::::::'、::::\ノ  ヽ、  |
  |      |:::::/:::::::::/:::::::::::::::::::::::::::::::::::'、',::::'、  /:\__/‐、
  |      |/:::::::::::/::::::::::::::::::::::::::::::::::O::| '、::| く::::::::::::: ̄|
   |     /_..-'´ ̄`ー-、:::::::::::::::::::::::::::::::::::|/:/`‐'::\;;;;;;;_|
   |    |/::::::::::::::::::::::\:::::::::::::::::::::::::::::|::/::::|::::/:::::::::::/
    |   /:::::::::::::::::::::::::::::::::|:::::::::::::::::::::O::|::|::::::|:::::::::::::::/
929132人目の素数さん:2012/05/06(日) 18:57:30.14
偏微分(熱伝導方程式)の問題なのですが、

v(x,t)=(√4πk/t)exp(x^2/4kt)u(x/t,1/t)
について、
dv/dt,dv/dtを求めよ

という問題で、u(x/t,1/t)
の部分をどう処理したらよいのかわかりません。助けてください。
930132人目の素数さん:2012/05/06(日) 20:34:09.70
おれもわからん
微分可能かも偏微分可能かもそもそも連続かも
931132人目の素数さん:2012/05/06(日) 20:37:21.65
> dv/dt,dv/dtを求めよ
俺には両方の微分が同じようにも見えるけど最近目が付かれているから
オレには分からないようななにか微量の違いがあるのかもしれない
932132人目の素数さん:2012/05/06(日) 21:03:32.48
>>929です。
>>931さんすみません。
ひとつはdv/dxでした。

全文をかくと、
u(x,t)が熱伝導方程式du/dt-k(d^2u/dx^2)=0の解であるとき、v(x,t)を

偏微分(熱伝導方程式)の問題なのですが、

v(x,t)=(√4πk/t)exp(x^2/4kt)u(x/t,1/t)
と定義する。
dv/dt,dv/dxを求めよ。

です。
933132人目の素数さん:2012/05/06(日) 21:12:29.64
>>932
出てくる微分記号は全部∂でなくdなのか?
934132人目の素数さん:2012/05/06(日) 21:13:23.86
すみません∂です。
935132人目の素数さん:2012/05/06(日) 21:13:48.41
936132人目の素数さん:2012/05/06(日) 21:25:16.36
1,A={(x,y)∈R^2 | 0<x^2+y^2<1}と平面R^2が対等であることを示せ
2, i≠j ⇒ M(i)≠M(j) かつ N=∪(i=1~∞) M(i)を満たす無限部分集合M(i)⊂N(i=1,2,・・・)が存在することを示せ(Nは自然数全体の集合)
3,集合Aが無限集合であることとAと対等である真部分集合B?Aが存在することは必要十分であることを示せ

よろしくお願いします
937132人目の素数さん:2012/05/06(日) 21:30:08.99
>>936 訂正
2
i≠j⇒M(i)∩M(j)= Φ ・・・

3
真部分集合B⊆A
938132人目の素数さん:2012/05/06(日) 21:32:13.32
>>936-937更に訂正
真部分集合B⊊A
939132人目の素数さん:2012/05/07(月) 01:09:47.80
>>932です。
本当にお願いします。
思いついたこととか、ヒントでも結構です。
940132人目の素数さん:2012/05/07(月) 01:14:03.85
課題をはやくやらなかったお前が悪い
941132人目の素数さん:2012/05/07(月) 01:26:38.93
>>939
合成関数 偏微分 でぐぐれ
942132人目の素数さん:2012/05/07(月) 01:31:30.88
>>936の2取り下げで1と3お願いしますー
943132人目の素数さん:2012/05/07(月) 01:38:17.07
課題をはやくやらなかったお前が悪い
944132人目の素数さん:2012/05/07(月) 01:44:42.33
          __ノ)-'´ ̄ ̄`ー- 、_
        , '´  _. -‐'''"二ニニ=-`ヽ、
      /   /:::::; -‐''"        `ーノ
     /   /:::::/           \
     /    /::::::/          | | |  |
     |   |:::::/ /     |  | | | |  |
      |   |::/ / / |  | ||  | | ,ハ .| ,ハ|
      |   |/ / / /| ,ハノ| /|ノレ,ニ|ル' 
     |   |  | / / レ',二、レ′ ,ィイ|゙/   私は只の数ヲタなんかとは付き合わないわ。
.     |   \ ∠イ  ,イイ|    ,`-' |      頭が良くて数学が出来てかっこいい人。それが必要条件よ。
     |     l^,人|  ` `-'     ゝ  |        さらに Ann.of Math に論文書けば十分条件にもなるわよ。
      |      ` -'\       ー'  人          一番嫌いなのは論文数を増やすためにくだらない論文を書いて
    |        /(l     __/  ヽ、           良い論文の出版を遅らせるお馬鹿な人。
     |       (:::::`‐-、__  |::::`、     ヒニニヽ、         あなたの論文が Ann of Math に accept される確率は?
    |      / `‐-、::::::::::`‐-、::::\   /,ニニ、\            それとも最近は Inv. Math. の方が上かしら?
   |      |::::::::::::::::::|` -、:::::::,ヘ ̄|'、  ヒニ二、 \
.   |      /::::::::::::::::::|::::::::\/:::O`、::\   | '、   \
   |      /:::::::::::::::::::/:::::::::::::::::::::::::::::'、::::\ノ  ヽ、  |
  |      |:::::/:::::::::/:::::::::::::::::::::::::::::::::::'、',::::'、  /:\__/‐、
  |      |/:::::::::::/::::::::::::::::::::::::::::::::::O::| '、::| く::::::::::::: ̄|
   |     /_..-'´ ̄`ー-、:::::::::::::::::::::::::::::::::::|/:/`‐'::\;;;;;;;_|
   |    |/::::::::::::::::::::::\:::::::::::::::::::::::::::::|::/::::|::::/:::::::::::/
    |   /:::::::::::::::::::::::::::::::::|:::::::::::::::::::::O::|::|::::::|:::::::::::::::/
945132人目の素数さん:2012/05/07(月) 08:09:40.99
課題とは何ですか
946132人目の素数さん:2012/05/07(月) 09:44:57.95
課題とは、人はどのように生きどのように善を積むかという問いのこと
947132人目の素数さん:2012/05/07(月) 10:30:10.39
とてもおおきいこと
948132人目の素数さん:2012/05/07(月) 12:01:37.34
それは過大やー
949132人目の素数さん:2012/05/07(月) 12:24:00.90
香川大学は?
950132人目の素数さん:2012/05/07(月) 12:32:43.25
          __ノ)-'´ ̄ ̄`ー- 、_
        , '´  _. -‐'''"二ニニ=-`ヽ、
      /   /:::::; -‐''"        `ーノ
     /   /:::::/           \
     /    /::::::/          | | |  |
     |   |:::::/ /     |  | | | |  |
      |   |::/ / / |  | ||  | | ,ハ .| ,ハ|
      |   |/ / / /| ,ハノ| /|ノレ,ニ|ル' 
     |   |  | / / レ',二、レ′ ,ィイ|゙/   私は只の数ヲタなんかとは付き合わないわ。
.     |   \ ∠イ  ,イイ|    ,`-' |      頭が良くて数学が出来てかっこいい人。それが必要条件よ。
     |     l^,人|  ` `-'     ゝ  |        さらに Ann.of Math に論文書けば十分条件にもなるわよ。
      |      ` -'\       ー'  人          一番嫌いなのは論文数を増やすためにくだらない論文を書いて
    |        /(l     __/  ヽ、           良い論文の出版を遅らせるお馬鹿な人。
     |       (:::::`‐-、__  |::::`、     ヒニニヽ、         あなたの論文が Ann of Math に accept される確率は?
    |      / `‐-、::::::::::`‐-、::::\   /,ニニ、\            それとも最近は Inv. Math. の方が上かしら?
   |      |::::::::::::::::::|` -、:::::::,ヘ ̄|'、  ヒニ二、 \
.   |      /::::::::::::::::::|::::::::\/:::O`、::\   | '、   \
   |      /:::::::::::::::::::/:::::::::::::::::::::::::::::'、::::\ノ  ヽ、  |
  |      |:::::/:::::::::/:::::::::::::::::::::::::::::::::::'、',::::'、  /:\__/‐、
  |      |/:::::::::::/::::::::::::::::::::::::::::::::::O::| '、::| く::::::::::::: ̄|
   |     /_..-'´ ̄`ー-、:::::::::::::::::::::::::::::::::::|/:/`‐'::\;;;;;;;_|
   |    |/::::::::::::::::::::::\:::::::::::::::::::::::::::::|::/::::|::::/:::::::::::/
    |   /:::::::::::::::::::::::::::::::::|:::::::::::::::::::::O::|::|::::::|:::::::::::::::/
951132人目の素数さん:2012/05/07(月) 17:22:13.67
>>936
1
例えば、Aの元を(rcosθ,rsinθ)と極座標表示してrを適当にいじくれば、AとR^2-{0}の間に全単射ができる。
R^2-{0}とR^2は対等(要証明)だからOK。

3
Aが無限集合なら、点列{a_n}[n=1,∞]⊂Aを、どの2つも異なるようにとれる。
これを使ってAとA-{a_1}の間に全単射を作る。
逆は背理法。
952132人目の素数さん:2012/05/07(月) 21:00:30.77
          __ノ)-'´ ̄ ̄`ー- 、_
        , '´  _. -‐'''"二ニニ=-`ヽ、
      /   /:::::; -‐''"        `ーノ
     /   /:::::/           \
     /    /::::::/          | | |  |
     |   |:::::/ /     |  | | | |  |
      |   |::/ / / |  | ||  | | ,ハ .| ,ハ|
      |   |/ / / /| ,ハノ| /|ノレ,ニ|ル' 
     |   |  | / / レ',二、レ′ ,ィイ|゙/   私は只の数ヲタなんかとは付き合わないわ。
.     |   \ ∠イ  ,イイ|    ,`-' |      頭が良くて数学が出来てかっこいい人。それが必要条件よ。
     |     l^,人|  ` `-'     ゝ  |        さらに Ann.of Math に論文書けば十分条件にもなるわよ。
      |      ` -'\       ー'  人          一番嫌いなのは論文数を増やすためにくだらない論文を書いて
    |        /(l     __/  ヽ、           良い論文の出版を遅らせるお馬鹿な人。
     |       (:::::`‐-、__  |::::`、     ヒニニヽ、         あなたの論文が Ann of Math に accept される確率は?
    |      / `‐-、::::::::::`‐-、::::\   /,ニニ、\            それとも最近は Inv. Math. の方が上かしら?
   |      |::::::::::::::::::|` -、:::::::,ヘ ̄|'、  ヒニ二、 \
.   |      /::::::::::::::::::|::::::::\/:::O`、::\   | '、   \
   |      /:::::::::::::::::::/:::::::::::::::::::::::::::::'、::::\ノ  ヽ、  |
  |      |:::::/:::::::::/:::::::::::::::::::::::::::::::::::'、',::::'、  /:\__/‐、
  |      |/:::::::::::/::::::::::::::::::::::::::::::::::O::| '、::| く::::::::::::: ̄|
   |     /_..-'´ ̄`ー-、:::::::::::::::::::::::::::::::::::|/:/`‐'::\;;;;;;;_|
   |    |/::::::::::::::::::::::\:::::::::::::::::::::::::::::|::/::::|::::/:::::::::::/
    |   /:::::::::::::::::::::::::::::::::|:::::::::::::::::::::O::|::|::::::|:::::::::::::::/
953132人目の素数さん:2012/05/07(月) 21:01:47.79
5×5行列で

-1 0 1 2 3
3 -1 0 1 2
2 3 -1 0 1
0 0 0 1 2
0 0 0 2 3

なんですが、答えは10で合ってるんでしょうか?
954132人目の素数さん:2012/05/07(月) 21:02:54.54
問題文を正確に。
955132人目の素数さん:2012/05/07(月) 21:19:34.34
行列がありました、だけじゃ問題じゃないな
956132人目の素数さん:2012/05/07(月) 21:25:48.16
|-1 0 1 2 3|
|3 -1 0 1 2|
|2 3 -1 0 1| の行列式はいくつなんでしょうか?
|0 0 0 1 2 |
|0 0 0 2 3 |

-10って出たんですが、合ってますか
957132人目の素数さん:2012/05/07(月) 21:33:12.09
958132人目の素数さん:2012/05/08(火) 00:26:04.73
任意の実数xについてf”(x)>0を満たす関数f(x)がある
点A(a,b)から曲線y=f(x)に少なくとも1本の接線を引くことができるならば
b≦f(a)が成り立つことを示せ

お願いします
959132人目の素数さん:2012/05/08(火) 03:22:35.90
>>958
接線の方程式を y=g(x) とすると、これは直線だから g"=0
h(x)=f(x)−g(x) とすると、h"=f"−g"=f">0 で x 軸に接しているから常に h(x)≧0
すなわち f(x)≧g(x)。点 (a,b) は接線上の点だから f(a)≧g(a)=b。
960132人目の素数さん:2012/05/08(火) 03:32:08.97
>>953
暗算で3角行列にすると対角成分が −1,−1,10,1,−1 になるから −10 で合ってる。
961132人目の素数さん:2012/05/08(火) 04:56:53.01
2つの円柱面y^2+z^2=a^2とx^2+z^2=a^2で囲まれる部分の面積を求めよ

という問題がわかりません
よろしくお願いします
962132人目の素数さん:2012/05/08(火) 05:58:13.14
y^2+z^2=a^2からy^2=a^2-z^2、つまりy=±√(a^2-z^2)
x^2+z^2=a^2からx^2=a^2-z^2、つまりx=±√(a^2-z^2)
-a<z<aなる各zにおいて、囲まれる部分は正方形になっている

∫[-a,a]{8√(a^2-z^2)}dz
あとは積分を頑張る

…寝ぼけてミスっていませんように
963132人目の素数さん:2012/05/08(火) 06:09:26.96
>>962
ありがとうございます

積分を頑張ってみたのですが
4z√(a^2-z^2) + 4a^2 arctan(z/√(a^2-z^2))となるのですが大丈夫でしょうか
zに-a,aを代入すると大変なことになってしまいます
964132人目の素数さん:2012/05/08(火) 07:04:11.69
S=∂V/∂a
965132人目の素数さん:2012/05/08(火) 07:12:07.53
×S=∂V/∂a
○S=dV/da
966132人目の素数さん:2012/05/08(火) 07:17:31.20
>>963
> arctan(z/√(a^2-z^2))
z→a-0,-a+0の極限をとってみる、または、z=a*sin(t)と置いてみる
967132人目の素数さん:2012/05/08(火) 13:22:52.54
わかりません、
すみませんがお願いします><

次の境界条件のもとでの1次元ラプラス作用素の固定値問題を、負の固定値がないことを前提に解きなさい。
{X"(x)+λX(x)=0 , -L<x<L
{X(-L)=X(L) , X'(-L)=X'(L)

よろしくお願いします。
何かヒントでもかまいません。
968132人目の素数さん:2012/05/08(火) 15:00:48.82
>>967
λの符号で場合分けして X"(x)+λX(x)=0 の一般解を求めて、
境界条件を満たすように積分定数とλの値を決める。
ところで「固定値」とは固有値のこと?
969132人目の素数さん:2012/05/08(火) 15:02:55.99
>>968
ありがとうございます!
はい、すみません固有値のことです^^;
970片山博文MZ ◆0lBZNi.Q7evd :2012/05/08(火) 15:06:05.33
p=|V(G)|=|V(G')|を満たす有向グラフG,G'について、
g,g'がそれぞれG,G'の誘導部分グラフとなり、g,g'がグラフ同型となり、
|V(g)|≦nとなる(g,g')の個数をPM(G,G',n)と表す(空グラフも含む)。
G,G'とPM(G,G',n)にどのような性質があるかを調べなさい。
例)G,G'がグラフ同型ならば、PM(G,G',p)=2^p。
よろしくお願いします。
971132人目の素数さん:2012/05/08(火) 16:22:50.47
複素関数fについて
I=〔0、m+1/2〕とします
xがI上を動く時、|f(x+iy)|はy→∞の時、一様に0に収束するとします
xがIを動く時
cotπ(x+iy)はy→∞の時、-iに一様収束していますが
xがIを動く時|f(x+iy)cotπ(x+iy)|は0に一様収束すると言えますか?
972132人目の素数さん:2012/05/08(火) 17:16:52.40
任意のε>0に対して、あるm>0が存在して
y>mならば|f(x+iy)|<εかつ|cotπ(x+iy) + i|<ε

|f(x+iy)cotπ(x+iy)| = |f(x+iy)cotπ(x+iy) - f(x+iy)(-i) + f(x+iy)(-i)|
≦|f(x+iy)cotπ(x+iy) - f(x+iy)(-i)|+|f(x+iy)|
=|f(x+iy)||cotπ(x+iy) + i| + |f(x+iy)|
<ε^2 + ε
973132人目の素数さん:2012/05/08(火) 17:52:32.65
>>972
ありがとうございました
一般に一様収束してる関数の積は一様収束するのでしょうか?
974132人目の素数さん:2012/05/08(火) 18:39:22.06
なあ、モノにあたった光が反射して、網膜に届くから目が見えるんだよな?
でも光っていろんな方向に反射するし、いろんな方向から飛んでくるよな?
だから、網膜の1点Pに、点Xから反射してきた光と、点Yから反射してきた光が同時(実際にはふたつだけじゃなく、もっと多いだろう)に当たったら、そこではふたつのモノが同時に見えてるわけじゃん?
1点Pの取り方は任意だったから、網膜全体としても当然、光の受け方は乱雑になって、きちんとモノが見えないと思うんだけど……
975132人目の素数さん:2012/05/08(火) 18:40:11.98
>>974
中学生からやり直せ
976132人目の素数さん:2012/05/08(火) 18:44:00.71
>>974
お前のいう通りだ
机の上にただ紙おいても白いままだろ?
977132人目の素数さん:2012/05/08(火) 18:44:35.20
>>974
レンズって知ってるか?
978132人目の素数さん:2012/05/08(火) 19:10:45.43
1対1 のUの分野で質問なんですが


f(x)=x^2-ax+5,g(x)=ax-1がある。
y=g(x)がy=f(x)によって切り取られる線分の長さが2√30 のときaを求めよ。


という問題で一般に求める線分の長さは[√(1+m^2)](β-α)

mは傾き α βはg(x)f(x)の交点

でいまいち意味がわかりませんどうしてこうなるのか教えてください。
979132人目の素数さん:2012/05/08(火) 19:15:09.96
端点の座標を求めて、三平方の定理

一般に(x(t),y(t))とパラメータ付けされた曲線の、t=aからbの間の長さは、∫[a→b]√((dx/dt)^2+(dy/dt)^2)dt
980132人目の素数さん:2012/05/08(火) 19:38:06.48
          __ノ)-'´ ̄ ̄`ー- 、_
        , '´  _. -‐'''"二ニニ=-`ヽ、
      /   /:::::; -‐''"        `ーノ
     /   /:::::/           \
     /    /::::::/          | | |  |
     |   |:::::/ /     |  | | | |  |
      |   |::/ / / |  | ||  | | ,ハ .| ,ハ|
      |   |/ / / /| ,ハノ| /|ノレ,ニ|ル' 
     |   |  | / / レ',二、レ′ ,ィイ|゙/   私は只の数ヲタなんかとは付き合わないわ。
.     |   \ ∠イ  ,イイ|    ,`-' |      頭が良くて数学が出来てかっこいい人。それが必要条件よ。
     |     l^,人|  ` `-'     ゝ  |        さらに Ann.of Math に論文書けば十分条件にもなるわよ。
      |      ` -'\       ー'  人          一番嫌いなのは論文数を増やすためにくだらない論文を書いて
    |        /(l     __/  ヽ、           良い論文の出版を遅らせるお馬鹿な人。
     |       (:::::`‐-、__  |::::`、     ヒニニヽ、         あなたの論文が Ann of Math に accept される確率は?
    |      / `‐-、::::::::::`‐-、::::\   /,ニニ、\            それとも最近は Inv. Math. の方が上かしら?
   |      |::::::::::::::::::|` -、:::::::,ヘ ̄|'、  ヒニ二、 \
.   |      /::::::::::::::::::|::::::::\/:::O`、::\   | '、   \
   |      /:::::::::::::::::::/:::::::::::::::::::::::::::::'、::::\ノ  ヽ、  |
  |      |:::::/:::::::::/:::::::::::::::::::::::::::::::::::'、',::::'、  /:\__/‐、
  |      |/:::::::::::/::::::::::::::::::::::::::::::::::O::| '、::| く::::::::::::: ̄|
   |     /_..-'´ ̄`ー-、:::::::::::::::::::::::::::::::::::|/:/`‐'::\;;;;;;;_|
   |    |/::::::::::::::::::::::\:::::::::::::::::::::::::::::|::/::::|::::/:::::::::::/
    |   /:::::::::::::::::::::::::::::::::|:::::::::::::::::::::O::|::|::::::|:::::::::::::::/
981132人目の素数さん:2012/05/08(火) 20:31:28.88
>>974
その複数の場所の光が同時にあたってる状態がピンボケ
982132人目の素数さん:2012/05/08(火) 21:25:27.38
大学生の下宿生の割合を調べるために学生500人の中から50人を調査した。
その結果、40人が下宿生だった。
このとき、下宿比率の誤差の範囲が 0.1 以下になるようなサンプル数を求めよ。
983132人目の素数さん:2012/05/08(火) 21:29:00.32
          __ノ)-'´ ̄ ̄`ー- 、_
        , '´  _. -‐'''"二ニニ=-`ヽ、
      /   /:::::; -‐''"        `ーノ
     /   /:::::/           \
     /    /::::::/          | | |  |
     |   |:::::/ /     |  | | | |  |
      |   |::/ / / |  | ||  | | ,ハ .| ,ハ|
      |   |/ / / /| ,ハノ| /|ノレ,ニ|ル' 
     |   |  | / / レ',二、レ′ ,ィイ|゙/   私は只の数ヲタなんかとは付き合わないわ。
.     |   \ ∠イ  ,イイ|    ,`-' |      頭が良くて数学が出来てかっこいい人。それが必要条件よ。
     |     l^,人|  ` `-'     ゝ  |        さらに Ann.of Math に論文書けば十分条件にもなるわよ。
      |      ` -'\       ー'  人          一番嫌いなのは論文数を増やすためにくだらない論文を書いて
    |        /(l     __/  ヽ、           良い論文の出版を遅らせるお馬鹿な人。
     |       (:::::`‐-、__  |::::`、     ヒニニヽ、         あなたの論文が Ann of Math に accept される確率は?
    |      / `‐-、::::::::::`‐-、::::\   /,ニニ、\            それとも最近は Inv. Math. の方が上かしら?
   |      |::::::::::::::::::|` -、:::::::,ヘ ̄|'、  ヒニ二、 \
.   |      /::::::::::::::::::|::::::::\/:::O`、::\   | '、   \
   |      /:::::::::::::::::::/:::::::::::::::::::::::::::::'、::::\ノ  ヽ、  |
  |      |:::::/:::::::::/:::::::::::::::::::::::::::::::::::'、',::::'、  /:\__/‐、
  |      |/:::::::::::/::::::::::::::::::::::::::::::::::O::| '、::| く::::::::::::: ̄|
   |     /_..-'´ ̄`ー-、:::::::::::::::::::::::::::::::::::|/:/`‐'::\;;;;;;;_|
   |    |/::::::::::::::::::::::\:::::::::::::::::::::::::::::|::/::::|::::/:::::::::::/
    |   /:::::::::::::::::::::::::::::::::|:::::::::::::::::::::O::|::|::::::|:::::::::::::::/
984132人目の素数さん:2012/05/08(火) 22:05:31.24
以下の問題がわかりません。どなたか出来る方お願いします。

x+y+z=x'+y'+z'
x^2+y^2+z^2=x'^2+y'^2+z'^2
x^3+y^3+z^3=x'^3+y'^3+z'^3

を満たすとき、集合として
{x,y,z}={x',y',z'}
となることを示せ。

二変数のときは無理矢理出来たのですが、
何か上手いやり方はないでしょうか。

また一般にn変数のときも成り立つと思うのですが
証明がわかりません。

よろしくお願いします。
985132人目の素数さん:2012/05/08(火) 22:12:18.70
          __ノ)-'´ ̄ ̄`ー- 、_
        , '´  _. -‐'''"二ニニ=-`ヽ、
      /   /:::::; -‐''"        `ーノ
     /   /:::::/           \
     /    /::::::/          | | |  |
     |   |:::::/ /     |  | | | |  |
      |   |::/ / / |  | ||  | | ,ハ .| ,ハ|
      |   |/ / / /| ,ハノ| /|ノレ,ニ|ル' 
     |   |  | / / レ',二、レ′ ,ィイ|゙/   私は只の数ヲタなんかとは付き合わないわ。
.     |   \ ∠イ  ,イイ|    ,`-' |      頭が良くて数学が出来てかっこいい人。それが必要条件よ。
     |     l^,人|  ` `-'     ゝ  |        さらに Ann.of Math に論文書けば十分条件にもなるわよ。
      |      ` -'\       ー'  人          一番嫌いなのは論文数を増やすためにくだらない論文を書いて
    |        /(l     __/  ヽ、           良い論文の出版を遅らせるお馬鹿な人。
     |       (:::::`‐-、__  |::::`、     ヒニニヽ、         あなたの論文が Ann of Math に accept される確率は?
    |      / `‐-、::::::::::`‐-、::::\   /,ニニ、\            それとも最近は Inv. Math. の方が上かしら?
   |      |::::::::::::::::::|` -、:::::::,ヘ ̄|'、  ヒニ二、 \
.   |      /::::::::::::::::::|::::::::\/:::O`、::\   | '、   \
   |      /:::::::::::::::::::/:::::::::::::::::::::::::::::'、::::\ノ  ヽ、  |
  |      |:::::/:::::::::/:::::::::::::::::::::::::::::::::::'、',::::'、  /:\__/‐、
  |      |/:::::::::::/::::::::::::::::::::::::::::::::::O::| '、::| く::::::::::::: ̄|
   |     /_..-'´ ̄`ー-、:::::::::::::::::::::::::::::::::::|/:/`‐'::\;;;;;;;_|
   |    |/::::::::::::::::::::::\:::::::::::::::::::::::::::::|::/::::|::::/:::::::::::/
    |   /:::::::::::::::::::::::::::::::::|:::::::::::::::::::::O::|::|::::::|:::::::::::::::/
986132人目の素数さん:2012/05/08(火) 22:59:18.19
6÷2(1+2)=?
っていうスレがありますがこの答えは結局なんなんですか?
6÷2(1+2)
=6×1/2(1+2)
=6×1/6
=1
違いますか?
987132人目の素数さん:2012/05/08(火) 23:11:29.31
>>986
気分次第
988986:2012/05/08(火) 23:26:30.59
ありがとうございます
これがもしも1でなかったら除法を乗法に改めることが出来なくなって代数学が困ったことになるような気分なので1ということにしておきます
989132人目の素数さん:2012/05/08(火) 23:26:33.86
>>986
割り算掛け算は左から順に計算するというルールがあるので、
6÷2を先に計算するのが正しい。
ただ、÷を表記しておきながら×を省略するという表記は通常しないので、
そもそも問題自体がルール違反とも言える。
そういう意地悪引っかけ問題だよ。
990132人目の素数さん:2012/05/08(火) 23:32:01.73
>>984
文字は全部実数とか複素数として
両辺を2乗したり3乗したり足したり引いたりして
x+y+z=x'+y'+z'
xy+yz+zx=x'y'+y'z'+z'x'
xyz=x'y'z'
を示す。
これより{x,y,z}と{x',y',z'}は共に同じ三次方程式の解の集合となる。

一般の場合も同様にできそうだが自分で考えれ
991132人目の素数さん:2012/05/08(火) 23:40:27.98
>>989
ありがとうございます
しかし除法はともかく乗法を左から順に計算するルールというのはおかしくありませんか?
なぜなら乗法は交換法則によってかける順序を変えられるからです
さらに乗法は除法に改められるので、上の式のようになると思うのですがどうでしょうか
992132人目の素数さん:2012/05/08(火) 23:44:10.55
>÷を表記しておきながら×を省略するという表記は通常しない

残念ながらそんなことはなく、「2x^2y÷xyを計算せよ」みたいな問題は
中学の本には載っている。解答には、正解は2xとある。

もちろんこんなのはルールを無視した糞問だと思う。
993132人目の素数さん:2012/05/08(火) 23:49:46.40
【問1】正の整数5桁の数値を記号列とみなすときのアルファベットΣを示せ。

【問2】Σ={1,2,3,4,5}上の関係R={(1,2),(3,4),(5,6)}に対する推移的閉包R+tp推移的かつ反射的閉包R*を示せ。

【問3】アルファベットA={10,20,30} ,B={15,25,35} とするときの集合に関する演算(AとBの和、AとBの共通部分、
AとBの差、AとBの直積、Aのべき集合、Bのべき集合)をそれぞれ示せ。

m(_ _)mおねがいします
994132人目の素数さん:2012/05/09(水) 00:08:19.07
年金数理もここで聞いても大丈夫ですか?

計算式を求めてほしいのですが
995132人目の素数さん:2012/05/09(水) 00:12:26.91
>>994
【生保・損保】アクチュアリー資格試験スレ【年金】
http://uni.2ch.net/test/read.cgi/math/1307326794/
996132人目の素数さん:2012/05/09(水) 00:17:52.57
>>995ありがとうございます。
本来公表してないのを求めようとしているので
そこできくのは少しマズいのですが
やはり保険数理と数学は違うのですね。
997132人目の素数さん:2012/05/09(水) 00:24:45.40
公表してはいけないことを聞くのはどこでもまずいだろ
倫理規程にひっかかるだろ
998132人目の素数さん:2012/05/09(水) 00:25:27.76
>>990

ありがとうございます。
助かりました!
999132人目の素数さん:2012/05/09(水) 00:26:18.99
>>998
一般の場合は、とても一筋縄じゃいかなさそうだ。
1000132人目の素数さん:2012/05/09(水) 00:26:57.61
          ,__
        o'⌒) `ヽ
         (;゙;`゙☆;゙)
          (。・∞・) 願い事を3つまで叶えろ・・・
          ( ∽)
            )ノ         _
          (_          /〜ヽ
          [il=li]       <('A` 。) えっ!俺が叶えんの ?
          )=(_        (  )>
         (-==-)       <^ヾ_
          `ー‐''
10011001
このスレッドは1000を超えました。
もう書けないので、新しいスレッドを立ててくださいです。。。